Download as pdf or txt
Download as pdf or txt
You are on page 1of 277

Nguyn Hu in

OLYMPIC TON NM 2000

.c

(Tp 1)

52 THI V LI GII

NH XUT BN GIO DC

.c

Li ni u
th gi lnh lamdethi.sty ti bin son mt s ton thi Olympic, m
cc hc tr ca ti lm bi tp khi hc tp LATEX. ph v cc bn ham
hc ton ti thu thp v gom li thnh cc sch in t, cc bn c th tham

kho. Mi tp ti s gom khong 50 bi vi li gii. Tp ny c s ng gp ca


Bi Th Anh, V Th Hng Hnh, Cao Th Mai Len, T Xun Ha, Nguyn
Th Loan, Nguyn Th Qu Su, Nguyn Th nh, Nguyn ngc Long.

.c

Rt nhiu bi ton dch khng c chun, nhiu im khng hon ton


chnh xc vy mong bn c t ngm ngh v tm hiu ly. Nhng y l ngun

ti liu ting Vit v ch ny, ti c xem qua v ngi dch l chuyn v


ngnh Ton ph thng. Bn c th tham kho li trong [1].

Rt nhiu on v mi hc TeX nn cu trc v b tr cn xu, ti khng


H Ni, ngy 2 thng 1 nm 2010
Nguyn Hu in

c thi gian sa li, mong cc bn thng cm.

51
89/176-05
GD-05

M s: 8I092M5

Mc lc
3

Mc lc . . . . . . . . . . . . . . . . . . . . . . . . . . . . . . . . . . . . . . . . . . . . . . .

Chng 1. thi olympic Belarus . . . . . . . . . . . . . . . . . . . . . . . . .

Chng 2. thi olympic Bungari . . . . . . . . . . . . . . . . . . . . . . . .

16

Li ni u . . . . . . . . . . . . . . . . . . . . . . . . . . . . . . . . . . . . . . . . . . . .

29

Chng 4. thi olympic Trung Quc . . . . . . . . . . . . . . . . . . . .

32

.c

Chng 3. thi olympic Canada . . . . . . . . . . . . . . . . . . . . . . . .

41

Chng 6. thi olympic Estonia . . . . . . . . . . . . . . . . . . . . . . . .

46

Chng 5. thi olympic Tip khc . . . . . . . . . . . . . . . . . . . . . .

51

Chng 8. thi olympic India . . . . . . . . . . . . . . . . . . . . . . . . . . .

56

Ti liu tham kho. . . . . . . . . . . . . . . . . . . . . . . . . . . . . . . . . . . .

59

Chng 7. thi olympic Hungary . . . . . . . . . . . . . . . . . . . . . . .

Chng 1
thi olympic Belarus

.1.1. Hai ng cho AC v BD ca t gic ABCD ct nhau M. ng phn


gic ca gc ACD ct tia BA K . Nu MA.MC +MA.CD = MB.MD
\ = CDB.
\
th BKC
Li gii: Gi N l giao im ca CK v BD. p dng nh l v ng

.c

phn gic cho tam gic MCD

MC
CD
=
ND
MN

Hay

CD =

MC.DN
MN

khi c MB.MD = MA.MC + MA

MC.DN
MD
= (MA.MC)
MN
MN
Hay MA.MC = MB.MN
V M nm trong t gic ABCN, theo nh l v phng tch ca mt
im th A, B, C v N cng nm trn mt ng trn.
T :
\ = ABN
\ = ACN
\ = NCD
\ = KCD
\
KBD
Suy ra K, B, C v D cng nm trn mt ng trn. Do c
\ = CDB.
\
BKC

Nguyn Hu in, HKHTN H Ni

ng xu v n ng xu xp dc theo
.1.2. Trong mt tam gic u xp n.(n+1)
2
mi cnh v lun c mt ng xu ngn( trn cng) Mt php th v
xc nh bi cp ng xu v tm A, B v lt mi ng xu nm trn on
thng AB. Hy xc nh nhng yu t ban u- gi tr ca n v v tr
ban u ca ng xu c mt tri m t c th khin cho tt c ng
xu hin ra mt tri sau mt s php th v.
Li gii: V mi php th v ca 0 hoc 2 ng xu trong 1 gc, tnh
chn l ca s ngn trong gc l c bo ton.
Nu ng xu cho thy mt tri khng trong mt gc, lun c 3 ng
xu trong gc l ngn, th lun c s ngn trong gc l l. Nh vy, s
lun c 3 gc khng ng thi cho mt tri ca ng xu.
Ngc li, nu trong mt gc c ng xu mt tri, chng ta s chng

minh rng th lm cho tt c cc ng xu hin mt tri


Ta hng tam gic sao cho gc i n vi mt cnh nm ngang;

.c

Trong mi (n - 1) ng ngang c hai hoc nhiu ng xu. Ta chn hai


ng xu k nhau v lt tri tt c cc ng xu trong ng ny. Tt c

cc ng xu s cho thy mt tri.


Do yu t ban u cn la chn l c ng xu c mt tri nm trong
1 gc.

b = gi M l trung im ca cnh huyn


.1.3. Cho tam gic ABC v gc C
2
AB, H l chn ng cao CH v P l im trong tam gic sao cho AP =
b = .
\
AC. Hy chng minh rng PM l phn gic BP
H khi v ch khi A
3

Li gii: Li gii th nht


im P nm trn ng trn tm A bn knh AC. ng trn ct
b = khi v ch
ng CH, MH v PH ti D, N v Q. V MA = MC, A
3

khi tam gic ACM u. Ngha l khi v ch khi MN. iu khng

nh PM l phn gic gc HP B khi v ch khi MN


Tht vy, AH l ng cao thuc y ca tam gic cn ACD, H l trung
im ca CD, CD l mt dy cung ca ng trn , theo nh l v
phng tch ca mt im c
P H.HQ = CH.HD = CH 2 .

thi olympic Belarus

V v CH l ng cao thuc cnh huyn ca tam gic vung ABC nn


CH 2 = AH.HB. Vy P H.HQ = AH.HB.
Do H l giao im ca AB v P Q nn t gic AP BQ ni tip. Xt trn
ng trn
[ = QAN
\ = 2.QP
\
\
QAB
N = 2.HP
N
Nh vy
\
[ = 2.HP
\
HP
B=\
QP B = QAB
N
V v N l giao im ca HB v PN phn gic ca gc HP B. Do
PM l phn gic ca gc HP B khi v ch khi M N

Li gii th hai
Khng mt tnh tng qut ta gi s AC = 1. Dng h trc ta vung
gc vi C lm gc, A c ta (0; 1) cn B c ta (n; 0) vi n > 0

Nu n = 1 th MN v PM khng th l phn gic ca gc BPH. Trong


b = 6= iu ny tri vi kt qu mong i
trng hp ny c A
4
3

.c

Chnh iu cho php ta chn n 6= 1 S dng cng thc khong cch


p
c AP = AC khi v ch khi P c ta dng ( m.(2 m); m) v

m nm gia 0 v 2. Ta ca M l ( n2 ; 12 ) v v CH c i n v H
2
trn AB, nn H cn tm c ta ( n2n+1 ; n2n+1 ) . S dng cng thc tnh

khong cch ta tnh c

BP
= n2 + 1
HP

S dng h thc trong tam gic vung AHC v ACB c AH =


b = CA, c = AB; t
MB
=
MH

c
2

c
2

b2
2

b2
c

vi

c2
n2 + 1
=
c2 + 2.b2
n2 1

\
Theo nh l ng phn gic PM l phn gic BP
H khi v ch khi
BP
MB
= M H . Gii phng trnh tng ng ta tnh c nghim khi v ch
HP
khi n2 (n2 3) = 0 v n > 0 nn PM l phn gic gc BPH khi v ch

b = .
khi n = 3, ngha l khi v ch khi A
3

.1.4. C tn ti mt hm f : N N sao cho

f (f (n 1)) = f (n + 1) f (n)

Nguyn Hu in, HKHTN H Ni


vi mi n > 2 ?
Li gii: Khi khng nh tn ti mt hm nh vy s dn n mu
thun. T phng trnh f(n -1) - f(n) > 0 vi n > 2 iu ny khng nh
hm f tng nghim ngt vi n > 2 nh vy, f (n) > f (2) + (n 2) >
n 1 vi n > 2 Chng ta c th lm nn f (n) nh sau: T phng

trnh cho mc nhin c f (f (n 1)) < f (n + 1) vi n > 2 hay l


f (f (n)) < f (n + 2) vi n > 1. V f l hm tng vi nhng bin ln hn
1, cho f (n) = 1 hoc f (n) < n + 2. T n 1 6 f (n) 6 n + 1 vi mi
n > 2. Ly n nguyn bt k b hn 4
Mt mt f (n) > 2 v (n 1) > 2
f (f (n 1)) = f (n 1) f (n) 6 (n + 2) (n 1) = 3

.c

Nh vy,(n 3) 6 3 v bt k n > 4 l iu v l. iu ny cho thy


khng nh ban u l khng ng v cho kt lun khng tn ti mt
hm nh th.

.1.5. Trong mt a din li vi m mt tam gic( cn cc mt khc vi hnh


dng khc), Ta lun c 4 cnh bn gp ti mi nh. Tm gi tr nh
nht c th ca m.

Li gii: Ly 1 a din vi m mt tam gic v 4 cnh bn gp nhau ti

mi nh. t F, E v V l s mt, cnh bn v nh ca a din. vi


mi cnh bn, m hai nh v cc u mt. V mi nh l u mt

ca 4 cnh bn, chng ta m nh 2 ln theo cch ny. Nh vy 2E =


4V
Ngoi ra, m s cnh bn trn mi mt v tng ca F cao nht t
c l mt s t nht l 3m + 4(F - m). Mi cnh bn c m 2 ln
theo cch ny, suy ra 2E > 3m + 4(F m)
Qua biu thc Euler cho biu phng, F + V E = 2.
Kt hp vi 2E = 4V ng thc ny l 2E = 4F 8
Nh vy

4F 8 = 2E > 3m + 4(F m)

Hay m > 8 s cn bng t c nu v ch nu mi mt ca a din l


tam gic hoc t gic, mt hnh tm mt u c nhng hnh nh vy.

thi olympic Belarus

Suy ra m = 8 l gi tr t c.

1
.1.6. a) Chng minh rng n 3 > n
vi tt c s nguyn dng n, trong
3
{x} c hiu l phn s ca x.

b) C t ti bt bin c > 1 m n 3 > nc 3 cho mi n nguyn
dng?.

Li gii: iu kin n 3 > nc 3 c th p dng vi n = 1 nu ch

nu 1> c3 v d 3 > c. t 1 6 c < 3 l mt bt bin vi mi


 
n, n 3 = n 3 n 3 ln hn nc 3 nu ch nu n 3 nc 3 . V

c < 3 < 3n2 , hai v ca bt dng thc ny l dng, chng ta ch c


th bnh phng mi v m khng lm i du bt ng thc.
h i2
c2
>
n 3
3n2

(*)

3n2 2c +

.c

Vi mi n, 3n2 1 khng phi l s chnh phng v khng c s chnh


phng no ng dh 2 mod3,
v 3n2 cng khng phi l s chnh phng.
i
 
3n2 s nguyn ln nht m bnh phng ca n
Nh vy, n 3 =
nh hn hoc bng 3n2 ti a 3n vi cn bng nu v ch nu 3n2 2

l s chnh phng. Chng ta yu cu rng s cn bng p dng ty


vi n.

Xc nh (m0 , n0 ) = (1, 1) v (mk+1 , nk+1) = (2mk + 3nk , mk , 2nk ) vi


k> 1.

D rng chng minh rng m2k+1 3n2k+1 = mk 3n2k . Nh vy, do ng


thc 3n2k 2 = m2k p dng vi k = 0, p dng vi tt c k > 1. Do

n1 , n2 , . . . l mt chui tng n dn n 3n2 2 l mt s chnh phng


vi n ty .
2
c2
2
2
3 cho
Nu c = 1 nh vy 3n2 2c + 3n
2 > 3n 2c = 3n 2c > n

tt c n. Nh vy (*) l bt ng thc p dng cho tt c n


c2
2
Tuy nhin, nu c> 1 th 3n2 2c + 3n
2 6 3n 2 cho tt c cc s ln n
tha mn. Nh vy, tn ti mt s n vi iu kin thm l 3n2 2 phi
l s chnh phng. Vi n ny (*) v ng thc b) l sai
Vy cu tr li i vi phn b) l "khng".

.1.7. Cho tp hp M = {1, 2, . . . , 40}. Tm gi tr n nh nht(n: s nguyn)


m c th chia tp M thnh n tp con ri nhau m bt k a, b v

10

Nguyn Hu in, HKHTN H Ni


0(khng nht thit khc bit) nm trong cng tp con, tha mn a 6= b+c.
Li gii: Gi s cho mc ch mu thun, c th chia tp M thnh 3
tp X, Y v Z. Khng mt tnh tng qut ta gi s rng |X| > |Y | > |Z|

cho cc x1 , x2 , . . . , x|X| l cc thnh phn ca X c sp xp theo th


t tng dn. Nhng s ny, bn cnh cc chnh lch xi x1 vi i = 2, 3,
|X|, phi l nhng thnh phn khc bit ca M. C 2|X|-1 s nh vy,

suy ra 2|X|-16 40 hay |X| 6 20. Ta c 3 |X| > |X| + |Y | + |Z| = 40,
suy ra |X| > 14. Ta c |X| . |Y | > 12 |X| (40 |X|) i trong X.Y. Tng
ca cc s trong mi cp i nh nht l 2 v ln nht l 80 c c
79 gi tr c th xy ra v 21 > |X| > 14 v hm t 21 + (40 t)
1
2

|X| .(40 |X|) >

l hm lm trn on 21 > t > 14 chng ta c




min 21 .14(26), 12 .21(19) =182 > 2.79.

.c

Theo nguyn tc Pigeonhole tn ti 3 cp i (x1 , y1), (x2 , y2),(x3 , y3 )


X.Y vi (x1 + y1 ) = (x2 + y2 ) = (x3 + y3 )
Nu bt c xi no bng nhau th tng ng yi s bng nhau, iu ny

l khng th xy ra v cp (xi , y i) l khc bit. Nh vy, chng ta


c th gi s, khng lm mt tnh tng qut rng x1 < x2 < x3 vi

1 6 j < k 6 3 gi tr xk xj nm trong M v khng th nm trong X v


mt khc xj + (xk xj ) = xk . Tng t yj yk
/ Y vi 1 6 j < k 6 3

Nh vy, 3 s chnh lch bng nhau x2 x1 = y2 y1 , x3 x2 = y3 y2 ,


x3 x1 = y3 y1 nm trong M \ X Y = Z. t a = (x2 x1 ),

b = (x3 x2 ), a = (x3 x1 ) ta c a = b+ c v a, b, c Z, suy ra mu


thun
Nh vy gi s ban u ca chng ta sai v khng th phn chia M
thnh 3 tp tha mn yu cu t ra.
By gi c th chng minh chia M thnh 4 tp vi yu cu t ra. Nu
ai {0, 1, 2} vi tt c i N v nu ai = 0 vi n > N, sau t
P
(. . . a2 a1 a0 ) v (aN aN 1 . . . a0 ) c hiu l s nguyn ni=0 ai 3i ng

nhin gi tr nguyn m c th vit di dng (. . . a2 a1 a0 ) theo mt cch


chnh xc vi c s 3. Ta t s nguyn m = (. . . a2 a1 a0 ) vo tng
A0 , A1 . . . nu a0 = 1 thay m vo A0 . Mt khc v a 6= 0, ai1 6= 0 vi
mt s i1 , bi v ch hu hn ai 6= 0, ai2 = 0, vi mt vi i2 > i1 , tip

11

thi olympic Belarus

n al 6= 0, al+1 = 0, vi mt vi l. Chn l nh nht vi thuc tnh ny


v thay m tai Al+1 = 0.
Nu m1 , m2 A1 v c s 3 biu din m1 + m2 c nhng n v s 2
nh vy m1 + m2
/ A1 . Nu m1 + m2 Al vi mt s l>1, nh vy:

0} suy ra 0 |22 {z
2} < m1 + m2 < 2 |00 {z
0}
0 |11 {z
1} < m1 , m2 < 1 |00 {z
l

nu m1 + m2 = (. . . a3 a2 a1 a0 ), th al = 1 suy ra m1 + m2
/ Al

By gi, t k > 1 l mt s nguyn dng v t S =




1, 2, . . . , 12 (3k 1) biu thc c s 3 ca 12 (3k 1) bao gm tt c
1s mi 12 (3k 1) A1 c s 3 trong tt c cc s khc trong S c
1 s 0 trong 3k1 v tr m mi s nguyn trong S nm chnh xc 1

trong cc tp hp A0 , A1 , . . . , Ak1 . Nh vy, S c th l mt phn trong


k tp hp. A0 S, A1 S, . . . , Ak1 S m a 6= b 6= c vi bt c a, b
v c trong cng mt tp. Suy ra kt qu k = 4 cho thy n = 4 l c th

t c nh yu cu.

.c

Ghi ch Vi n, k N v s phn chia ca {1, 2, . . . , k} trong n cp 3


(a,b,c) sao cho a + b = c v a,b,c trong cng tp gi l Schur triple vi
mi n N tn ti k nguyn ti a m khng c Schur triplec cho

s phn chia no {1, 2, . . . , k} vo n tp hp. S nguyn ny c


biu th bi S(n) v c gi l nth Schur number. Mc d nh hn v

n
v

nhng S(n)=44.

ln hn gii hn tn ti vi tt c S(n) khng c dng tng qut no


bit n, gii hn nh hn c tm thy trong gii php cho n =1,2,3

.1.8. Mt s nguyn dng gi l monotonic(u) nu nhng ch s ca n


trong c s 10, c t tri sang phi theo th t khng gim dn. Chng
minh rng vi mi n N tn ti n ch s monotonic l s bnh phng
Li gii: Bt k s c 1 ch s l bnh phng( v d 1,4 hoc 9) l
monotonic chng minh yu cu bi vi n = 1 Chng ta gi s n > 1
Nu n l s l, vit n = 2k - 1 cho mt s nguyn k 2
t xk = (10k + 2)/6 = |166 {z
67}
k2

12

Nguyn Hu in, HKHTN H Ni


Nh vy
x2k = (10k 2k + 4.10k + 4)/36 =
Quan st thy:
2 |77 {z
7} + 79

102k
36

72
= 102k2 .( 36
+

28
)
36

102k 10k 1
+
+
36
9
9

(*)

= 2.102k2 + 79 .102k2 =

2k2

Vy v phi ca ng thc (*) bng: 2 |77 {z


7} + 79 + |11 {z
1} + 19 =
2k2

2 |77 {z
7} 9 |88 {z
8} l mt s c n ch s monotonic l s bnh phng.
k2

k1

Nu n l s chn, vit n = 2k vi s nguyn k 1 v


yk = (102 k + 2)/3 = 4 |33 {z
3}

k1

Nh vy

102k
10k 4
+4
+
9
9
9
1
4
5}
4} + = |11 {z
1} 6 |55 {z
= 1 |11 {z
1} + + 4 |44 {z
9
9

.c

yk2 = (102k + 4.10k + 4)/9 =

k1

2k

mt s gm n ch s monotonic chnh phng (pcm)

.1.9. Cho cp (
r ,
s ) vect trong mt my bay mt dch chuyn gm chn

mt s nguyn khc khng k v sau thay i (


r ,
s ) thnh hoc (i)

( r + 2k s , s ) hoc (ii) ( r , s + 2k r ) Tr chi gm ly mt hu hn


cc chui dch chuyn, lun phin nhau dch v dng (i) v dng (ii)
cho mt vi cp vect ban u.
a) C th t c cp ((1, 0), (2, 1)) trong tr chi vi cp ban u
((1, 0), (0, 1)) nu s dch chuyn u tin l dng (i)? b) Tm tt c
cc cp ((a, b), (c, d)) c th t c trong tr chi vi cp ban u
((1, 0), (0, 1)) trong dch chuyn u tin l mt trong hai dng trn
?

Li gii: t ||
z || biu th cho chiu di ca vect
z v t |z| biu
th cho gi tr tuyt i cu s thc z

13

thi olympic Belarus

a) t (
r ,
s ) l cp vect m
r v
s c th thay i qua tr choi

quan st thy rng nu x , y l vect nh l ||


x || > ||
y ||
Nh vy:

||
x + 2k
y || > ||2k
y || ||
x || > 2||
y || ||
y || = ||
y ||

Sau ln dch chuyn u tin dng (i) ta c


r = (1, 2) v
s = (0, 1)

cho s k 6= 0 m || r || > || s || p dng kt qu trn vi x >


s v

y >
r chng ta c th thy trong dch chuyn tip theo (dng(ii))

di ca
r khng thay i trong
s tng cao hn ||
r ||, p dng kt

qu trn ln na vi x > r v y > s chng ta c th thy trong dch

chuyn tip theo (dng(i)) di ca


s khng thay i trong
r

tng vt qua || s ||, tip tc nh vy ta thy rng || r || v || s || khng

bao gi gim. Bi v sau ln dch chuyn u tin vect u tin c


di hn 1, s khng bao gi t c ((1, 0), (2, 1))

.c

b) Chng ta thay i tr chi bng cch khng yu cu dch chuyn


lun phin gia dng (i) v (ii) v bng cch cho php s la chn k =
0. ng nhin bt c cp no c th t c theo quy nh ban u

phi t c theo nhng quy nh mi ny.


iu ngc li ng v bng cch loi bt c dch chuyn no theo nhng

quy nh mi vi k = 0 v kt hp bt c dch chuyn mi no ca cng


dng vo 1 dch chuyn ta t c chui dch chuyn theo quy lut ban

u v c cng 1 cp. ((, x), (y, z)) i din cp ca nhng vect


vi , x, y v z thay i qua tr chi.

D dng kim tra gi tr ca z xy v tnh chn l ca x v y l


khng thay i theo bt c dc chuyn no trong tr chi. Trong mt
tr chi m bt u vi ((, x), (y, z)) = ((1, 0), (0, 1)), ta phi lun lun
c z xy = 1 v b c 0(mod2). Bi v x v y lun lun chn v
z khng i mod4, ta phi c z 1(mod4) thng qua tr chi.

Gi 1 cp ((a, b), (c, d)) tha mn khi ad - bc = 1, a d 1(mod4) v


b c 0(mod2) trn ta thy rng bt c cp i t c trong tr

chi vi cp ban u ((1, 0), (0, 1)) phi tha mn v by gi ta chng


minh iu ngc li.
Gi s, nhm mc ch thy c s mu thun rng c nhng cp i
((a, b), (c, d)) tha mn iu kin a ra. t ((e, f ), (g, h)) l cp m

14

Nguyn Hu in, HKHTN H Ni


ti thiu ha |ac|
Nu g = 0 th eh = 1 + fg = 1 bi v e h 1(mod4) e = h = 1,

nu f = 0 cp ny chc chn c c. Mt khc, bng cch dch chuyn


dang (i) vi k = f2 chng c th thay i dng ((1, 0), (0, 1)) thnh dng

((e, f ), (g, h)) dn ti mu thun


Nh vyg 6= 0 by gi g l s chn e l s l, |e| > |g| hoc |e| < |g|
ta c e 2k0 g nm trong on (-|e|, |e|) cho k0 {1, 1}. Thc hin
dng (i) dch chuyn n ((e, f ), (g, h)) vi k = k0 th t c mt
cp mong mun khc ((e0 , f 0), (g, h)). Bi v |e| < |e| v g 6= 0, chng ta

c |eg| < |eg|. Nh vy, bng khi nim ti thiu ((e, f ), (g, h)) cp i
mi c th t c t ((1, 0), (1, 0)) vi mt dy dch chuyn S no .
Nh vy, chng ta t c ((e, f ), (g, h)) t ((1, 0), (0, 1)) bng cch p
dng trc tin dch chuyn trong S ti ((1, 0), (0, 1)) sau p dng

thm dch chuyn dng (i) vi k = k0 . Nh vy cp i cc tiu t


c dn n mu thun.

.c

Mt chng minh tng t nu |e| < |g| , khi chng ta thay la chn r0
vi g 2k0 e (|g|, |g|) v thc hin dng dch chuyn (ii). Nh vy
trong tt c cc trng hp chng ta c s mu thun. Hay chng ta c

th kt lun rng bt c cp i t c u thc s tha mn. iu


ny hon ton c chng minh.
(a + b + c)3
a3 b3 c3
+ +

x
y
z
3(x + y + z)

.1.10.Chng minh:

Vi tt c cc dng s thc a, b, c
Li gii: Qua chng minh khng cn bng ca Holder
3

( ax +

b3
y

c3 31
) (1
z

+ 1 + 1) 3 (x + y + z) 3 (a + b + c)

ly tha 3 c 2 v v chia c 2 cho 3(x + y + z) ta c pcm


.1.11.Gi P l giao im ca hai ng cho AC v BD ca t gic li ABCD
trong AB = AC = BD. Gi O v I l circumcenter v tm ni tip
ca 3 phn gic ca tam gic ABP. Chng minh rng nu O 6= I th
ng thng OI v CD vung gc.

15

thi olympic Belarus

Li gii: u tin ta chng minh mt lun rt hu ch XY v UV ,


t X v Y l chn gc vung ca X v Y, ni ng thng UV. S
dng khong cch trc tip, XY UV nu ch nu
UX - XV = UY - YV
v UX + XV =UV = UY + YV, php tnh trn t c nu ch nu
UX 02 X 0 V 2 = UY 02 Y 0 V 2 , hoc UX 2 XV 2 = UY 2 Y V 2 .

Nh vy n tha mn ng thc DO 2 CO 2 = DI 2 CI 2 . t AB=


AC=BD = p, PC = a v PD = b nh vy AP = p - a v BP = p b. t R l bn knh ng trn ngoi tip tam gic ABP. ta c pb =

DP.DB = DO 2 R2 ngoi ra pa = CO 2 R2 .
Nh vy DO 2 CO 2 = p(b a), v tam gic ADB l cn vi BA =
\ ID = IA ngoi ra
BD v I nm trn ng phn gic ca gc ABD,
IB = IC t T l im tip xc ca vng trn ni tip tam gic ABC

.c

vi cnh AB. Nh vy BT = (p + a - b)/2 v IT vung gc vi AB,


AI 2 BI 2 = AT 2 BT 2 . t cc tham s li vi nhau chng ta thy

rng.

DI 2 CI 2 = AI 2 BI 2 = AT 2 BT 2
= (AT + BT )(AT BT )

= p(b a)
= P O 2 CO 2 (pcm)

Chng 2
thi olympic Bungari
.2.12.Mt ng thng l i qua trc tm ca tam gic nhn ABC. CMR cc

ng thng i xng vi l qua cc cnh ca tam gic ng quy.

.c

Li gii: Gi H l trc tm ca tam gic ABC. V tam gic ABC


nhn nn trc tm H nm trong tam gic ABC. Khng mt tnh tng

qut chng ta gi s l ct AC v BC ti P v Q. Nu l k AB, ly R l


im ty trn ng thng i xng vi l qua ng thng AB. Nu
l khng song song vi AB th ly R l giao im ca ng thng l i

xng vi ng thng AB v ta c th gi s R nm trn tia BA.


LyA1 , B1 , C1 , tng ng l cc im i xng vi H qua cc ng

thng BC, CA, AB. Khi , A1 ,B1 ,C1 nm trn ng trn ngoi tip
\
\ \
\
w ca tam gic ABC (Ch :A
1 CB = BCH = HAB = A1 AB =)

Ta cn chng minh: A1 P ,B1 Q,C1 R ng quy


V hai ng thng AC v BC khng song song, nn hai ng thng
B1 Q v A1 P khng song song. Ly S l giao im ca A1 P v B1 Q V
\
\ \ \ \ \
SA
1 C+SB1 C=P A1 C+QB1 C=P HC+QHC= nn t gic SA1 CB1 l
im hi t ng trn
Do , S l giao im ca ng thng B1 Q v ng trn w. Tng
t, hai ng thng B1 Q v C1 R khng song song v giao im ca
chng cng chnh l giao im ca B1 Q v ng trn w. Do vy, cc
ng thng A1 P , B1 Q, C1 R ng quy ti mi im nm trn ng
trn ngoi tip tam gic ABC

17

thi olympic Bungari

.2.13.C 2000 qu cu trng trong mt chic hp. Bn ngoi chic hp cng


c cc qu cu trng, xanh v vi s lng khng hn ch. Trong mi
ln thay i , chng ta c th thay i 2 qu cu trong hp bi 1 hoc 2
qu cu theo cch sau:
2 qu trng bi 1 qu xanh, 2 qu bi 1 qu xanh, 2 qu xanh bi 1
qu trng v 1 qu , 1 qu trng v 1 qu xanh, bi 1 qu hoc 1
qu xanh v 1 qu bi 1 qu trng. (a) Sau mt s hu hn ln thc
hin nh trn cn li 3 qu cu trong hp. CMR c t nht 1 qu xanh
trong 3 qu cu cn li. (b) Liu c th xy ra sau mt s hu hn ln
thc hin nh trn trong hp cn li ng mt qu cu.
Li gii: Ta gn gc gi tr i cho mi qu cu trng, i cho mi qu

cu , v -1 cho mi qu cu xanh. Ta c th kim tra li rng cc


php thay th cho khng lm thay th cc gi tr ca cc qu cu

trong hp. Tch cc gi tr ca cc qu cu ban u l i2000 = 1.


Nu trong hp cn li ba qu cu khng c qu no mu xanh th tch

.c

cc gi tr ca chng s l : i, mu thun. Do , nu trong hp cn


li ba qu, th phi c t nht 1 qu mu xanh, (a) c chng minh.

Hn na, v khng c qu no c gi tr 1 nn trong hp phi cha t


nht hai qu cu. Do , khng th xy ra trng hp trong hp cn
li 1 qu( chng minh (a), chng ta c th gn gi tr 1 cho mi qu

xanh, -1 cho mi qu cu hoc trng.

.2.14.ng trn ni tip tam gic cn ABC tip xuc vi cc cnh AC v

BC tng ng ti M v N. ng thng t tip xc vi cung nh MN,


t giao vi NC v MC tng ng ti P v Q. Gi T l giao im ca
hai ng thng AP v BQ.
(a) Chng minh T thuc MN .
(b) CM: Tng din tch cc tam gic AT Q v BT P t gi tr nh nht
khi t k AB
Li gii: (a) Hnh lc gic suy bin AMQP NP c ngoi tip bi cc
ng trn ni tip tam gic ABC. Theo nh l Brianchon, cc ng
cho AD, MN , QB l ng quy. Do , T thuc MN.
Chng ta c th s dng cch gii s cp hn. Gi R v S tng ng l

18

Nguyn Hu in, HKHTN H Ni


cc tip im ca ng trn ni tip vi cc cnh AB v P Q : Gi T1 ,
T2 tng ng ca tam giac ABC l cc giao im ca BQ vi MN v
SR.
\ = P\
V QMN
NM =

d
M
N
2

\ = sinP\
\.
nn ta c: sin QMN
NM = sinBNM

p dng nh l hm s sin trong tam gic cho cc tam gic MQT1 v


NBT1 .
\
\
QT1
N
M
1
= sin QM
.
= sin BN
= BT
QM
BN
\
\
sin QT
M
sin
BT
1
1N
QT1
MQ
QT2
SQ
hay BT1 = BN . Tng t: BT2 = BR
.

QT1
=
Theo tnh cht ca tip tuyn, BN= BR v QM= QS. Do : BT
1
QT2
. V T1 v T2 u thuc BQ nn ta phi c T1 T2 . Do , BQ,MN ,
BT2

SR ng quy
Mt cch tng t, ta chng minh c AP , MN , SR ng quy. T
[ = CBA
[ v B= ACB.
[
T MN . Gi = CAB

.c

Gi f = [AQT ] + [BP T ] = [ABQ] + [ABP ] 2.[ABT ]


d k AB,
d suy ra [ABT ] l hng s. Do , f
V tam gic ABC cn, MN
t gi tr nh nht [ABC] + [ABP ] t gi tr nh nht.
rng : 2f 0 = AB.(AQ + P B). sin

= AB.(AB + P Q). sin


Trong : AQ + P B = AP + QP v t gic ABCD c ng trn ni

tip.
f t gi tr nh nht khi P Q t gi tr nh nht. Gi I l tm ng

trn ni tip cu tam gic ABC, do I l tm ng trn bng tip ca


[
tam gic CP Q. Do , P C+CQ+QP = 2.CM khng i. t CP
Q=p
[ =q. Th p + q= khng i. p dng nh l hm s sin
v CQP
cho tam gic CP Q ta c :
2. sin

CM
PQ

= 1+

CP
PQ

CQ
PQ

= 1+

sin p+sin q
sin

p+q
. cos pq
2
2

1+
.
sin
ln nht. T , [AT Q] + [BT P ]
P Q t gi tr nh nht khi cos pq
2
nh nht khi p = q, tc l khi P Q k AB.

.2.15.Cho n im trn mt phng (n >= 4) sao cho khong cch gia 2 im


bt k trong n im l mt s nguyn. CMR t nht 16 trong s cc
khong cch chia ht cho 3.
Li gii: Trong bi gii ny, cc ng d xt theo modul 3. Trc ht

19

thi olympic Bungari

ta chng minh nu n = 4, th t nht c hai im ri nhau m khong


cch gia chng chia ht cho 3. K hiu 4 im lA, B, C, D. Gi s
cc khong cch AB, BC, CD, DA, AC, BD khng chia ht cho 3.
\ = BAC
[ =\
Khng mt tnh tng qut, ta gi s BAD
CAD.
[ v y = \
Gi x=BAC
CAD.
Gi

2.AB.AC. cos x,

2.AD.AC. cos y v

2.AB.AD. cos (x + y). p dng nh l hm s cosin cho cc tam


giac ABC, ACD, ABD ta c
BC 2 = AB 2 + AC 2
CD2 = AC 2 + AD 2
BD2 = AB 2 + AD 2

V bnh phng mu khong cch l mt s nguyn nn , v cng l


cc s nguyn. Do :

.c

2.AC 2 . = 4.AC.AB.AD. cos (x + y)

= 4.AC 2 .AB.AD (cos x. cos y sin x. sin y)

= . 4.AB.AD. sin x. sin y.


l s nguyn. V vy: 4.AC 2 .AB.AD. sin x. sin y l mt s nguyn chn

v
p
sin x. sin y = (1 cos2 .x).(1 cos2 .y) l mt s hu t, khi vit di
dng ti gin t s l s khng chia ht cho 3. t p = 2.AB.AC v
q = 2.AD.AC, do

v cos y = q
2 2 2 2
(p ).(q p )
V sin x.siny =
pq
L s hu t nn t s v phi cng l mt s nguyn. T s chia ht
cos x =

cho 3 v p2 1( modul 3) v 2 1 (modul 3


Nhng mu s khng chia ht cho 3. Do , khi sin x. sin y vit di
dng ti gin th t ca n chia ht cho 3, iu ny mu thun. Do ,
iu gi s ban u l sai. Vy c t nht mt khong cch chia ht cho3

20

Nguyn Hu in, HKHTN H Ni


vi n = 4
Xt trng hp n 4. T mt tp n im, c Cn4 cc tp con cha 4
im c t nht hai im trong mi tp ri nhau c khong cch chia
2
ht cho 3, v mi khong cch c m trong t nht Cn1
tp con.
vy c t nht

4
Cn
2
Cn2

2
Cn
6

cc khong cch chia ht cho 3

.2.16.Trong tam gic ABC, CH l ng cao v CM v CN tng ng l cc


ng phn gic ca cc gc ACH v BCH. Tm ng trn ngoi tip
ca tam gic CMN trng vi ng trn ni tip ca tam gic ABC.
CMR: [ABC] =

AN.BM
.
2

Li gii: Gi I l tm ng trn ni tip ca tam gic ABC, gi tip


im ca ng trn ni tip tam gic ABC vi cc cnh l AC, AB ln
\.
IN = 21 .MIN
lt l E v F . V IM = IN v IF IM, nn ta c F[

.c

Hn na , v I l tm ng trn ngoi tip ca tam gic CMN nn:


1 \
\ = 1 .ABC
[ = ECI.
[
.MIN = MCN
2
2
[
Do F[
IN = ECI.
[ Nn NF I IEC.
[I = = IEC.
ta cng c NF
2

V NI = NC, nn hai tam gic ny l ...(congruent), v NF = IE = IF .


[ = 2.F[
IN =
Tam gic NF I l tam gic vung cn, F[
IN = 4 v ACB

.
2

\ = CBH
\ = BAC
[ v
Do , HCB
2
\
\ = ACB
[ 1 .HCB
\ = BAC
ACN
2

\ = (ACN
\ + NAC)
\
T suy ra, CNA
\
\ v AN = AC.
= BAC = ACN

Tng t, BM = BC. Do vy: 12 AN.BM = 12 .AC.BC = [ABC].


.2.17.Cho dy s (an ):
a1 = 43, a2 = 142 v an+1 = 3.an + an1 vi mi n > 2.
CMR
(a) an v an1 l nguyn t cng nhau vi mi n >= 1.
(b) Vi mi s t nhin m, tn ti v hn s t nhin n sao cho an 1
v an+1 1 u chia ht cho m.

Li gii: (a) Gi s c n, g > 1 sao cho


Khi g chia ht an1 = an+1 3.an

g
an

g
.
an+1

21

thi olympic Bungari

Nu n 1 > 1 th g chia ht an+1 , an , ...a2 , a1 , nhng iu ny khng


th xy ra v WCLN (a2 , a1 ) = 1. Do , an v an+1 l nguyn t cng
nhau vi mi n > 1.
(b) Xt dy (a0n ) c xc nh nh sau: a01 , a02 a0n+1 = 3.a0n + an1 vi
mi n > 2.
D thy: a03 = 4, a04 = 13, a05 = 43, a06 = 142
Tc l a1 = a05 , a2 = a06 . M hai dy (an ) v dy (a0n ) c cng cng thc
truy hi nn ta c:
an = a0n+4 , vi mi n > 1.
Gi bn l s d khi chia a0n cho m, v xt cp s (bn , bn+1 ) vi n > 1. V
c mt s v hn cc cp s nh vy nhng ch l m2 cp cc s nguyn
(r, s) vi 0 6 r, s < m do phi c hai trong s cc cp trng nhau,
chng hn (bi , bi+1 ) = (bi+t , bj+t ) vi t > 0.

S dng cng thc truy hi, ta d dng chng minh c bng qui
np theo n: bi+n = bi+n+t vi mi n tha mn (i + n) > 1. Do ,

.c

(b1+kt , b2+kt ) = (b1 , b2 ) = (1, 1) vi mi k > 1. Do , akt3 1 v


akt2 1 u chia ht cho m vi mi k > 4.

\ = CDA,
\ ng phn gic ca gc ABC
.2.18.Cho t gic li ABCD c BCD
ct CD ti im E.
[ = khi v ch khi AB = AD + BC.
CMR: AEB
2

[ = th \
Li gii: Nu AEB
CEB < 2 . T suy ra c im F nm
2
\ = BEC.
\ Khi , c hai tam gic BEC
trn cnh AB sao cho BEF
\ Do
v BEF bng nhau, suy ra BC = BF v \
BF E = \
BCE = EDA.
[ = v
, t gic ADEF l t gic ni tip ng trn. V AEB
2
\
\
[
\
CEB = BEF nn ta c F EA = AED.
\
\ = AF
\
T suy ra F
DA = F[
EA = AED
D. Do :
AF = AD v AB = AF + BF = AD + BC
Nu AB = BC + AD th c im F thuc AB sao cho AF = AD v
BF = BC. Khi hai tam gic BCE v BF E l bng nhau v t gic
ADEF l t gic ni tip c ng trn.
\
\
\ do
Cng c F
DA = AF
D. Do , F[
EA = \
F DA = \
AF D = AED,
\
ng thng AE l phn gic ca gc F
ED.
[ do vy AE
V BCE = BF E nn EB l phn gic ca gc CEF

22

Nguyn Hu in, HKHTN H Ni


[ = /2.
BE v AEB

.2.19.Trong

ta

Oxy,

mt

tp

gm

2000

im

(x1 , y1 ), (x2 , y2), ...(x2000 , y2000 ) c gi l tt nu 0 6 xi 6 83,


0 6 y 6 1 vi i = 1, 2, .., 2000 v xi 6= xj khi i 6= j .Tm s nguyn
dng n ln nht sao cho vi mi tp tt phn trong v bin ca hnh
vung n v no cha ng n im trong tp l phn trong v phn
bin ca tp tt ;
Li gii: Trc ht ta chng minh rng vi mi tp tt, mt hnh vung
n v no cha ng 25 im ca tp tt .
Ta gi mt hnh vung n v l proper (ring) nu 2 cnh ca n nm
trn cc ng thng y = 0 v y = 1.

Mi im cho trc u nm trong min


R = (x, y)|0 6 ex 6 83, 0 6 y 6 1

.c

Min R c th c chia thnh cc hnh vung n v proper m cc


cnh bn tri nm trn cc ng thng c phng trnh : x = i vi
i = 0, 1, .., 8.

V 83.24 < 2000, nn mt trong cc hnh vung phi cha nhiu hn


25 im. V 83.26 82 > 2000 nn mt trong cc hnh vung cha

t hn 26 im. Hn na trong 83 hnh vung n v , xt cc hnh


vung n v proper m cc cnh bn tri nm trn cc ng thng

dng x = xi hay x = xi 1.
Th t cc hnh vung n v t tri qua phi gi s l: S1 , S2 , ...Sk ,

trong cnh bn tri ca Si nm trn ng x = ti vi i = 1, 2..., k 1,


c nhiu nht mt trong cc im cho trc nm trong min c xc
nh bi zi 6 x < zi+1 , c nhiu nht 1 trong cc im cho trc nm
trong min c xc nh bi zi+1 < x 6 zi+1 = 1.
Do , vi mi i s cc im trong Si khc vi cc im trong Si+1 hoc
l 1, 0 hay 1. V c Si1 cha t nht 25 im v c Si2 cha nhiu
nht 25 im , t suy ra c Si3 (i3 nm giai1 v i2 ) cha ng 25
im. By gi ta chng minh rng
83
, xi = (i 1). 12 .d vi i = 1, 2, ..., 2000 v y2k1 = 0, y2k = 1
t d = 2. 1999
vi k = 1, 2, ..., 2000.
Vi 2 im phn bit bt k (x1 , y1 ) m cng nm trn ng nm ngang

23

thi olympic Bungari

2
(y = 0hocy = 1) th khong cch gia chng thp nht l d > 25
Gi XY ZW l 1 hnh vung n v. Vi j = 0, 1 min Ro b chn bi

hnh vung giao vi mi ng thng y = j trong mt khong ng


c di ri . Nu t nht mt trong cc s ro , r1 l 0 th khong tng
ng cha nhiu nht mt im (xi , yi ). Khong khc c chiu di nhiu

nht 2, v do c th cha nhiu nht [ d2 ] + 1 6 18 cc im nh

vy ni chung khng vt qu 19. Ta cng c, nu XY ZW c mt cp


cnh nm trn cc ng nm ngang th Ro cha nhiu nht [ 1d ]+1 6 25
2

cc im nh vy/

Mt khc, Ro giao vi ng y = 0 v y = 1 ti cc im P, Q v R, S,
trong P v R nm bn tri Q v S. Ta cng c P Q v RS cha nhiu
] + 1 cc im chn.
nht [ PdQ ] + 1 v [ RS
d

Dch chuyn Ro theo hng song song vi cc cnh ca hnh vung n


v m tm ca n nm trn ng thng y = 21 . Gi R1 l nh ca Ro

.c

v gi P 0 , Q0 , R0 v S 0 l giao ca n vi cc ng y = 0 v y = 1 c
xc nh tng t nh trn. Khi : P 0Q0 + R0 S 0 = P Q + RS. Ta cng

c P 0Q0 = R0 S 0 do tnh i xng. Gi R2 l min thu c bi php


quay R1 quanh tm ca n.
Khi min R1 R2 R1 R2 l hp ca 8 min tam gic bng nhau.

Gi T v U l cc nh bn tri v bn phi ca min R2 trn ng


y = 1 v gi V l nh ca R1 trn ng y = 1.

Gi K v L l cc nh trn cng ca cc cnh thng ng ca R2 (v


cng thuc min b chn R1 ). Ta c: KT R0
= S 0 V R0
= S 0 V L Ta

cng c:
T R0 + R0 S 0 + S 0 V = T U = 1

Mt khc, theo bt ng thc tam gic T R0 + S 0 V = R0 V + S 0 V > R0 S 0


T suy ra R0 S 0 < 12 .
V P 0Q0 = R0 S 0 , s cc im (xi , yi) nm trong XY ZW nhiu nht:
]+2 6
[ PdQ ] + [ RS
d

P 0 Q0 +R0 S 0
d

+ 2 < d1 + 2 < 15 Bi ton c chng minh.

.2.20.Cho tam gic nhn ABC


(a) CMR c duy nht ba im A1 , B1 , C1 tng ng nm trn
BC, CA, AB tha mn:nu ta chiu hai trong ba im ln cnh
tng ng (cn li), th trung im ca hnh chiu l im cn li.

24

Nguyn Hu in, HKHTN H Ni


(b) CMR tam gic A1 B1 C1 ng dng vi tam gic c cc nh l trung
im ca ABC
Li gii: (a) Trc ht ta xem xt ngc li, gi s c tam gic
A1 B1C1 c tnh cht nh vy.
Gi T l trung im ca A1 B1 theo nh ngha C1 T AB
Gi P l trng tm ca A1 B1 C1 . V P A1 BC, P B1 CA v
P C1 AB, P xc nh duy nht A1 B1 C1 .
R rng cc t gic AB1 P C, BC1 P A1 , CA1 P B1 l cc t gic ni tip
c ng trn.
[ = ABC,
[ x = A\
\
t = CAB,
1 B1 P v y = B1 A1 P
\
B
1 CD = y

\
\
V cc t gic AB1 P C1 v CA1 P B1 l ni tip c JP
B1 = , JP
A1 =
\
, A1CP = x,

.c

p dng nh l hm s sin cho cc tam gic


A1 T P v B1 T P ta c:
sin y
sin x
= TTAP1 = TTBP1 = sin
sin

sin y
sin x
= sin
hay sin

Mt cch tng t ta CM c :

\
sin ACE
\
sin BCF

= sin
sin
Trong , F l trung im ca cnh AB

V tam gic ABC nhn nn ta suy ra:


\
[
\
\
A
1 CP = x = ACF v B1 CD = y = BCF

Do cc ng CP v CF i xng qua ng phn gic ca gc


ACB. Ta c kt qu tng t cho cc ng AP v AD, BP v BE,
trong D v E l cc trung im ca cc cnh BC v CA
T suy ra P l "isognal cnugate" ca G, G l trng tm ca
ABC. Do , P l duy nht v bc ngc li ch ra rng P xc nh
duy nht A1 B1 C1 tha mn iu kin ca bi ton.
(b) Ko di AG v pha G n K sao cho GD = DK. Khi , BGCK
l hnh bnh hnh v
CK = BG = 32 .BE, CG = 23 .CF
GK = AG = 32 .AD

25

thi olympic Bungari

Do , tam gic CGK ng dng vi tam gic to bi cc ng trung


bnh ca ABC. Ta cn chng minh A1 B1 C1 v CGK l ng dng
Tht vy:
\
\
\
\
B\
1 C1 A1 = B1 C1 P + A1 C1 P = B1 AP + A1 BP
[ + GBA
[ = KGB
\ + GKC
\
= BAG
Chng minh tng t ta c:
\
C\
1 A1 B1 = KCG
.2.21.Cho p > 3 l mt s nguyn t v a1 , a2 , ..., ap2 l mt dy cc s
nguyn dng sao cho p khng chia ht cho ak hoc akk 1 vi mi
k = 1, 2, ..., p 2.
CMR tch ca mt s phn t ca dy ng d vi modulo p.

Li gii: Ta chng minh bng qui np theo k = 2, ..., p 2 c cc s


nguyn bk,1 ...bk,i sao cho:

.c

(i) mi bk,i hoc bng 1 hoc l tch ca mt s phn t ca dy


a1 , a2 , ..., ap2 v

(ii) bk,m 6= bk,n (modp) vi m 6= n


Vi k = 2, ta c th chn b1,1 = 1 v b1,2 = a1 1 (modp)

Gi s chng ta chn c bk,1 , ..bk,k .


V ak 6 1 (mod p), ta c:

(ak bk,1 )(ak bk,2 )...(ak bk,i) 6 bk,1 bk,2 , ...bk,i (modp)
Do , chng ta khng th hon v (ak bk,1 ...ak bk,k ) sao cho mi phn t
l ng d theo modulop vi phn t tng ng trong (bk,1 ..., bk,k ).

V cc s ak bk,i l khc nhau theo modulop nn phi ko sao cho cc


s bk,1 , ..., bk,k , ak .bk,1 khng c hai s no *ng d thep modul p. t
bk+1,1 , bk+1,2 , ..., bk+1,k+1 l cc s trn. Mi b k + 1 s ny u bng 1
hoc l tch ca mt s phn t ca dy a1 , a2 , ..., ap2 . Php quy np
c chng minh hon ton.
Xt cc s bp1,1 , ..., bp1,p1 . Chc chn mt trong cc s ny ng d
vi 2 theo modul p v s khc 1 v ng d vi tch ca mt s s ak
.2.22.Cho tam gic nhn ABC cn ti A. Gi D l trung im ca AB. Chn
E trn AB, v ly O l tm ng trn ngoi tip ca ACE. Chng
minh rng ng thng qua D vung gc vi Do , ng thng qua E

26

Nguyn Hu in, HKHTN H Ni


vung gc vi BC v ng thng qua B song song vi AC l ng quy.
Li gii: Gi l l ng thng i qua B v song song vi ng thng
AC, gi F1 v F2 l cc im trn ng thng l sao cho OD DF1
v BC EF2
Gi H1 vH2 ln lt l hnh chiu vung gc ca F1 v F2 l ng

thng AB. V gc CAB nhn nn im 0 nm trong ABC. T ,


suy ra F1 nm gia hai tia AB v AC.
V gc ABC nhn nn F2 cng nm gia hai tia AB v AC.
Ta cn chng minh F1 H1 = H2 F2 .
Gi G l tm ng trn ngoi tip ca tam gic ABC v gi O1 , G1

ln lt l hnh chiu vung gc ca 0 ln AB v G ln 001 .


V OD DF1 , OO1D DH1 F nn

OO1
=O
(1)
1D
[ = CBA
[ = x. V AG = GC v AO = OC, GO l phn gic
t BAC
[ = x.
ca gc AGC nn CGO
\
[
V CG k OO1 , G
1 OG = CGO = x

.c

DH1
F 1 H1

Do , cc tam gic vung GOG1 v F1 BH1 ng dng v:


BH1
1
= OG
(2)
BF1
OG
T (1) v (2) ta suy ra:

F1 H1 =

DH1.O1 D
BH1 .O1 D
=
OG1
OO1

DH1 .O1 D BH1 .O1 D


OO1 OG1

BD.O1 D
BD.O1D
=
G1 O 1
GD

.
\ = ACB
[ = 2.x, ta thu c
V DGB
F1 H1 = tan2x.O1 D

27

thi olympic Bungari


Gi I l giao im ca BC v EF2
[ = 2x v H\
V BF2 k AC, \
F2 BI = ACB
2 BF2 = x
rng BE = AB AE
= 2.(AD AO1 ) = 2O1 D
T suy ra:

F2 H2 = BF2 . sin H2 BF2 = BF2 . sin x

BE. cos x sin x


= O1 D.tan2x = F1 H1
cos 2x

.c

BI. sin x
BI
. sin x =
cos 2x
cos \
F2 BI

Ta c iu phi chng minh.

.2.23.Cho n l mt s nguyn dng. Mt dy s c gi l dy nh phn


nu cc phn t ca n l 0 hoc 1. Gi A l tp tt c cc dy nh
phn c n phn t , v gi 0 A l dy m cc phn t u l 0. Dy

c = (c1 2, ..., cn ) c gi l tng a + b ca cc dy a = (a1 , a2 , ..., an ) v


b = (b1 , b2 , ..., bn ) nu ci = 0 khi ai = bi v ci = 1 khi ai 6= bi

Gi f : A A l nh x vi f (0) = 0 sao cho nu a v b c ng n


phn t khc nhau th f (a) v f (b) cng c ng n phn t khc nhau.
CMR: nu a, b, c A sao cho a + b + c = 0 th f (a) + f (b) + f (c) = 0

Li gii: Xt dy e1 = (1, 0, 0, ..., 0), e2 = (0, 1, 0, ..., 0), ..., en =


(0, 0, ..., 0, 1). Vi mi i, 0 v ei khc nhau do phn t 1, nn f (0) v

f (ei ) cng khc nhau. Nh vy tc l f (ei = ej ) vi j no .


Xt mi dy ty x = (x1 , x2 , ..., xn ) vi f (x) = (y1 , y2 , ..., yn ). Nu x
c tham s 1 th f (x) cng c tham s 1. Nu f (e1 ) = ej v xi = 1 th
ei v x c t 1 phn t khc nhau . iu ny ch xy ra nu yj = 1, v
nu khng ej v f (x) s c t + 1 phn t khc nhau. Mt cch tng t
, nu xi = 0 th yj = 0

28

Nguyn Hu in, HKHTN H Ni


Nu a = (a1 , a2 , ..., an ), b = (b1 , b2 , ..., bn ), c = (c1 , c2 , ..., cn ) v a+ b+ c =
0 th ai + bi + ci l chn vi i = 1, 2, ...n
Vi mi ej ta c th chn ej sao cho f (ei ) = ej
Cc phn t th j ca f (a), f (b), f (c) tng ng l ai , bi , ci nn tng

.c

ca chng l mt s chn . Do , f (a) + f (b) + f (c) c phn t th j


l 0 vi j v f (a) + f (b) + f (c) = 0.

Chng 3
thi olympic Canada

.3.24.Cho a1 , a2 , ..., a2000 l mt dy s nguyn lin tip trong khong


2000
P
[1000, 1000]. Gi s
ai = 1
i=1

.c

Chng minh rng iu kin xc nh l c dy con ca a1 , a2 , ..., a2000 c


tng bng 0

Li gii: Ta thy rng c th sp xp li dy

a1 , a2 , ..., a2000

thnh dy
n
P

i=1

bi [999, 1000] vi n = 1, 2, 3, ..., 2000

sao cho

b1 , b2 , ..., b2000

Chng ta gii hn bi . Khng phi tt c cc ai = 1000 do vy chng

ta c th t b1 bng ai no thuc [999, 1000].


n nh ch s i ny. Gi s chng ta xy dng dy b1 , b2 , ..., bk (1
k < 2000) vi k c n nh.
k
P
bi [999, 0] hoc [1, 1000] th tng ca cc
Nu
i=1

ai c th khng xc nh hoc xc nh.


V vy t nht mt ai l xc nh ( hoc khng xc nh). t bk+1
k+1
P
bi [999, 1000]
[1, 1000] hoc [1000, 0], c ngha l
i=1

30

Nguyn Hu in, HKHTN H Ni


C lp li qu trnh trn ta xy dng c dy b1 , b2 , ..., b2000 . Bng cch
xy dng trn ta xy dng c dy tng
n
P
ring n =
bi (1 n 2000) bng 1 ca 2000 s nguyn thuc
i=1

[999, 1000].
Bi vy nu i 6= j vi i<j hoc tri li i = 0 vi mt vi i.

Trong trng hp u tin ta c dy con bi+1 , bi+2 , ..., bj c tng bng 0.


Trong trng hp th 2 c dy con b1 , b2 , ..., bi c tng bng 0.
Vy ta c pcm.
[ = 2ADB,
[ ABD
[ = 2CDB,
[ AB = CD.
.3.25.Cho t gic ABCD c CBD
Chng minh rng AB=CD.

[ y =CDB,
[ CBD
[ = 2x, ABD
[ = 2y
Li gii: t x = ADB,
p dng nh l Sin trong tam gic ABD v tam gic CBD ta c:
= BD
= BD
= sin((2x+y))
BA
BC
sin y
sin(2y + x) sin y = sin(2x + y) sin x

sin((2y+x))
sin x

.c

12 (cos(y + x) cos(3y + x)) = 21 (cos(x + y) cos(3x + y))


cos(3y + x) = cos(3x + y)
[ < 0 < 3y + x + (3x + y) < 2
Do 0 < x + y = 12 ABC
2

3y + x = 3x + y
[ = CBD
[ AD = CD
x = y ABD

.3.26.Cho dy s thc a1 , a2 , ..., a100 tha mn a1 a2 ... a100 0(1) v


a1 + a2 100(2), a3 + a4 + ... + a100 100(3)

Tm max ca a21 + a22 + ... + a2100 v ch ra cc ai t c.

Li gii: Vi i 3 ta c 0 ai a2 v suy ra ai (ai a2 ) 0


Du "=" xy ra nu ai {0, a2 }
Suy ra
100
100
X
X
2
ai
ai a2
i=3

Theo (3) du "=" ch xy ra nu

i=3

100
P

i=3

ai = 100 hoc a2 = 0

T (1) v (2) suy ra 0 a2 100 a1 100 a2 hoc 0 a2 50

31

thi olympic Canada

2a2 (a2 50) 0


Du "=" xy ra nu a2 = 0 hoc a2 = 50
100
100
P 2
P 2

ai = a21 + a22 +
ai (100 a2 )2 + a22 + 100a2
i=1

i=3

= 10000 + 2a2 (a2 50) 10000


Du "=" ch xy ra nu :
(a) {a3 , a4 , ..., a100 } {0, a2 }
100
P
(b) ai = 100 hoc a2 = 0
i=3

(c)a1 = 100 a2

(d)a2 {0, 50}


T iu kin trn dy a1 , a2 , ..., a100 c th l:
100, 0, 0, ..., 0

.c

hoc 50, 50, 50, 50, 0, 0,...,0


Vy tng ln nht l 10.000

Chng 4
thi olympic Trung Quc
.4.27.Cho tam gic ABC tha mn BC CA AB.Gi R v r ln lt l

bn knh cc ng trn ngoi tip v ni tip tam gic ABC.Tm theo


gc C ca tam gic BC + CA 2R 2r l dng ?,m hoc bng 0

?.

.c

Li gii: t AB = c,BC = a,CA = b, gc A = 2x, gc B = 2y, gc C

t s =BC + CA 2R 2r = a

= 2z.
Ta c :0 < x y z v x + y + z =
+ b 2R 2r.

a
b
c
a
b
c
=
=
=
=
=
sin A
sin B
sin C
sin2x
sin 2y
sin 2z

2R =

p dng cng thc sau :

A
B
C
sin sin = 4R sin x sin y sin z
2
2
2
Ta suy ra c s = 2R(sin2x + sin2y -1 4sinxxinyxinz).
r = 4R sin

+) nu ABC l tam gic vung ti C vi C = 2 .Ta c : 2R = c v 2r


= a + b c s=0.
Do ,chng ta nhm tha s chung cos2z trong biu thc s :

s
= 2 sin (x + y) cos (x y) 1 + 2(cos (x + y) cos (x y)) sin z
2R
= 2 cos z cos (x y) 1 + 2 (sin z cos (x y)) sin z

33

thi olympic Trung Quc


= 2cos (x y) (cos z sin z) cos2z

cos2 z sin2 z
cos2z
= 2cos (y x) .
cos z + sin z


2 cos (y x)
=
1 cos2z
cos z + sin z

T chng ta c th a vo gi tr cosz + sinz bi n l dng khi


0 < z < 2 .
Ch rng : y x < min {y, x + y} min{z, 2 z}.

nn ta c:
n

o
cos (y x) > max cos z, cos
z
= max {cos z, sin z}
2

V z

T suy ra :

2 cos (y x)
1 >0
cos z + sin z

.c

V vy s = pcos2z i vi p > 0 hay s = BC + CA - 2R - 2r c th


dng,bng 0 hoc m nu gc C tng ng l nhn,vung,t.

.4.28.Dy s v hn

a1 , a2 , ...

c xc nh 1 cch quy nh sau :

a1 = 0, a2 = 1


v an = 21 nan1 + 12 n (n 1) an2 + (1)n 1 n2
Vi n 3,tm mt cng thc nh ngha cho hm :

fn = an + 2 (n1 ) an1 + 3 (n2 ) an2 + ... + n nn1 a1

Li gii: Cch gii 1: Vit li mi quan h quy thnh :


1
1
an = (1)n + nan1 + n((1)n1 + (n 1) an2 )
2
2

Nu
(1)n1 + (n 1) an2 = an1
Ta c:

1
1
an = (1)n + nan1 + nan1 = (1)n + nan1
2
2

34

Nguyn Hu in, HKHTN H Ni


Do dng phng php quy np t biu thc
an = (1)n + nan1
ta d dng tm ra c:
an = n!

n!
n! n! n!
+
+ ... + (1)n
1! 2!
3!
n!

V th,theo cng thc ni ting Euler


an l chui s ca s xo trn ca b s (1, 2, . . . , n), ngha l s hon
v ca b n s m khng c im c nh. mi cp (, j)ca hon v
phn bit t 1 phn t hay 1 s nguyn j trong dy 1, 2, . . . , n,ta
xc nh 1 im ch nu j l 1 im c nh ca

Vi k xc nh k = 1, 2, . . . , n, c nnk ak hon v vi n-k im xc

nh, c (nnk ) cch chn cc im c nh ny,v chui ak ca k im


cn li.Vi mi hon v nh vy,c n-k cp (, j) c xc nh.

n
P

k=1

Khi tng

= fn

n
nk

n
X
k=1

(nnk )ak = fn (n! 1)


ak m c n! 1 hon v t hn n im c nh.

k=1

(n

k) (nnk )ak

n
X

.c

Xt tng cc hon v,ta c tng s im ch c xc nh:

Mt khc:
Vi mi j, j {1, 2, ..., n} c (n 1)! 1 hon v phn bit t phn t

xc nh j.
V vy,xt ton b tng,ta c tng s im xc nh ch c ch ra
l:
n
P

j=1

((n 1)! 1) = n(n 1)! n

Cho 2 tng trn bng nhau,ta c:


fn = 2.n! n 1
Lu : sau khi ch ra c fn = 2.n! n 1 i vi cc gi tr nh ca
n,ta c th s dng mi quan h quy v ng nht ng thc cc
phn t chng minh cng thc l ng vi mi n.

35

thi olympic Trung Quc

Cch gii 2: Chng ti gii thiu 1 phng php chng t rng an l


chui s ca hon v (1, 2, . . . , n).Vi n 3, ta c :
an = nan1 + (1)n = an1 + (n 1) an1 + (1)n


= (n 1) an2 + (1)n1 + (n 1) an1 + (1)n
= (n 1) (an1 + an2 )

Gi bn l chui s ca hon v ca (1, 2, . . . , n).Mi hon v l mt


trong s nhng dng sau y:
a)Vi k 6= 1 1 nh x ti k v k nh x ti 1.Nh th s c n-1 gi tr
cho k v vi mi k c bn2 hon v cho n-2 phn t cn li .Do vy,c

(n 1) bn2 hon v nh trn.


b) 1 nh x ti k nhng k khng nh x ti 1.gi tr k c nh. Nh vy

.c

7 t

tn ti 1 song nh gia cc hon v v cc hon v c ch s 1 l c


nh,thng qua nh x

y t l s chuyn i gia 1 v k.Bi c bn1 nh x trong ch s


1 l c nh ,nn c bn1 php hon v .

Cho k bin thin t 2 n n,ta thy c (n 1) bn2 hon v ca dng


(b).

minh.

V vy bn = (n 1) (bn1 + bn2 )
T a1 = b1 = 0 v a2 = b2 = 1, an = bn vi n 1 nh yu cu chng
.4.29.Mt cu lc b bng bn mun t chc 1 gii u i,mt lot nhng
trn u m trong mi trn u mt cp ngi chi s thi u vi mt
cp khc.Gi s trn u ca mt ngi chi trong mt gii u l s
trn u m anh(c) ta tham gia.
Cho dy s
A = {a1 , a2 , ..., ak }
phn bit,nguyn dng,chia ht cho 6.
Xc nh s lng ngi chi ti thiu c th thit lp mt gii u
i m :

36

Nguyn Hu in, HKHTN H Ni


(i) mi ngi tham gia nhiu nht l 2 cp u.
(ii) bt k 2 cp khc nhau c nhiu nht 1 trn u gp nhau.
(iii) nu 2 ngi chi cng mt cp,h khng bao gi phi thi u vi
nhau.
(iv) S lng cc trn u ca ngi tham gia c thit lp l A.
Li gii: B .
Gi s rng :k 1 v 1 b1 < b2 < < bk .Nh th tn ti mt

th c bk + 1 nh trong b s {b1 , b2 , ..., bk } l s ca cc nh


trong bk + 1 nh.
Chng minh:
Ta chng minh b bng phng php quy np theo k.

Nu k = 1,ton b th gm nhng nh b1 tha mn.


Nu k = 2,ly b2 + 1 nh, phn bit cc nh ny vi nh b1 v ni

.c

2 nh bng mt ng thng khi v ch khi mt trong s cc nh l


phn bit.
Ta cn chng minh b ng vi k = i 3 v gi s n ng khi

k < i .Ta dng th G ca bi + 1 nh,to thnh cc ng thng gia


hai im v t thay i ca cc nh trong mi nh.Chn nhng

im c bi + 1 nh,v chia chng ,v chia chng thnh 3 b gi tr


S1 , S2 , S3 vi |S1 | =b1 , |S2 | = bi1 b1 + 1, v |S3 | = bi (bi1 + 1).

Theo gi thit quy np,ta c th dng cc ng thng gia cc


nh trong S1 trong ca cc nh c xc nh t tp hp

{b2 b1 , ..., bi1 b1 }


Ngoi ra dng cc ng thng c nh trong S1 l im cui.Mi nh
trong S1 by gi c bi , mi nh trong S3 c b1 , v ca cc
nh trong S2 c xc nh t tp hp {b2 , ..., bi1 }.
T kt hp li tt c cc ca bi + 1 nh trong th G c xc
nh t tp {b1 , b2 , ..., bi }.
iu ny hon tt bc quy np v c iu phi chng minh.
Gi s rng ta c 1 gii u i trong n ngi chi tha mn
iu kin a ra.C 1 nht 1 ngi chi,ta gi l X,c s trn u l
max(A).
Gi m l s cp khc anh(c) y phi thi u.Mi cp ny c 2 ngi

37

thi olympic Trung Quc

chi v c tnh l 2m.Bt k ngi chi no c tnh nhiu nht 2


ln theo cch cu thnh ny bi v mi ngi chi thuc nhiu nht 2
cp .
Do ,ngi chi X s phi u vi t nht m ngi chi khc.Nu X
trong j cp (vi j = 1 hoc 2),s c nhiu nht tng s m + j + 1 ngi
chi.
Ngoi ra X chi nhiu nht jm trn,ko theo jm max (A) .
Do n

m + j + 1

max (A) /j + j + 1
min {max (A) + 2, max (A) /2 + 3}.

V max (A) 6 ,ta c max(A) + 2 > max(A)/2 + 3,ko theo


n max (A) /2 + 3

Ta cn chng minh n = max(A)/2 + 3 l s trn nhiu nht c th


t c.
+ 1 nh trong c

max(A)
6

T b ,ta c th dng th ca
xc nh t tp hp { a61 , a62 , ..., a6k }.

.c

Chia n ngi chi trong max(A)


+ 1 thnh ba phn ,v 2 ngi chi
6
cng trong 1 i khi v ch khi h cng nm trong mt phn ca ba
phn trn .Gn cho mi phn (v cng mt thi im,hnh thnh 3 cp

ngi chi vi nhng cu th chi tng ng) ng vi cc nh ca


th G,v 2 i c xc nh thi u khi v ch khi cc nh tng ng

l lin k.
Gi s rng ta c 1 i c sp xp nh v

ai
.Vi
6

mi

ai
6

nh w lin k vi v, l i phi u trong mt phn ba c sp xp


cho nh w tng cng a2i trn.
Mi ngi chi c xp trong v l trong 2 i,t c s trn u l
2. a2i = ai .
V vy s lng cc trn u ca ngi tham gia chi l {a1 , a2 , ..., ak}
,khi cn thit.

.4.30.Cho s nguyn n 2 . i vi bt k tp hp n s ca dy s thc


A = (a1 , a2 , ..., an )
Cho li im ca A l s k {1, 2, ..., n} trong ak > aj vi mi gi
tr 1 j < k .

38

Nguyn Hu in, HKHTN H Ni


Xt tt c cc hon v A = (a1 , a2 , ..., an ) ca (1, 2, . . . , n) vi im
li im l 2.Xc nh v chng minh ngha s hc ca phn t u
tin a1 trong hon v ny?.
Li gii: Vi mi tp hp n s ca dy s thc
A = (a1 , a2 , ..., an )
Nu ak > aj vi mi gi tr 1 j < k, ta gi ak l 1 li im.
Nu mt hon v
A = (a1 , a2 , ..., an )
ca (1, 2, . . . , n) c s li im l 2 th 2 li im ny phi l a1 v

n,trong n = ak i vi nhng gi tr k tha mn 2 k n.


C nh m trong dy s {1, 2, ..., n 1}.Ta gi cc s m + 1, m + 2, ...n

.c

l cc s ln, v 1, 2, ..., m 1 l cc s b.Trong mt hon v vi 2 li


im ak = m, n s phi xut hin trong hon v trc tt c cc s ln
khc.V vy, xc nh tt c cc hon v ny, ta chn n m v tr u

tin l cc s ln,t n ti v tr u v sp xp n m 1 cc s ln
khc vo phn cn li ca cc v tr chn.Sau sp xp tt c cc

s b vo m 1 v tr cn li.

n1
V th,ta c xm = nm
(n m 1)! (m 1)! =

(n1)!
nm

hon v

V vy ngha s hc ca phn t u tin a1 c trung bnh cng mong


mun l:

Pn1 m
Pn1 m
Pn1
(n 1)! m=1
m=1 nm
nm
m=1 m xm
=
Pn1
Pn1 1 = Pn1 1
(n 1)! m=1 nm
m=1 xm
m=1 nm
Pn1 n Pn1 m
m=1 m
n1
m
=n
= m=1P
n1 1
1
1 + 2 + ... + n1
m=1 m

.4.31.Tm tt c cc s nguyn dng n trong

n1 , n2 , ..., nk > 3
vi
1

n = n1 n2 ...nk = 2 2k (n1 1)(n2 1)...(nk 1) 1

39

thi olympic Trung Quc

Li gii: Nu mt s nguyn dng n tha mn iu kin a ra,th


n = 2m 1 vi m l s nguyn dng.D dng kim tra thy 3 l s

nguyn m duy nht nh hn 10 lm cho n = 2m 1 tha mn iu kin


a ra.

Cho m 10,ta phi chng minh 2m 1 khng tha mn iu kin a


ra.
Gi s, ch ra s mu thun, lp phng trnh biu din mt s k v
n1 , n2 , ..., nk :
1
(n1 1) (n2 1) ... (nk 1) 10
2k
3
3
Vi ` 10 ,ta c `+1
< 54 < 2
`
S dng kt qu ny,ta d dng chng minh c bng phng php

m=

.c

quy np 2` 1 > `3 vi cc s ngyn ` 10


V vy
3 
3 
3

n2 1
nk 1
n1 1
m
3
...
. (1)
2 1 >m =
2
2
2

ni 1
2

3

5.Do

V n = 2m 1 l l, ni l chn,v vi mi ni > 3, ni nh nht phi l


4

ni 1
> ni (2)
2

Cho i = 1, 2, . . . , k. t (1) v (2) cng nhau,ta thu c


n = 2m 1 > n1 n2 ...nk = n
iu ny mu thun.
Nh vy gi thit a ra l sai v n = 23 1 = 7 l p n duy nht.
.4.32.Mt bi thi bao gm 5 cu hi nhiu la chn,mi cu c 4 la chn
khc nhau.C 2000 sinh vin lm bi thi,v mi sinh vin ch c chn
ng 1 p n trong mi cu hi.
Tm gi tr nh nht ca n v nhng bi thi ca sinh vin tha mn iu
kin: trong bt k n bi thi,tn ti 4 bi thi trong bt k 2 bi no c
nhiu nht 3 p n ging nhau.

40

Nguyn Hu in, HKHTN H Ni


Li gii: Trc tin,ta chng minh n 25 .Gi 1,2,3,4 l cc la chn
ca tng cu hi. Th hin mi cu tr li ca sinh vin bng mt chui
c trt t bao gm 5 phn t
(a1 , a2 , a3 , a4 , a5 ), ai {1, 2, 3, 4}
Trong p n ca sinh vin trong cu hi i l ai .Ta ni rng 2 bi thi
l cng loi nu chui s 5 phn t ca n thuc vo mt b ca dng
sau
{ (k, a2 , a3 , a4 , a5 ) |k {1, 2, 3, 4} },
Trong
a2 , a3 , a4 , a5 {1, 2, 3, 4} .

V v c 256 b,v 2000 = 256x7 + 208,nn c t nht 8 cu bi thi l


cng loi theo nguyn l Dirichlet. Trong 1992 bi thi cn li,c 8 bi
na cng loi.Cui cng,trong 1984 bi cn li,thm 8 bi cng loi na.

.c

Xt tp A ca 24 bi thi ny.Cho 2 bi thi bt k trong tp A,chng phi


cng loi,ngha l,p n cho 4 bi thi cui cng l ging nhau.iu ny

mu thun vi gi thit c 4 bi thi trong tp A,trong c 2 bi bt


k c nhiu nht l 3 cu tr li ging nhau.

Do n 25
By gi chng ta ch ra rng n=25 l kt qu chp nhn c.Xc nh
b s

S = { (a1 , a2 , a3 , a4 , a5 ) |

X5

i=1

ai 0 (mod4) , ai {1, 2, 3, 4}}.

Khi |S| = 44 = 256 v bt k 2 bi thi c nhiu nht 3 p n ging


nhau nu 5 chui phn t tng ng ca chng l cc yu t phn bit
ca S.
Chn bt k 250 yu t ca S,v gi nh rng chnh xc c 8 bi thi
tng ng vi mi chui 5 phn t trong 250 yu t ny.V 25 > 3 nn
c 8 bi thi,c 4 bi v c 5 chui tng ng l cc yu phn bit ca
S,v tha mn iu kin a ra.
V vy, n = 25.

Chng 5
thi olympic Tip khc
.5.33.Chng minh rng:
3

2(a + b)

1 1
+
a b

a
+
b

a2 +

b2

.c

vi mi s thc a, b. Du "=" xy ra khi no ?

3
Li gii: Nhn 2 v ca bt ng thc vi ab ta c bt ng thc
tng ng
p
3
2(a + b)2

t 3 a = x, 3 b = y, chng ta thy rng iu tha mn chng t


rng
p
x2 + y 2 3 2(x3 + y 3 )2 ()
vi x, y > 0.
p dng bt ng thc trung bnh ta c

3x4 y 2 x6 + x3 y 3 + x3 y 3
Du "=" xy ra nu v ch nu x6 = x3 y 3 = y 6 hoc x=y.
Cng 2 v ca 2 bt ng thc vi nhau v cng x6 + y 6 vo 2 v ca
bt ng thc ta c:
x6 + y 6 + 3x2 y 2(x2 + y 2 ) 2(x6 + y 6 + 2x3 y 3)
Bt ng thc (*) xy ra khi x=y hoc a=b.

42

Nguyn Hu in, HKHTN H Ni

.5.34.Tm tt c t gic li ABCD m tn ti mt im E nm bn trong t


gic tha mn iu kin sau y:
Bt k ng thng no qua E v ct cc cnh AB v CD u chia t
gic li ABCD thnh 2 phn bng nhau.
Li gii: Gi s rng t gic li ABCD c tnh cht . Ly X1 , X2 , X3
l 3 im nm trn cnh AB
vi AX1 < AX2 < AX3 . Nh vy cnh Xk E ct on CD ti Yk vi
k=1,2,3.

Suy ra EX1 .EX2 = EY1 .EY2


Tng t c EX2 .EX3 = EY2 .EY3

Do t gic ABCD li v CY1 < CY2 < CY3 nn ta c:


0 = 21 [ABCD] 12 [ABCD] = [AX1 Y1 D] [AX2 Y2 D]
= [EY1 Y2 ] [EX1 X2 ] = 12 sin Y\
1 EY2 (EY1 .EY2 EX1 .EX2 )

Do EX1 /EY1 = EX3 .EY3 v Y1 EY3 X1 EX3


Cho nn X1 X3 k Y1 Y3 . iu c ngha l AB k CD

.c

Mt khc ta c ABCD l t gic li vi AB k CD. C E l trung im


ca on M1 M2 . im M1 v M2 tng ng l trung im ca on AB

v CD.
Gi s mt cnh i qua E v ct cnh AB ti X v ct cnh CD ti
Y. Kt qu l n i qua M l giao im ca AB v CD.

V XM1 vi Y M2
iu ta c XM1 = Y M2 v AX + DY = BX + CY

Khi mi t gic AXYD v BXYD l hnh thang hoc hnh bnh hnh
c cng ng cao v cng chiu di cnh y.
Bi vy chng c din tch bng nhau.(pcm)
.5.35.Bi 3: Cho tam gic u ABC vi cnh y AB v ng cao CD. im
P nm trn cnh CD. im E l giao im ca AP vi BC v F l giao
im ca BP vi AC. Gi s rng ng trn ni tip tam gic ABP v
t gic PECF bng nhau. CMR ng trn ni tip tam gic ADP v
BCP cng bng nhau.
Li gii: Cho 1 v 2 l ng trn ni tip t gic CEPF v tam gic
ABP, v hai im I1 v I2 ln lt l tm ca ca 2 ng trn 1 v

43

thi olympic Tip khc

2 .
Bi v chng i xng qua CD, I1 v I2 l 2 im trn on CD vi P
nm gia 2 im .
Bi v 1 v 2 l hai ng trn bng nhau v v ni tip trong gc
i nh, chng i xng qua mi im P.
Do P I1 = P I2
Do t gic ADBP v BDP ng dng, chng ta ch cn chng minh
rng bn knh ng trn ni tip r1 ca tam gic BCD bng bn knh
ng trn ni tip BDP.
Gi X v Y tng ng l giao im ca 3 ng phn gic trong ca
tam gic BCP v BDP.
Nhn xt rng I1 nm trong tam gic CBF, nh vy I1 nm trn ng
phn gic ca gc CBF, tc l gc CBP.

Nh vy X nm trn on BI1 v tng t Y nm trn BI2


Do P I1 = P I2 , [BI1 P ] = [BI2 P ] nn

.c

r1 (P I1 + BP ) = 2 ([I1 P X] + [XP B]) = 2 [I1 BP ] = 2 [P I2 B] =


2 ([P I2 Y ] + [P IB])
= r2 (P I2 + BP )

Do r1 = r2

.5.36.Cho 2000 im trong tam gic ca mt b mt thuc mt mt phng.


Vi mi nh ca hnh tam gic qua mt php tnh tin l 1 tam gic bao
22
.
9

gm trng tm.
Chng minh din tch ca cc tam gic ny nh hn

Li gii: nh hng ca mi hnh trong bi ton ca 2000 im


cho trong mt tam gic c 1 cnh cng vi nh i din trn.
cho tam gic ABC c 1 tam gic vi AB nm ngang v A nm bn
tri B. Nh vy khng c 1 im no khc trong 1999 hnh tam gic c
b mt nm ngang bn di cnh AB.
Chng ta bt u bi khi nim v khong cch t khong cch Oclit
v chng ta nh ngha chnh thc m t mt vi mi lin h gia 2000
tam gic.
Xc nh mt im hay k mt ng thng song song vi BC. cho
khong cch d t mt im khc tn ti khong cch gia hai ng

44

Nguyn Hu in, HKHTN H Ni


thng song song n BC i qua tam gic .
Chn a0 = d (BC, A)
nh ngha tng t vi , v khong cch gia CA v AB vi :
b0 = d (CA, B) > 0, c0 = d (AB, C) > 0 .
Chng ta bit rng s tnh tin nh qua 1 khong cch no cho nh
ging nhau vi khong cch gia v .
Gi s rng XY Z v X0 Y0 Z0 l 2 hnh tam gic trong s 2000 tam
gic vi XY k X 0 Y 0 k AB v Y Z k Y 0 Z 0 k BC.
Php tnh tin bin XY Z thnh X0 Y0 Z0 .
Bi v trng tm ca XY Z l im trn pha tri ca Y 0 Z 0 = T (Y Z),
d (Y Z, T (Y Z)) 31 a0 do :
d (X, X 0 ) = d (X, T (X)) 31 a0 suy ra

4
d (Y Z, X 0 ) a0
3

Tng t d (ZX, Y 0 ) 43 b0 , d (XY, Z 0 ) 34 c0 .

.c

Php tnh tin bin nh tam gic ABC di mt phng cha C vi t


s 13 . Nh th [T ] = 91 . Php tnh tin T ca u1 v v1 ca hai im tng

ng C, A v php tnh tin u1 v v2 ca 2 im tng ng C v B.


Vi T1 , T2 l 2 php tnh tin vi T1 [u1 ] = v1 , T2 [u2 ] = v2 .

cho bao li ca 4 tam gic cha F b chn bi l1 bn phi v l2 bn


tri, l3 trn nh v di y AB. Quan st rng F l 1 nh ca

hnh thang vi din tch 24


.
9
4 0
Bi v d (AB, l3 ) = 3 c , nhng im bao gm trng tm ca tam gic
ABC nm trn hay di l3 . Tng t d (l2 , B) = 43 b0 , d (l3 , C) = 43 c0

theo th t trong tam gic ABC bao gm trng tm ca 1 tam gic


nhng im ca tam gic phi nm trn hoc bn tri l2 hoc nm
bn tri ca l1 .
T hp vi cc tr c xc nh ca tam gic ABC ko theo h qu l
min < b ph bi 2000 tam gic l sai vi hnh thang F c xc nh
ca ng k qua mt song song ca 2000 tam gic.
Do :
[<] [F ]

22
22
1 1
k =
k <
9 9
9
9

45

thi olympic Tip khc

.c

(pcm)

Chng 6
thi olympic Estonia

.6.37.Cho 5 s thc, chn 3 s bt k th hiu ca tng 3 s v tng hai s


cn li s l mt s dng. Chng minh rng tch ca tt c 10 hiu s

.c

(ty theo s kh nng 3 s c chn) nh hn hoc bng tch ca bnh


phng 5 s .

Li gii: Cho 5 s l x1 , x2 , x3 , x4 , x5 ta c 5 hiu s tng ng l


a1 , a2 , a3 , a4 , a5 v b1 , b2 , b3 , b4 , b5 , y:
ai = xi2 + xi1 + xi + xi+1 xi+2

bi = xi2 xi1 + xi xi+1 + xi+2


vi i = 1, 2, 3, 4, 5. Vi mi i, chng ta c

i 2
i 2
x2i ai bi = ( ai +b
) ai bi = ( ai b
) >0
2
2
2
hoc xi > ai bi v ai bi > 0 cho mi i, chng ta c th nhn 5 bt ng

thc ai bi > x2i vi 1 6 i 6 5 th thu c


Q5
Q5
2
i=1 ai bi
i=1 xi >

.6.38.Chng minh rng khng th chia mt tp bt k gm 18 s nguyn dng


lin tip thnh hai tp A v B, vi tch ca cc phn t trong A bng
tch ca cc phn t trong B.
Li gii: Chng minh phn chng. Gi s, chng ta chia c mt tp
S = n, n + 1, , n + 17 ca 18 s nguyn dng lin tip thnh tp A,
Q
Q
B sao cho aA a = bB b v tch ca cc phn t trong A bng tch
ca cc phn t trong B, nu 1 tp cha mt bi s ca 19, th tp cn
li cng phi nh vy. Do vy, S khng cha bi s no ca 19 hoc cha

47

thi olympic Estonia

t nht hai bi s ca 19. V c duy nht 1 trong 18 s nguyn dng


lin tip c th l bi s ca 19, S phi khng cha bi s no. Bi vy,
n, n + 1, , n + 17 ln lt ng d vi 1, 2, 3, , 18 mod 19( chia ly
d). Do vy, theo qui tc Wilson
Q
Q
aA a
bB b = n(n + 1) (n + 17) 18! 1 (mod 19)
Tuy nhin, hai tch ca bn tri l bng nhau, iu ny khng c kh
nng v -1 khng l bnh phng ca php mod 19. Bi vy, khng tn
ti hai tp A v B.

.6.39.Cho M, N v K l cc im tip xc ca ng trn ni tip tam gic


ABC vi cc cnh ca tam gic, gi Q l tm ng trn i qua trung
cc on thng MN, NK, KM. Chng minh rng tm ng trn ni
tip v ngoi tip tam gic ABC thng hng vi Q.

Li gii: chng minh, gi s M, N v K nm trn cc cnh BC, CA,

.c

AB v X, Y, Z l trung im cc on thng NK, KM, MN. Theo


bi, Q l tm ng trn ngoi tip tam gic XYZ. ng thng AX l

trung tuyn on thng KN ca tam gic cn AKN, va l ng phn


gic v l ng cao ca tam gic. Do vy, A, X v I l thng hng,
\ = . Do , Tam gic vung AXK v AKI l ng dng, v
v AXZ

IA.IX = IK 2 . Bi vy, X la hnh chiu ca A di php nghch o qua


ng trn n tip ca tam gic ABC. Tng t, Y l hnh chiu ca

B v Z l hnh chiu ca C vi cng php nghch o. Dn n, nghch


o nh x ng trn ngoi tip tam gic ABC n ng trn ngoi

tip XYZ, nn tm ca ca cc ng trn ny l thng hng vi I. Ni


mt cch khc, Q, I v tm ng trn ngoi tip ca tam gic ABC l
thng hng.
.6.40.Tm tt c cc hm f: N N sao cho:

f(f(f(n))) + (f(f(n)) + f(n) = 3n vi x N .

Li gii: Nhn xt rng nu f(a) = f(b), th vi n = a v n = b cho


biu thc 3a = 3b hay a = b. Bi vy, f l duy nht. Chng ta chng
minh bi qui np vi n Z th f(n) = n. Gi thit rng vi n < n0 ,
f(n) = n Chng ta chng minh rng f (n0 ) = n0 ( Mnh p dng vi
n = 1). V f l duy nht, nun > n0 > k th f (n) 6= f (k) = k, v vy

48

Nguyn Hu in, HKHTN H Ni


f (n) > n0 (*) vi n > n0 .c bit, (*) p dng vi n = f (n0 v tng
t vi f (f (n0 )) Thay n = n0 trong hm cho, chng ta thy:
3n0 = f (f (f (n0))) + f (f (n0 )) + f (n0 ) > n0 + n0 + n0
Du = xy ra khi f (n0 ) = n0

.6.41.Trong tam gic ABC chng ta c AC 6= BC . Ly im X bn trong


b = B,
b =C
b v = BCX.
\ Chng t rng:
tam gic v gi = A,
sinsin
sin()

sinsin
= sin()
Nu v ch nu X nm trn trung tuyn ca tam gic ABC k t im
C.
0
\ v 0 = MCB.
\
Li gii: Ly M l trung im ca AB t = ACM
Khng gim tnh tng qut, gi s > v BC > AC, theo quy tc

ng phn gic, phn gic gc ACB gp cnh AB ti im gn A hn


0
0
B, v d ti mt im trn AM . Do > .

.c

Chiu B qua on CX c im B v t im D trn on CB, trn


0
\ p dng inh
pha i din on thng AB ti C, r rng = BAD.
l Ceva trong tam gic ABC i vi cc on thng ng qui AD, XD
0
0 CB nh vy
\
v BD 4BCB 0 l tam gic cn vi BC = BC v 2 = B
\0 = CBB
\0 CBA
[ = 0
\0 = 0 . Do ABB
CBB
2

\0 = v tam gic MAB l tam gic cn, v


v MA = MB = MB, ABB
2
vy, cc gc ca tam gic ny l:
0 AM = B
0 AB = ABB
\
\
\0 = 0
B
2
0 A = 0 +
\
MB

\0 = 2( 0 + ) = ( ) + ( 0 0 )
AMB
0
0
ng thc cui cng ng v + + + =
\ \
v MAB 0 > 0 = MAD,
D nm gia bn knh AM v AB v do nm
\ = 0 =BCM
\ = MCB
\0 = MCD
\
trong tam gic AXB. Tng t MAD
dn n t gic MCAD l ngoi tip. Chng ta tnh cc gc cc on
AD,MD v BD to vi cc cnh ca tam gic AMB. u tin chng
0
0
0 M = CB
0M =
\ = 0 , DMA
\ = \
\
\
ta c MAD
DCA = v DB
0 M = Kt hp cc biu thc ca cc gc tam gic AMB, ta c
\
DB
biu thc
\0 = MAB
\0 MAD
\ =
DAB

49

thi olympic Estonia


0 MD = B
0 MA DMA
\
\ =
B\
0 D = AB
0 M DB
0M = 0
\
\
\
v AB

p dng nh l Ceva v nh l sin, chng ta thy:


0 DsinB\
0M D
\
\
sinM
ADsinAB
0 M sinDM
\0 sinDB
\
\
sinDAB
A
0
0
sin()
sin
sin
= sin sin sin( 0 0 )
0
sin()
B
=M
sin
sin(
0
MC
sin
0 )
0
sin()
sin
MA
=M
sin sin( 0 0 )
C
0
0
sin()
= sin
sin
sin(
0
0
sin
sin
)

1=

Nu X nm trn on CM, th (, ) = ( , ) v biu thc bng nhau


trn dn n biu thc cho. Ngc li, gi s c biu thc
[ v t f () = sin.sin() . Chng c cho rng
cho. t = BCA
0

sin(())

f () = f ( ), gi tri chung ny 6= 0 (nonzero) v 0 < , < v vy


0

c nh ngha v nhn gi tr ti = v = . Tuy nhin:

1
f ()

.c

1
sin.cos( ) cos.sin( )
=
= cot cot( )
f ()
sin.sin( )

y l mt hm ng bin nghch vi 0 (0, ). Bi vy = gamma

v X phi nm trn on thng CM. Mnh c chng minh.

.6.42.Chng ta gi mt tp v hn dy cc s nguyn dng l F - sequence


nu mi s hng ca tp(Bt u t s hng th 3 ) bng tng ca 2 s

hng trc . C th tch tp tt c cc s nguyn dng thnh


a) Hu hn

b) V hn
s lng ca F - sequence khng c cc phn t chung?
Li gii: a) Gi s chng minh phn chng tn ti m tp F c
phn chia t cc s nguyn dng. t dy ith l F1 , F2 , ,, v
(i)
(i)
(i)
(i)
Fn+2 , Fn+1 tng vi n > 2 nn tn ti Ni sao cho Fn+2 , Fn+1 > m vi
(i)

(i)

mi n > Ni .
t N = max{N1 , N2 , , Ni }, v chn mt s nguyn dng k vt

qu s hng N u tin ca mi dy s. Theo nguyn l Pigeonhole,


hai trong s cc s k, k + 1, , k + m tn ti trong cng dy F cc s
nguyn ny khc nhau ti mi m, y l mu thun. V vy cu tr li
i vi phn a) l khng.

50

Nguyn Hu in, HKHTN H Ni


b) nh ngha dy Fibonasi {Fn } vi F0 = F1 = 0 v quy Fn+1 =
Fn + Fn1 vi n > 1 n c th hin th bng cch quy np j c duy
nht biu din Zeckendorp ak .ak1 .k1 vi cc tnh cht ak = 1, mi
P
ak = i hoc 1 khng c 2 s lin tip bng 1 v j = ki=1 ak .Fk . C v

hn cc s nguyn dng m c biu din Zeckendorpf kt thc vi a =


1. Vi mi m, nh ngha mt dy Fm nh sau: t s hng th n l s
Pn
k=1 ak .Fk+n1 biu din Zeckendorpf l ak .ak1 .k1 tip theo n - 1
cc s 0. Th tng ca s hng th n v n+ 1 l
Pn
Pn
k=1 ak .Fk+n
k=1 ak .Fk+n1 +
Pn
= k=1 ak .(Fk+n1 + F k + n)
P
= nk=1 ak .Fk+n+1

L s hng th n+2. Do , Fm l mt dy F, bt k s nguyn dng


j tn ti trong F ng vi mt s nguyn dng m, mt biu din

.c

Zeckendorpf biu din l ging j, tr bt k cc s 0. Do , cc dy s


nguyn dng, dn n cu tr li cho phn b) l c.

Chng 7
thi olympic Hungary

.7.43.Tm tt c cc gi tr dng ca p bit rng tn ti cc s nguyn dng


n, x,y tho mn pn = x3 + y 3

.c

Li gii: p=2 v p=3 ta c: 21 = 13 +13 v 32 = 13 +23 Gi s rng p>3


v khng xy ra mu thun trong phngtrnh gm 3 s thc dng.

Chn n, x, y sao cho n nh nht V p 6= 2 ta c (x, y) 6= (1, 1) . Do


x2 xy + y 2 = (x y)2 + xy > 1 nh l (x + y). V c 2 gi tr ny u
chia ht cho x3+y3, chng phi l nhng bi s ca p. Do p phn

tch thnh (x + y)2 (x2 xy + y 2) = 3xy.V p khng chia ht cho3,


p chia ht cho t nht 1 s hoc x hoc y. Hn na p khng th chia

ht ch l mt trong x v y v p/(x+y). Theo pn = x03 + y 03 . Ti


(n,x,y) +(n-3, x/3, y/3). Nhng n < n tri vi gi thit n nh nht.

Vy ch c p=2 v p=3 tha mn yu cu bi ton.


.7.44.C hay khng mt a thc f bc 1999 vi h s l f(n) , f(f(n)) , f(f(f(n))),
. . . l cc cp gi tr tng i vi bt k s nguyn n no?
Li gii: Cho g(x) l a thc bt k bc 1997 vi nhng h s nguyn
v cho
f(x) =x(x-1)g(x) +1 . Ta chng minh rng nu f tho mn yu cu bi
ton th mnh trn ng.
Tc la phi ch ra rng vi n l s nguyn bt k v p l 1 gi tr phn
tch thnh f(n), vy th p - f k (n) vi bt k s nguyn dng k>1.
c bit hn, ta chng minh rng f k (n) 1(modp)vi mi k> 1. Ta

52

Nguyn Hu in, HKHTN H Ni


t k chng minh bi ton. Bit rng cho mt a thc h vi cc h
s nguyn a b(modc) tc l h(a) h(b)(modc). Xt trng hp k=2,

f (n) 0(modp) suy ra f (f (n)) f (0) 1(modp).


Vy th f (f k(n)) f (1) 1(modp). iu phi chng minh.

.7.45.Chn ng cc ng phn gic ca cc gc tam gic ABC l X, Y v


Z. ng trn ngoi tip tam gic XYZ ct 3 on thng AB, BC v
CA. Chng minh rng tng di ca hai trong nhng on thng ny
ph thuc vo di cnh th 3.
Li gii: Ta t cc k hiu khong cch trong sut bi ton ti
a=BC ,b=CA , c=AB l dng. ng thi, cho ng trn ct BC ti
X v P, CA ti Y v Q, v AB ti Z v R, t x = PX, y=QY, z=RZ.

Theo nh l ng phn gic ta c: YA=bc/(c+a) , AZ=bc/(a+b). Do


, QA=bc/(c+a)+y , AR=bc/(a+b)-z. p dng nh l v phng tch

+ y) =

.c

bc
( bc
c+a c+a

ca 1 im cho im A ta c

bc
( bc
b+a ab

z)

a
z
b+a

abc
(b+a)2

abc
(a+c)2

b
x
b+c
c
y
a+c

abc
(b+c)2
abc
(a+c)2

abc
(a+b)2
abc
(c+b)2

c
y
c+a

Sau khi nhn cc v vi a/bc v sp xp li ta c

Tng t ta c

b
z
b+a
c
x
b+c

+
+

Khi cng 3 phng trnh li ta nhn ra iu n gin lx+ y +z = 0.


Theo hai trong cc s hng x,y,z phi cng du v s hng th 3
phi tri du hai s cn li. Do tng gi tr tuyt i ca hai s hng
trc bng tr tuyt i ca s hng sau, l iu phi chng minh.
.7.46.Cho kv t l cc s nguyn t cng nhau ln hn 1. Bt u t php
hon v (1,2,. . . .. n) ca dy s 1,2 . . . . . . ..n, ta co th i 2 s nu
chng khng l k hoc t . Chng minh rng ta c th thc hin bt k
php han v no ca 1,2,. . . n khi v ch khi n > k + t 1

Li gii: Dng th G m cc nh ca n l cc s nguyn 1,2,. . . .n


vi hiu ng bin gia a v b khi v ch khi |a b| {k, t} Ta thy iu

53

thi olympic Hungary

kin trva nu tng ng vi (i) mi php hon v c th thc hin


c, (ii) G l ng lin; (iii) n > k + t 1 (i) (ii) V mi bi trao

i 2 s trong cng 1 nhnh ca G, Theo khng s no c th v v


tr c c thay th bi 1 s mt b phn khc. Vy th , ta khng

th thc hin c mi php hon v tr khi tt c cc s u nm trn


mt nhnh.
(i) (ii): Ta chng minh bng php quy np trn m rng xc nh
mt th lin vi cc s nguyn m., bt k php hon v no ca cc s
nguyn ny c th c c t php hon v khc bt k tao bi s trao
i lin tip ca cp (a,b), ti a v b l cc nh lin k ca th.
Yu cu r rng khi m = 1. Mt khc, chn mt nh a sao cho phn
th cn li l ng lin sau khi b a v du, ta c th cho im
A l im cui ca cc nh. Mt s qu o ca cc nh phn bit

a0 a1 . . . ar ni lin a0 = 1 (a) v ar = a Bng cch chuyn v tr lin


tip (a0 a1 ), (a1 a2 ) . . . (ar1 ar ) ta c th di chuyn a ti v tr ban u

.c

chim ch bi 1 (a) Bng gi thit quy np, cc bin s thay th a c


th hon v khi cn, do t c hon v . p dng b ny vi
m=n v th G chng minh rng (ii) (i)

(ii) (iii): Nu t nht k l n th mi cnh bin s ni 2 bin s ng


d theo mun t. Vy s khng c ng no gia 1 v 2 (mu thun

). Do , ta phi c k <n; tng t , t<n . Vy c n k cnh bin ca


dng thc {a, a + k} v n-t ca cnh bin ca dng thc{a, a + t} S

lin thng yu cu ita nht n -1 cnh bin, do (a k) + (n t) >


n1 n>k+t1
(iii) (ii): Tt nhin trong trng hp ny k,t < n. ch rng 2 s
m ng d m un k s c ni vi nhau ( cc cnh bin ngang qua
ca dng {a, a + k} ) do ch ra rng tt c cc s 1,2,. . . .,k

c ni ln nhau. V t l ga tr tng ng vi k nn 1, 2t, 3t, . . . , kt


biu din bi tt c cc ng d mun k. Do , ta c th sp xp li
1, 2, . . . ., k theo th t b1 , b2 , . . . , bk ti bi it(modk). Ch rng
k 0 kt(modk) do bk = k Vy khi k 1 > i > 1 ta c k 1 > bi

v v th nn n > k + t 1 > bi + t Do tn ti nh bi + t v c
ni bi mt cnh bin ti bi Hn na, bi + t bi+1 (modk) nn bi + t

54

Nguyn Hu in, HKHTN H Ni


c ni ti bi+t . V th cho nnbi , c ni ti bi+1 trong G vi mi
i = 1, 2, . . . , k 1. Vy cc s ny c ni lin ln nhau v bi ton

c gii quyt.

.7.47.Cho s nguyn dng bt k k, cho e(k) l s c dng chn ca k. v


P
cho o(k) l s c dng l ca k. Vi n>1, chng minh rng : nk=1 e(k)
P
v nk=1 o(k) khc nhau vi hu ht n.
Li gii: S nguyn c th chia c bi d nm chnh gia dy s
1,2,. . . ,n l |n/d| Do , tng ca o(k) (rerp.e(k)) n khong k=

1,2. . . ,n bng vi tng ca |n/d| n khong tt c cc s nguyn dng


n
] v s nguyn dng a v n, ta c
chn d. V [ nd ] > [ d+1
Pn

k=i

o(k)

Pn

k=i

e(k) =

n
k=i ([ 2i+1 ]

n
[ 2i
])

o(k)

Pn

e(k) = [ n1 ]

.c

k=i

k=i

Pn

t:

Ti tng hn c xc nh v cc s hng bng 0 khi i> [ n2 ]. Tng


P

n
k=i ([ 2i ]

n
[ 2i+1
]) 6 n

.7.48.Cho mt tam gic trong khng gian, hy nu cch dng im P nm bn


trong tam gic tho mn iu kin sau: nu h mt ng thng vung

gc qua P ti cc nh ca tam gic. Chn ca cc ng vung gc to


thnh mt tam gic nhn P l trng tm.

Li gii: a: Gi tam gic l ABC vi di cc cnh l a=BC, b= CA,

c=AB. Gi P l mt im nm trong tam gic, v tr xc nh sau. Gi X,


Y, Z l chn cc ng vung gc k ti cc cnh BC, CA, AB, theo th
[ =
t , x =PX, y= PY, z=PZ. Chu rng sin Y[
P Z = sin( BAC)
[ Tng t sin ZP
\
[ v sin XP
\
[
sin BAC.
X = sin CBA
Y = sin ACB.
Tng ng vi cc iu sau
P l trng tm tam gic XYZ
Cc tam gic YPZ,ZPX, XPY c din tch bng nhau
\
\
yz sin Y[
P Z = zx sin ZP
X = xy sin XP
Y

sin<BAC/x = sin<CBA/y = sin<ACB/z


a/x = b/y = c/z (nh l hm Sin)

Dng ng thng song song vi BC, cch BC mt khong l a nm

55

thi olympic Hungary

cng pha vi A. Tip tc dng n thng song song vi CA, cch


CA mt khong l b, nm cng pha vi B. t Q l giao im ca
chng,ch rng tia CQ i qua min trong tam gic Ly P bt k trn
CQ, xt t s khong cch ti BC vi khong cch ti AB. Nu P=Q
t s ny bng a/b ;bi v mi im P l hnh nh ng dng ca nhau
vi C,nn t s ny khng ph thuc vo P v lun bng a/b. Hn na
ta c th dng 1 tia t A hng vo tam gic m vi mi P trn tia
th t s k/c t AB vi k/c t CA bng c/b. Hai tia ny giao nhau ti
im P no trong tam gic . Nu ta t P l giao im ca chng,

.c

ta thu c a/x = b/y v b/y = c/z ,v bi ton c li gii.

Chng 8
thi olympic India

.8.49.Cho 4 ABC khng u. Gi P l mt im nm trong tam gic . Cc


on thng qua P v ni nh vi cc cnh i din ca tam gic

c di bng nhau v bng tha mn < min(AB, BC, CA). Chng


minh rng c im P , 6= P tha mn cc tnh cht ging nh P.

.c

Li gii: Xt 3 on l AD, BE, v CF , gi cc ng cao ca tam


gic l AHa , BHb , v CHc .

Ly i xng cc on thng trn qua cc ng cao tng ng ta


,
,
,
,
thu c cc on AD , BE , v CF . Nu AD khng nm trong tam

gic th ta suy ra c l AB hoc AC phi nm trong 4ADD , . Th


nhng iu ny khng xy ra v AD = AD , = < min(AB, AC).
,
,
,
Do vy AD phi nm trong 4 ABC, tng t nh vy BE , v CF

cng phi nm trong 4 ABC.


By gi ta dng di hnh hc. Ch rng

BD.BD, = (BHa +Ha D)(BHa Ha D) = BHa2 = (AB 2 AHa2 )(AD 2 AHa2 )


= AB 2 AD 2 = AB 2 2 (8.1)

Tng t nh vy ta c EA.EA, = AB 2 2
Tht vy, EA.EA, = BD.B , D. Nh vy F B.F , B = CE.CE , v
DC.D,C = AF.A, F

57

thi olympic India

By gi p dng nh l Ceva cho 3 ng ng quy AD, BE, v CF


. CE . AF = 1
ta c BD
DC EA F B
Do vy

BD , CE , AF ,
BD, BD CE , CE AF , .AF
.
.
=
(
).(
).(
)
D,C E , A F , B
D , C.DC E , A.EA F , B.F B
BD , .BD CE , CE AF , .AF
=( ,
)(
)(
) = 1 (8.2)
E A.EA F , B.F B D , C.DC
,

Nhng theo nh l Ceva AD , BE , v CF ng quy ti im P , nm


ngoi 4 ABC. Nu P trng vi P , th P s l trc tm ca tam gic,
nhng nu vy th theo gi thit 3 ng cao ca 4 ABC c di
bng nhau v bng .. iu ny l v l, v 4 ABC khng u. Do

P , 6= P , v cc on thng qua P , c di bng nhau v bng .

.8.50.Cho m, n l cc s nguyn dng tha mn m

n2
4

v mi c nguyn

.c

t ca m u nh hn hoc bng n. Hy chng minh rng m l c ca


n!

Li gii: Ta ch ch cn chng mnh rng vi mi c nguyn t p ca


m v vi mi s nguyn k 1 m P k |m th pk |n! (1)

Tht vy nu k = 1 thy ngay (1)ng v p n. v do vy p|n! . Mt


p
2
2
khc v m n4 nn ta c pk n4 . hay n 2 pk .
Ta thy, nu n kp th t nht k s thuc vo tp 1, 2, ....., n l bi
ca p, ko theo pk |n!.

pk

k2

chng minh n kp ta i chng minh 2 p k p 2 k2 (*) (V


p
khi n 2 pk kp)
i chng minh (*): Vi k = 2 th (*) 1 1 (ng). Nu k 4 th p

.(p 1) k2 .
dng BT Bernoulli ta c p 2 = (1 + (p 1)) 2 1 + k2
2
Cui cng, nu k = 3 th (*) lun ng ngoi tr p = 2. Nu k = 3 v
k2

k2

p = 2 th m 8; n 5 v n! thc s chia ht cho 8 (8 = 23 ), do vy ta


c pk l c ca n! trong mi trng hp, bi ton c chng minh.
.8.51.Cho G l mt hnh vi n nh vi n 4 v m cnh. Chng minh rng

nu m > n( 4n 3 + 1)/4 th G cha mt 4- Chu trnh.


Li gii: Chng ta m s cc b 3 cc khong cch gia cc nh (v, a,
b) sao cho v l cnh k c a v b. S cc b nh th vi mi v c nh l

58

Nguyn Hu in, HKHTN H Ni


deg(v).(deg(v)-1). V tng cc deg(v) ly theo tt c cc v l 2m, v x(x1) l hm li theo x, bt ng thc Jensen cho ta tng deg(v)(deg(v)-1)
ly theo tt c cc v b nht l n.(2m/n)((2m/n)1) = 2m, (2m/n1).
Nu G khng c 4-chu trnh no, th vi bt k a, b c nh no c nhiu
nht mt nh k vi c a v b. Ko theo c nhiu nht n(n-1) b 3
ni trn. Do vy, c mt 4- chu trnh nu 2m, (2m/n1) > n(n1)

4m2 (2n)m n2 (n 1) > 0. Bt ng thc ny lun ng i vi m


ln hn nghim ln ca phng trnh 4x2 (2n)x n2 (n 1) iu ny
c ngha l G c 4-chu trnh nu nh m ln hn nghim ln ca phng

trnh bc 2 trn, hay m > 2n+ 4n +16n


8
thit cho ta iu phi chng minh.
2

3 16n2

m > n.

4n3+1
.
4

T gi

) = f (y)
.8.52.Cho hm f : Q {0, 1} tha mn f(x) = f(y) th f (x) = f ( x+y
2

vi x, y Q,
Nu f(0)=0 v f(1)=1 hy chng minh f(q) = 1 vi q Q

.c

Li gii: Trc tin ra chng minh b sau: Cho a, b l cc s hu


t. Nu f (a) 6= f (b) th f(n(b - a) + a) = f(b) vi mi n nguyn dng

(1)
Tht vy, ta s chng minh b trn bng quy np.

Vi n = 1 (1) hin nhin ng

Gi s mnh ng vi n k.
t (x1 , y1 , x2 , y2 ) = (b, k(b a) + a, a, (k + 1)(b a) + a). Theo

gi thit quy np, f (x1 ) = f (y1 ). Ta cn cn chng minh f (x2 ) 6=


f (y2 ). Tht vy gi swr f (x2 ) = f (y2) khi xt (x, y) = (x1 , y1 ) v
1
),
(x, y) = (x2 , y2 ) vo trong iu kin bi, ta c f (b) = f ( x1+y
2
2
) . Tuy nhin iu ny khng xy ra v x1 + y1 =
v f (a) = f ( x2 +y
2
x2 + y2 . Do f (y2 ) 6= f (x2 ) hay f (y2 ) 6= f (a).

Vy f (y2 ) = f (b). chng minh xong b .


p dng b trn vi a = 0 v b = 1 ta thy rng f(n)=1 vi

mi n nguyn dng. Tip theo ta thy f (1 + rs ) 6= 0 r, s N,


v nu ngc li, p dng b vi a = 1, b =1 + rs v n = s th
f(1+r) = 0 mu thun vi f(n) = 1 trn. Do vy, f(q) = 1 vi mi
s hu t q 1.

Ti liu tham kho


[1] Titu Andreescu, Zuming Feng, and George Lee, Jr. Mathematical
Olympiads 20002001, Problems and Solutions From Around the World,
The Mathematical Association of America, 2002.

[2] Nguyn Hu in, Phng php irichle v ng dng, NXBKHKT, 1999.

[3] Nguyn Hu in, Phng php Quy np ton hc, NXBGD, 2000.

.c

[4] Nguyn Hu in, Nhng phng php in hnh trong gii ton ph
thng, NXBGD, 2001.

[5] Nguyn Hu in, Nhng phng php gii bi ton cc tr trong hnh
hc, NXBKHKT, 2001.

[6] Nguyn Hu in, Sng to trong gii ton ph thng, NXBGD, 2002.

[7] Nguyn Hu in, a thc v ng dng, NXBGD, 2003.

NXBHQG, 2004.

[8] Nguyn Hu in, Gii phng trnh v nh nghim nguyn,

[9] Nguyn Hu in, Gii ton bng phng php i lng bt bin,
NXBGD, 2004.

Nguyn Hu in

OLYMPIC TON NM 2000

.c

(Tp 2)

49 THI V LI GII

NH XUT BN GIO DC

.c

Li ni u
th gi lnh lamdethi.sty ti bin son mt s ton thi Olympic,
m cc hc tr ca ti lm bi tp khi hc tp LATEX. ph v cc bn

ham hc ton ti thu thp v gom li thnh cc sch in t, cc bn c th


tham kho. Mi tp ti s gom khong 50 bi vi li gii. Tp ny c s ng
gp ca Trnh Quang Anh, Nguyn Th Bnh, Nguyn Th Thanh Bnh, o

.c

th Kim Cc, Nguyn Hong Cng, Gip Th Thy Dung, Mai Xun ng,
Hong H, Nguyn Th Thanh H.

Rt nhiu bi ton dch khng c chun, nhiu im khng hon ton


chnh xc vy mong bn c t ngm ngh v tm hiu ly. Nhng y l ngun

ti liu ting Vit v ch ny, ti c xem qua v ngi dch l chuyn v


ngnh Ton ph thng. Bn c th tham kho li trong [1].

Rt nhiu on v mi hc TeX nn cu trc v b tr cn xu, ti khng

c thi gian sa li, mong cc bn thng cm.

51
89/176-05
GD-05

H Ni, ngy 2 thng 1 nm 2010


Nguyn Hu in

M s: 8I092M5

Mc lc
3

Mc lc . . . . . . . . . . . . . . . . . . . . . . . . . . . . . . . . . . . . . . . . . . . . . . .

Chng 1. thi olympic Israel. . . . . . . . . . . . . . . . . . . . . . . . . . . .

Chng 2. thi olympic Italy . . . . . . . . . . . . . . . . . . . . . . . . . . . .

Chng 3. thi olympic Nht Bn . . . . . . . . . . . . . . . . . . . . . .

14

Chng 4. thi olympic Korea . . . . . . . . . . . . . . . . . . . . . . . . . .

18

.c

Li ni u . . . . . . . . . . . . . . . . . . . . . . . . . . . . . . . . . . . . . . . . . . . .

24

Chng 6. thi olympic Rumani . . . . . . . . . . . . . . . . . . . . . . . .

32

Chng 7. thi olympic Nc Nga . . . . . . . . . . . . . . . . . . . . . .

39

Chng 5. thi olympic Mng c . . . . . . . . . . . . . . . . . . . . . . .

Chng 8. thi olympic i Loan . . . . . . . . . . . . . . . . . . . . . . .

45

Chng 9. thi olympic Th Nh K . . . . . . . . . . . . . . . . . . . .

50

Chng 1

thi olympic Israel

.1.1. nh ngha f (n) = n!. Cho

.c

a = 0.f (1)f (2)f (3)....

Ni cch khc, thu c s biu din phn thp phn ca a vit cc


biu din thp phn ca f (1), f (2)., ... trong mt hng, a c phi l s

hu t khng?

Li gii: Nu a l s hu t th cc con s trong phn thp phn phi


xut hin mt cch tun hon. V f(n) lun bao gm mt s khc khng,

nn phn tun hon ca phn thp phn khng th ch bao gm ton


s khng. Tuy nhin, n ln, s cc s 0 cha trong f(n) tin ti v
cng, v vy phn tun hon ca phn thp phn phi cha ton s 0
mu thun. V vy a khng l s hu t.
.1.2. . ABC nh l nhng im nguyn. Hai trong ba cnh c di thuc

tp
17, 1999, 2000 . Tm gi tr ln nht c th ca din tch
ABC.
Li gii: Khng mt tng qut, gi s cnh AB, BC c di thuc

17, 1999, 2000 th

[ 1 2000 2000 sin = 1000.


SABC = 1 AB.BC sin BCA
2

Nguyn Hu in, HKHTN H Ni


ng thc c th xy ra, chng hn trong m nh l (0,0); (44,8) v

(-8, 44) chnh xc 2 cnh di 2000 v 442 + 82 = 2000 v gc gia 2


cnh l 2 . T , din tch ln nht ca l 1000.

.1.3. Bi ton 3.Cc im A, B, C, D, E, F nm trn 1 ng trn v cc


ng thng AD, BE, CF ng quy. Ly P, Q, R l cc trung im cnh
AD, BC, CF tng ng. 2 on (dy cung) AG, AH c v sao cho AG
// BE v AH//CF chng minh rng PQR v DGH ng dng.
Li gii: Cc gc nh hng mun . Gi s on thng AD, BE,
CF ng quy (ct nhau) ti X v O l tm ng trn cho bi.
\
\ = ORX
\ = , suy ra O, P, Q, R v X cng
Hin nhin OP
X = OQX
2

thuc 1 ng trn.
\ = DAH
\ = DXC
\ = CXP
\ = RXP
\ = P[
V vy DGH
QR Tng
\
[
t DGH = P RQ, t suy ra PQR DGH.

.c

.1.4. Mt hnh vung ABCD cho trc, mt php c tam gic ca hnh vung
l 1 s phn chia hnh v thnh cc tam gic sao cho bt k 2 tam gic

u c tch ri, ch chung 1 nh hoc chung nhau ch 1 cnh c th.


Khng nh no ca 1 tam gic c th nm phn trong ca cnh tam

gic khc). Mt php c tam gic tt ca 1 hnh vung l php c


trong mi tam gic u nhn.

a. Cho 1 v d v php c tam gic tt ca hnh vung.


b. Tm s nh nht ca cc tam gic cn c mt php c tam gic
tt?
Li gii: Ta a ra 1 v d v php c tam gic tt vi 8 tam gic.
t hng hnh vung sao cho on AB t nm ngang v A l nh
trn bn tri. Ly M v N l cc trung im cnh AB v CD tng ng,
\
v P l 1 im trung on MN khc trung im MN. Cc gc MP
A,
\
\
AP
D v DP
N v cc gc phn x ca chng qua MN - tt c u l
cc gc nhn. Ta chn Q, R trn ng thng nm ngang qua P sao cho
Q, P, R nm theo th t t tri qua phi v QP, PR c di rt nh
(khng ng k) chia hnh vung thnh cc bng cch v on QA,
QM, QN, QD, RB, RM, RN, RC v QR. Nu ta chn Q sao cho PQ
\ \
\ s gn bng s o gc MP
\
nh th s o cc gc MQA,
AQD, DQN
A,

thi olympic Israel

\
\
AP D, DP
N, v vy nhng tam gic ny s nhn.
\ BRC,
[ CRN
\ s
Tng t, nu chn R sao cho PR nh th MRB,
cng nhn. D kim tra rng cc gc trong s phn chia trn l nhn
nh yu cu.
b.Ta s chng minh s nh nht l 8. Ta ch ra rng 8 l gi tr c
th thc hin c. V vy, ch cn ch ra nhng php c tam gic tt
no vi t hn 8 tam gic. Nhn xt rng trong 1 php c tam gic tt,
mi nh ca ABCD l nh ca t nht 2 tam gic bi v gc vung
phi c chia thnh cc gc nhn. Nh vy, bt k nh no nm trn
cnh ABCD phi l nh ca t nht 3 tam gic v bt k nh nm
phn trong phi l nh ca t nht 5 tam gic.
Tm li, ta c th chng minh mt kt qu mnh hn v mi gc ca

hnh vung ABCD. Phi c mt tam gic m cnh bt u t nh hnh


vung v im cui nm trn phn trong hnh vung ABCD. Khng

.c

mt tng qut, gi s gc (nh) l A. Cnh AX no ca tam gic


chia gc vung ti A ra.Gi s phn chng rng X khng nm phn
trong hnh vung ABCD, khng mt tng qut, gi s X thuc on

BC (khng trng B). Bng nh ngha ca php c tam gic : khng c


nh khc ca mt tam gic trong php c tam gic nm trn on AX.

V vy, c 1 im Y trong ABX sao cho AXY l mt thnh phn C


\
\=
phn t ca php c tam gic tt. Nhng nu vy AY
X ABX
2

: mu thun.
Ta xt 1 php c tam gic tt bt k ca ABCD. Ly i l s ca cc

nh trong cc nh trong php c tam gic m nm bn trong hnh


vung ABCD. Theo trn i 1. Trc tin gi s rng c mt nh trong
P. Kt qu ca on trc cho ta: on PA, PB, PC, PD phi l cc
[
cnh ca cc tam gic trong php c tam gic. Mt trong gc AP
B,

[
\
\
\
BP C, CP D, DP A phi ln hn gi s l AP B. Gc ny phi c
2

chia ra trong php c tam gic ny bng cnh PQ no , vi Q l


[ v \
im thuc on AB. Nhng c AQP
BQP c s o t nht l 2 nn
Q phi nm trong cnh ca tam gic no m khng nm trong on
QA, QB hoc QP. Tuy nhin khng th to c mt cnh m khng
ct AP hoc BP v cnh khng kt thc mt nh trong th hai.

Nguyn Hu in, HKHTN H Ni


Gi s tip i 2. Vi mi mt n cc tam gic, ta c th m 3 cnh
c tng 3n; mi cnh nm trn bin hnh vung c m 1 ln, cc
cnh khc c m hai ln. Nu i = 2 th vi mi 2 im trong t nht
5 cnh tam gic nhn im lm im cui, nhiu nht 1 cnh tam
gic cha c hai nh trong, nn t nht 9 cnh tam gic khng nm
bin ca hnh vung. Nu i 3, ly bt k 3 nh trong. Mi nh thuc

t nht 5 cnh tam gic v nhiu nht 3 cnh tam gic cha 2 trong 3
nh . V vy t nht 3 x 5 3 = 12 cnh tam gic. Khng thuc bin
hnh vung. Trong c hai trng hp u c t nht 9 cnh tam gic
khng thuc bin hnh vung, v hn na li c 4 cnh tam gic thuc
bin hnh vung. V vy 3n9 x 2 + 4 = 22 hay n 8. V vy trong

.c

mi trng hp phi c t nht 8 tam gic tho mn yu cu.

Chng 2
thi olympic Italy
m

.2.5. Gi s ABCD l mt t gic li, vi = DAB; = ACB; =

.c

DBC; v  = DBA. Gi thit rng < /2, + = /2 v + 2 =


, chng minh rng

(DB + BC)2 = AD 2 + AC 2 .
D0

Li gii: Gi s D 0 l im i xng ca D qua ng thng AB. Ta


c D 0 BA = DBA = , nn D 0 BC = D 0 BA + ABD + DBC =
2 + = . Vy, D 0 , B, C l thng hng. Cng c AD 0 C + ACD0 =
ADB + ACB = + = /2, nn D 0 AC = /2 v tam gic A0 AC
vung. Theo nh l Pythagorean, D 0 C = AD 02 + AC 2 , ko theo
(DB + BC)2 = (D 0 B + BC)2 = D 0 C 2 = AD 02 + AC 2
= AD 2 + AC 2 ,

10

Nguyn Hu in, HKHTN H Ni


c iu phi chng minh.

.2.6. Cho s nguyn c nh n > 1, Alberto v Barbara chi tr chi sau, bt


u vi bc u tin v sau xen k gia ln th hai v ln th ba :
Alberto chn mt s nguyn dng.
Barbara chn mt s nguyn ln hn 1 l mt bi hoc c ca s

nguyn ca Alberto, c th chn ng l s nguyn ca Alberto.


Alberto cng hoc tr 1 t s ca Barbara.
Barbara chin thng nu c y chn ra n vi 50 ln chi. Vi gi tr no
ca n c y l ngi thng cuc.

Li gii: Mc ch ca chng ta l Barbara l ngi thng cuc nu v


ch nu t nht l mt iu kin sau c tha mn :

n = 2;
4| n ;

.c

c s nguyn m > 1, sao cho (m2 1)| n.


u tin chng ta ch ra rng khi v ch khi ba iu kin ny l ng,
th Barbara l ngi chin thng. Nu Barbara chn ln u tin a l

mt s chn th Barbara c th chn 2 trong ln u tin. Nu thay a


bng mt s l, th Barbara c th chn chnh l s a l tt nht. Nu

a = n, c y chin thng; ni cch khc, ln chn th hai ca Alberto


phi l s chn, v Barbara c th chn s 2 trong ln chn th hai. Gi

s a1 , b1 , a2 , b2 , . . . l cc s c chn sau khi Barbara chn 2 cho ln


chn u tin.

Trng hp 1 :
(a) n = 2, trong trng hp ny Barbara thc s chin thng.
(b) 4| n. Nu a1 = 1, th Barbara c th chn b1 = n v chin thng.
Ni cch khc, a1 = 3, Barbara c th chn b1 = 3, a2 bng 2 hoc
4, v Barbara c th chn b2 = n.
(c) C s nguyn m > 1, (m2 1)| n. Nh trng hp 2, Alberto phi
chn a1 = 3 ngn Barbara thng cuc. Thc t, c ng mt
s nguyn trong cc s m 1, m v m + 1 chia ht cho 3, ngha
l hoc 3 chia ht m hoc 3 chia ht m2 1v v v 3 chia ht n.
Trong trng hp u tin, Barbara c th chn b1 = m, bt buc
a2 = m 1 v ko theo Barbara chn b2 = n. trong trng hp

11

thi olympic Italy


sau, Barbara c th chn b1 = n.

By gi chng ta thy rng Barbara c mt chin thut chin thng nu


t nht mt trong cc iu kin l ng. By gi chng ta gi thit rng
khng iu kin no l ng vi n > 1 v chng minh rng Alberto c
th lun lun ngn cn Barbara tin n chin thng. Bi v iu kin
th nht v th hai l khng ng v bi v iu kin th hai l sai vi
m = 2, chng ta c n 6= 2, 3, 4,. Vy n > 4.

Gi mt s nguyn dng n l s hy vng nu a|n v n|a. Chng ta


chng minh rng vi s nguyn b > 1, tn ti a {b 1, b + 1} sao cho

a l s hy vng. u ny ko theo rng Alberto c th bt u chn


mt vi s hy vng v cng chn mt vi s hy vng theo sau ngn

cn Barbara tin n chin thng sau 50 ln.


Gi s v iu kin l mc ch trn l sai vi s nguyn b > 1. Nu
b > n, th b 1 v b + 1 phi l bi ca n. Do n chia ht hiu, tc l

.c

2, mu thun.
Ni cch khc, b n. Bi v n khng chia ht n + 1 hoc n + 2 vi n > 2,

chng ta phi c (b 1)|n v (b + 1)|n. Nu b 1 v b + 1 l chn, th


mt trong chng phi chia ht cho 4 - nhng 4| n, mu thun. Vy, b 1

v b + 1 l l. iu ny ko theo chng nguyn t cng nhau v tch ca


chng b2 1 chia ht n, mu thun vi gi thit iu kin th ba l sai.

.2.7. Gi s p(x) l mt a thc vi h s nguyn sao cho p(0) = 0 v 0


p(1) 107 , v sao cho tn ti cc s nguyn a, b tha mn p(a) = 1999
v p(b) = 2001. Xc nh cc gi tr c th ca p(1).

Li gii: Nu p(x) = 2000x2 x, th p(0) = 0, p(1) = 1999, v p(1) =


2001. Nu p(x) = 2000x2 +x, th p(0) = 0, p(1) = 2001, v p(1) = 1999.
Do , c th p(1) = 1999 hoc 2001.
By gi gi s rng p(1) 6= 1999, 2001. Th a, b 6= 1. Bi v p(0) = 0,

chng ta vit c p(x) = xq(x) vi a thc q(x) c h s nguyn. Bi


v q c h s nguyn, q(a) l mt s nguyn, v c th vit q(x) q(a) =
(xa)r(x) vi a thc r(x) c h s nguyn. V bi v r c h s nguyn,
r(b) l mt s nguyn, v chng ta c th vit r(x) r(b) = (x b)s(x)

12

Nguyn Hu in, HKHTN H Ni


vi a thc s c h s nguyn. Do :
(*)

p(x) = xq(x) = xq(a) + x(x a)r(x)


= xq(a) + x(x a)r(b) + x(x a)(x b)s(x).
c bit, khi cho x = a v x = b, chng ta tm c
1999 = aq(a)
2001 = bq(a) + b(b a)r(b).

Bi v p(0), p(a) v p(b) l cc s phn bit, v vy 0, a v b cng phn


bit. Do , chng ta c th gii hai phng trnh trn tm c
1999
a
2001 bq(a)
.
r(b) =
b(b a)

(*)

.c

q(a) =

Bi v a 6= b, chng ta c |a b| chi ht p(a) p(b). V vy |a b| bng

|a| {1, 1999}.

1 hoc 2. Cng vy, vi mi x Z, chng ta c p(x) = xq(x) v v vy


x| p(x). c bit, a| 1999, cho nn

Vi hn ch ny, kt hp vi iu kin |a b| {1, 2}, b|2001, a 6= 1, v


b 6= 1, ko theo rng (a, b) bng mt trong cc cp sau :
(1999, 2001), (1, 3), (1999, 2001).

C nh (a, b) l chung cho ba cp trn. T (*) chng ta bit rng q(a)


2001 b
q
1999
v r(b) phi bng r =
. Cho x = 1 v (*)
phi bng q =
a
b(b a)
tm p(1) :
(a, b)

q(a)

r(a)

p(1)

(1999, 2001)

1 + (2000.2002)s(1)

(1, 3)
(1999, 2001)

1999 666
1

3331 + 8s(1)
1 + (1998.2000)s(1).

13

thi olympic Italy

V vy, p(1) c dng m + ns(1) vi s nguyn c nh m, n. Tht vy,


gi s rng c mt s dng m + n
s gia 0 v 107 , t s l s nguyn.
Chng ta c
p(x) = qx + rx(x a) + sx(x a)(x b),
chng ta c p(0) = 0, p(a) = 1999, p(b) = 2001, v p(1) = m + n
s.
Do , cc gi tr c th ca p(1) l 1999 v 2001, v cc s gia 0

.c

v 107 ng d vi 1 (mod 2000.2002), 3331 5 (mod 8), hoc


1 (mod 1998.2000).

Chng 3
thi olympic Nht Bn
m

.3.8. Ta tro mt lot cc l bi nh s a1 , a2 , . . . , a3 n t tri qua phi bng

vic sp xp cc l bi theo th t mi:

.c

a3 , a6 , . . . , a3 n, a2 , a5 , . . . a3n1 , a1 , a4 , . . . , a3n2

V d nu 6 l bi c nh s 1, 2, . . . , 6 t tri qua phi th vic tro


chng 2 ln s thay i trt t ca chng nh sau:

1, 2, 3, 4, 5, 6 3, 6, 2, 5, 1, 4 2, 4, 6, 1, 3, 5

Bt u vi 192 qun bi dnh s 1, 2, . . . , 192 t tri qua phi, liu ta

c c trt t 192, 191, . . . , 1 sau s ln tro hu hn?


Li gii: Vi mi n, cho f (n) l v tr trong chui cc qun bi
qun bi i v tr th n sau mi ln tro. Ta thy rng sau k ln tro,
f k (n) v tr th n. Ta c bit rng f (1), . . . , f (192) bng 3, 6,
. . . , 192, 2, 5, . . . , 191, 1, 4, . . . , 190. Trong trt t ny, s khc bit
gia bt k s hng no vi s hng ng trc n l ng d t 3 ti
modul 193. V f (1) 3 (mod 193) ta c f (n) 3n (mod 193) vi mi
n.
Trong trt t (33 )20 , (33 )21 , . . . , (33 )26 , vi mi s hng l bnh phng
ca s hng trc n. t nht mt s hng trc n (s hng u tin 2t)
khng ng d vi mt modul 193; gi s N = 3d ( d l s nguyn

15

thi olympic Nht Bn

dng) l gi tr ln nht vi thuc tnh ca n, v 193 l s nguyn


t, theo nh l Fermat c: (33 )26 (33 )192 1 (mod 193), do vy 3d

khng phi l s hng cui cng trong trt t ny. Do vy, N 2 s hng
tip theo ca N trong trt t l ng d vi 1 modul 193. V 193 chia
c cho N 2 1 nhng khng chia c cho N - 1, n phi chia ht cho
(N 2 1)(N 1) = N + 1 = 3d + 1, c ngha l 3d 1 (mod 193) .

Vin = 1, 2, . . . , 193 ta c f d (n) 3d n n (mod 193). Do vy


f d (n) = 193 n, c ngha l trt t 192, 191, . . . , 1 xut hin sau d
ln tro.
Ch : Gi tr d tm thy trn thc t l bng 24. S nguyn dng
k nh nht tha mn 3k 1 (mod 193) l 8, c ngha l trt t
192, 191, . . . , 1 xut hin ln u tin sau 8 ln tro bi.

.3.9. Trong mt phng cho cc im phn bit A, B, C, P, Q, khng c 3

im no thng hng. Chng minh rng:

.c

AB + BC + CA + P Q < AP + AQ + BP + BQ + CP + CQ.

Li gii: Trong bi gii ny, khi gi mt a gic V1 . . . Vn l li nu

V1 , . . . , Vn to thnh mt a gic li trong trt t . (V d nu ta ni


hnh vung ABCD l li th ta khng ni rng ACBD l li.)
Ta ni rng iu kin (a) c nh nu t gic XYPQ l li vi X, Y

{A, B, C}. Trong trng hp ny ta chng minh bt ng thc cn


chng minh l c nh. Khng mt tnh tng qut, ta c th gi thit

rng t gic ABPQ l li. Nu AP giao vi BQ ti O, th bt ng thc


tam gic cho ta AB AO + BO v P Q P O + QO. Cng 2 bt ng

thc ny ta c:

AB AO + BO + OP + OQ = AP + BQ
V khng c 3 trong s cc im cho no thng hng nn bt ng
thc tam gic cng ch ra rng BC < BP + P C v CA < CQ + QA
Cng 3 bt ng thc cui cng ny ta c kt qu cn chng minh.
Tip n ta ni ti iu kin (b) c nh, nu xem X nm trong tam gic
YZM vi s hon v (X, Y, Z) ca (A, B, C) v vi M {P, Q}. Ta chng

16

Nguyn Hu in, HKHTN H Ni


minh rng bt ng thc cn chng minh c nh trong trng hp ny.
Khng mt tnh tng qut, gi s A nm trong tam gic BCQ. S
chuyn im P ty ti mi cnh PB, PC l cc hm li ngt, c ngha l
P P B + P C cng l mt hm li ngt. Do vy, trn tt c cc im

P hoc trong tam gic BCQ hm ny ch t cc i khi P trng vi B,


C hoc Q. Vy nn: AB +AC < max{BB +BC, CB +CC, QB +QC} =
QB + QC cng bt ng thc ny vi bt ng thc BC < BP + PC
v PQ < PA + QA, c bt ng thc tam gic ta c kt qu
cn chng minh.
Do vic i tn cc im, bao li ca 5 im cho hoc phi l tam
gicBC, hoc ABP, hoc APQ, hoc t gic li ABCD, hoc ABPQ,

hoc APBQ, hoc ng gic li ABCPQ hoc ABPCQ.


Nu tam gic ABC l bao li th Q phi nm pha trong mt trong cc

tam gic APB, BPC, CPA. Khng mt i tnh khi qut gi thit rng
Q nm trong tam gic APB. V C khng nm bn trong tam gic APB

.c

nhng nm cng pha ng AB so vi Q, do vy QC phi giao vi mt


trong 2 on thng AP v P B. Nu QC giao vi AP , th t gic ACPQ
l li v iu kin (a) c nh; tng t iu kin (a) c nh nu QC

giao vi P B.
Nu tam gic ABP l bao li th C phi nm trong tam gic ABP v

iu kin (b) c nh.


Nu tam gic APQ l bao li th ta c th gi thit C khng gn hn

PQ so vi B m khng mt di tnh tng qut. Vy nn iu kin (b) c


nh.
Nu t gic ABCP l bao li th Q nm trong tam gic APB hoc trong
CPB. Trong trng hp u t gic BCPQ l li v trong trng hp
th hai t gic BAPQ l li. Vy nn iu kin (a) c nh.
Nu t gic li ABPQ, ng gic li ABCPQ hay ng gic li ABPCQ
l bao li th t gic ABPQ l li v iu kin (a) c nh.
Cui cng, nu t gic li APBQ l bao li th C hoc nm trong tam
gic ABP hoc ABQ; c trong 2 trng hp iu kin (b) c nh.
Do vy, trong tt c cc trng hp, hoc iu kin (a) hoc iu kin
(b) c nh, t suy ra bt ng thc cn chng minh l ng.

17

thi olympic Nht Bn

.3.10.Cho 1 s t nhin n 3, chng minh rng tn ti 1 tp hp An vi 2


thuc tnh sau:
(i) An bao gm n s t nhin ring bit.
(j) Vi bt k a An , tch s ca tt c cc phn t khc trong An c

s d l 1 khi c chia bi a.

Li gii:
Gi s a1 , a2 , . . . , ak (vik 2) l cc s nguyn ring bit ln hn 1 tha

mn a1 a2 . . . ak (mod ai ) khi 1 i k. Gi s  {1, 1} v xc


nh aa+1 = a1 a2 . . . ak . V ak+1 2ak 1 > ak vi tt c cc k, cc

s nguyn a1 , a2 , . . . , ak+1 vn l cc s nguyn ring bit ln hn 1.


Xem xt biu thc
(mod ai )

a1 a2 . . . ai1 ai+1 . . . ak+1 

r rng n khng i vi i = k + 1. Vi i < k n khng i v

.c

(a1 a2 . . . ai1 ai+1 . . . ak )ak+1 (1)() 

(mod ai )

Bt u vi cc s a1 = 2, a2 = 3, ta p dng cch ny n-3 ln tp hp

 = 1 v mt ln tp hp  = 1. Tp hp An bao gm cc s kt qu
l a1 , a2 , . . . , an do tha mn iu kin u bi.

Chng 4

thi olympic Korea

.c

.4.11.Ch ra rng vi mi s nguyn t cho trc p th tn ti nhng s t


nhin x, y, z, tho mn x2 + y 2 + z 2 .p = 0 v 0 < < p.

Li gii: Vi trng hp p = 2, ta c th ly x = 0, y = z = = 1.
By gi ta xt trng hp p > 2. Trc tin ta xt trng hp 1 l ng

d bnh phng modun p, khi tn ti mt s t nhin a, 0 < a < p1


sao cho a2 1(modp). B (x, y, z) = (0, 1, a). V x2 + y 2 + z 2 = a2 + 1

chia ht cho p nhng 1 + (p 1)2 < p2 nn tn ti {1, 2, ..., p 1}


sao cho x2 + y 2 + z 2 .p = 0 .

Tip theo, gi s (1) khng l ng d bnh phng modunp. Ta phi


tm mt s k no c k v p k 1 u l ng d bnh phng.
l ng d bnh phng th chn k = p1
. Nu ngc li, th
Nu p1
2
2

cc ng d bnh phng khc khng s ri


mi ng d trong s p1
2
  p+1
, 2 . Theo nguyn
vo trong cc cp {1, p 2} , {2, p 3} , ..., p3
2

l Pigeonhole Principle s c mt cp (k, p k 1) m c hai s k v


(p k 1) u l ng d bnh phng nh ta nh tm.


V vy, ta c th chn x, y 0, 1, ..., p1
sao cho x2 k(modp) v
2
y 2 p k 1(modp). Cho z = 1, ta c x2 + y 2 + z 2 chia ht cho p v

x2 + y 2 + z 2 < p2 . Gi tr s c xc nh nh trng hp trc.

19

thi olympic Korea


.4.12.Tm tt c cc hm f : R R tho mn
f (x2 y 2) = (x y) [f (x) + f (y)]
vi mi x, y R.

Li gii: Cho x = y, ta c f (0) = 0.


Cho x = 1, y = 0 ta c f (1) = f (1).

Cho x = a, y = 1, sau cho x = a, y = 1 ta c:


f (a2 1) = (a 1) [f (a) + f (1)]
f (a2 1) = (a + 1) [f (a) f (1)]

Cho cc v phi ca cc phng trnh bng nhau v gii phng trnh


i vi f (a) ta c f (a) = f (1).a vi mi a.

Nh vy, mi hm s no tho mn rng buc cho phi c dng


f (x) = kx vi hng s k no . Ngc li, bt k hm s no c dng

f (x) = kx vi hng s k no r rng u tho mn yu cu bi ton.

.c

.4.13.Cho t gic li ABCD l t gic ni tip. Gi P, Q, R, S ln lt l cc


\ v ADB,
\ DAB
\
giao im ca hai ng phn gic ngoi cc gc ABD
\ ACD
\ v ADC,
\ DAC
\ v DCA
\ tng ng. Chng minh rng
v DBA,

bn im P, Q, R, S cng nm trn mt ng trn.

D
C

20

Nguyn Hu in, HKHTN H Ni


Li gii: Cc gc xt n u l cc gc nh hng ngoi tr cc
trng hp ni khc i.
Gi s chng ta c mt tam gic tu XY Z vi tm ng trn ni
tip l im I v tm ng trn bng tip IX i din vi gc X. Suy
\
\
ra X, I, IX thng hng. Ta c IY
IX = 2 = IZI
X v vy t gic IY IX Z l
\
\ [
\ [
ni tip c v XI
X Y =IIX Y =IZY hay Y IX X=Y ZI.
X

IX

.c

Gi I1 , I2 ln lt l tm ng trn ni tip cc tam gic ABD v tam


gic ACD.
T gi thit ta suy ra P , Q l cc tm ng trn bng tip ca tam

gic ABD i din vi gc A v gc D, tng t R, S l cc tm ng


trn bng tip ca tam gic ACD i din vi gc A v gc D.

p dng kt qu ca phn trn vi (X, Y, Z, IX ) l (A, D, B, P ),


\
(D, A, B, Q), (A, D, C, R) v (D, A, C, S) ta c, AP
D=I\
1 BD,
\
\1 , ARD=
[ I\
[ \
AQD=ABI
2 CD, v ASD=ACI2 .
\ \
Khi coi cc gc sau l khng nh hng, ta thy I\
1 BD, ABI1 , I2 CD
v \
ACI2 u bng

\
AQD
2

\
ACD
.
2

Hn na, cc gc trn u cng mt hng, nn nu coi chng l nhng


gc nh hng, chng s bng nhau. Nh vy (tr li vi nhng gc nh
\
\ = ARD
[ = ASD
[ v bn im P, Q, R, S
hng) ta c: AP
D = AQD
cng nm trn cung trn trng bi A, D.
.4.14.Cho p l mt s nguyn t sao cho p 1 (mod4). Hy tnh

p1 
X
2k 2
k=1

k2
2
p



21

thi olympic Korea

Li gii: Vi mi s thc x, t
{x} = x [x] [0, 1).
Ta c

1
2
1
2


 2
 2  2
k
k
2k
2k 2
2
=2

p
p
p
p


 2
k2
k
k2
=

p
p
p

th 2 {x} {2x} = {x} 2 {x} = 0

Ta c

Nu {x} <


 2
2k 2
2k 2
2k
=

p
p
p

.c

Nu {x} th 2 {x} {2x} = 2{x} (2 {x} 1) = 1


Nh vy, tng cnntnh
o trong bi ra s bng l s cc phn t k trong
2
[1, p 1] sao cho kp 12 , hay bng vi s ng d k khc khng m


k 2 l ng d m un p vi mt s no trong p+1
,
p

1
.
2

V p l s nguyn t ng d vi 1 m un p, ta bit 1 d2 (modp),


vi d l mt s no . Phn chia cc ng d m un p khc khng

cp dng {a, da} sao cho a2 (da)2 (modp).


thnh p1
2
V vy, c ng mt ng d trong mi cp m bnh phng ca n


, p 1 , v c tt c p1
ng d
ng d vi mt s no trong p1
2
2
nh th.
T suy ra tng cho bng p1
.
2
.4.15.Xt nhng hnh L sau y, mi hnh c to bi bn hnh vung n
v ghp li.

22

Nguyn Hu in, HKHTN H Ni


Cho m v n l cc s t nhin ln hn 1. Chng minh rng mt hnh
ch nht kch thc mxn s c xp bi cc hnh cho khi v ch khi
m.n l bi s ca 8.
Li gii: Trc tin ta chng minh rng nu 8\mn, th hnh ch nht
mxn c th c xp bi cc hnh cho.
Trng hp 1: C m v n u l s chn. Khng mt tnh tng qut ta
gi s rng 4\m, 2\n. Hai hnh cho c th ghp c mt hnh ch
nht kch thc 4x2, v m.n/8 hnh ch nht nh vy s ghp thnh
mt hnh ch nht kch thc mxn (gm n/2 hng v m/4 ct).
Trng hp 2: Hoc m hoc n l. Khng mt tnh tng qut, ta gi s

.c

rng m l s l. Khi 8\n. V m > 1 nn m 3. Ta c th ghp c


mt hnh kch thc 3x8 nh hnh v sau:

Nhng hnh 3x8 nh vy c th ghp thnh hnh ch nht (3xn).

Nu m = 3, ta ghp xong. Trong trg hp ngc li, m > 3, th


phn cn li (m3)xn c th ghp nh trong trng hp 1 v 2\(m3).

By gi ta s chng minh rng: nu hnh ch nht c kch thc (mxn)


c ghp bi cc hnh trn th 8\m.n. V mi mt hnh L c din tch

l 4 nn 4\(m.n). Khng mt tnh tng qut, gi s 2\n, v t m hng


trong hnh ch nht mxn thnh hai mu en trng cnh nhau. Mi
mnh hnh ch L trong hnh ch nht c ghp s gm mt s l en
 
hnh vung. V c tt c 1 s chn (nx m2 ) vung mu en, nn hnh
ch nht c ghp cha 1 s chn cc hnh ch L, m ta t s l
2k. Nh vy m.n = 8k, hay 8\mn.
.4.16.Cho nhng s thc a, b, c, x, y, z tho mn a b c > 0 v x y
z > 0.
Chng minh rng:
b2 y 2
c2 z 2
3
a2 x2
+
+

(by + cz)(bz + cy) (cz + ax)(cx + az) (ax + by)(ay + bx)


4

23

thi olympic Korea

Li gii: t v tri ca bt ng thc l S. V a b c > 0 v


x y z > 0 nn ta c bz + cy by + cz suy ra



(by + cz)(bz + cy) (by + cz)2 2 (by)2 + (cz)2

t = (ax)2 ; = (by)2 ; = (cz)2 , khi ta c:

a2 x2
a2 x2

=
2
2
(by + cz)(bz + cy)
2 [(by) + (cz) ]
+
p dng tng t cho hai bt ng thc, ta c

S (
+
+
)
2 + + +

+
+
)(( + ) + ( + ) + ( + ))( + + )2
+ + +

.c

S dng bt ng thc Cauchy-Schwarz, ta c

m v phi bng

 3
1
( )2 + ( )2 + ( )2 + 3( + + ) (2+2+2)
2
2

Do ,

1
3
( + + )2
1
+
+
)

S (
2 + + +
2 (2 + 2 + 2)
4
Vy bi ton c chng minh.

Chng 5

thi olympic Mng c

.5.17.t rad (1) = 1, vi k > 1, t rad (k) l tch cc s nguyn t ca k.


Mt dy cc s t nhin a1 , a2 , ... vi s hng u a1 c xc nh bi

.c

mi quan h: an+1 = an + rad (an ). Hy ch ra vi mi nguyn dng


N, dy an+1 = an + rad (an ) gm N s hng lin tip trong mt cp s

cng.

Li gii: *) B 1: Trong dy rad (a1 ) , rad (a2 ) , ... mi s hng l


c ca s hng tip sau n.

Chng minh:
V rad (an ) l c ca c an v rad (an )
nn rad (an ) l c ca an + rad (an ) = an+1
mi tha s nguyn t ca rad (an ) l c ca an+1
V rad (an ) v rad (an+1 ) l tch ca cc c s nguyn t
T cho ta kt qu rad (an ) l c ca rad (an+1 )
an
*) Vi mi s nguyn dng n t bn = rad(a
v zn =
n)

rad(an+1 )
rad(an )

V rad (an ) l c ca rad (an+1 ) nn bn l mt s nguyn dng n


Do b 1, ta cng c kt qu tng t vi zn
Mt khc zn v rad (an ) l nguyn t cng nhau do rad (an+1 ) l tch
ca cc c s nguyn t.

25

thi olympic Mng c


V vy ta c:
bn+1

an +rad(an )
rad(an )
rad(an+1 )
rad(an )

an+1
=
=
rad (an+1 )

bn+1 + 1
zn

*) B 2: Vi mi N, tn ti mt s nguyn dng M tha mn:


zM = zM +1 = ... = zM +N 2 = 1
Chng minh:
C vi s nguyn t p nh hn 2N tha mn iu kin tn ti mt s n
sao cho p l c ca an
p dng b 1, tn ti mt s nguyn m ln sao cho am chia ht

cho mi s nguyn t.
Gi M l s ln hn m sao cho bM l nh nht. Ta cn chng minh M

.c

tha mn iu kin ca b ny.


Tht vy, gi s kt qu trn l khng ng, khi ta cn ch ra s k
dng nh nht tha mn zM +k1 6= 1

Mt khc k N 1 v zM = zM +1 = ... = zM +N 2 = 1
do bM +k1 = bM + k 1

Ta cn ch ra khng c s nguyn t no nh hn 2N c th chia ht


cho zM +k1 . iu ny l ng v zM +k1 l tch cc s nguyn t chia

ht cho aM +k nhng khng chia ht cho aM +k1 v do aM +k1 chia ht


cho rad (aM ) v rad (aM ) chia ht cho mi s nguyn t nh hn 2N.

Do chia ht cho mi an . T suy ra zM +k1 2N.


V vy:
bM +k =

bM + k
bM + k
bM + N 1
bM +k1 + 1
=

< bM
zM +k1
zM +k1
2N
2N

iu ny mu thun vi gi thit M l s t nhin ln hn m v bM l


s t nhin nh nht. Vy b 2 c chng minh.
*) p dng b 2, vi mi N, tn ti mt s t nhin M tha mn:
rad (aM ) = rad (aM +1 ) = ... = rad (aM +N 1 )
Vy aM , aM +1 , ..., aM +N 1 l cc s hng lin tip trong mt cp s
cng.

26

Nguyn Hu in, HKHTN H Ni

.5.18.Trong mt phng, cho ba ng trn 1 , 2 , 3 i mt tip xc ngoi


nhau. Gi P1 l tip im ca 1 , 3 , P2 l tip im ca 2 , 3 . A, B
l hai im trn ng trn 3 khc P1 , P2 sao cho AB l ng knh
ca ng trn 3 . ng thng AP1 ct li ng trn 1 ti X, ng
thng BP2 ct li ng trn 2 ti Y . Cc ng thng AP2 , BP1 ct
nhau ti Z. Chng minh rng X, Y, Z thng hng.
Li gii: Xt cc gc l c hng theo modulo
Gi P3 l tip im ca hai ng trn 1 , 2 v O1 , O2 , O3 l tm ca
ba ng trn 1 , 2 , 3 tng ng.
Gi 4 l ng trn ngoi tip tam gic P1 P2 P3 v O4 l tm ng
phng ca ba ng trn 1 , 2 , 3 th ta c: O4 P1 = O4 P2 = O4 P3 .

Do O4 l tm ca ng trn 4 .
V O4 O1 O1 O3 nn O1 O3 l tip tuyn ca 4 .

Chng minh tng t ta cng c O1 O2 , O2 O3 l tip tuyn ca 4 .


\
\
V O3 AB nn ta c: P\
2 P1 Z = P2 AO3 = O3 P2 A.

.c

Nu gi Z 0 l giao im th hai ca AP2 v 4 th do O3 P2 l tip tuyn


ca 4 nn ta c:
\0
\0
O\
3 P2 A = O 3 P2 Z = P2 P1 Z

0
\0
Do P\
2 P1 Z = P2 P1 Z v Z thuc ng thng BZ

V Z v Z 0 cng thuc AP2 , AP2 6= BZ nn suy ra Z Z 0 . Do


Z 4 .
\
V O\
4 P1 O3 v XP1 Z cng vung (do AB l ng knh) nn ta c:
\
\
\
\
\
\
ZP
1 O3 = ZP1 O4 + O4 P1 O3 = XP1 Z + ZP1 O4 = XP1 O4
\
\
(1) V P1 O4 l tip tuyn ca 1 nn ta c: XP
1 O4 = XP3 P1 (2)
\
\
T (1) v (2) suy ra ZP
1 O3 = XP3 P1
Gi l l ng thng ZP3 nu Z P3 hoc ng thng tip xc vi
\
4 ti P . Khi : (l,\
P3 P1 ) = ZP
1 O3 (v O3 P1 l tip tuyn ca 4 .
\
Kt hp iu ny vi kt qu trn, ta suy ra: (l,\
P3 P1 ) = XP
3 P1
Xl
Chng minh tng t ta c Y l .
V Z l nn ta suy ra ba im X, Y, Z thng hng.

27

thi olympic Mng c


.5.19.Mt hm s f : R R tha mn cc iu kin sau:
1) |f (a) f (b)| 6 |a b| a, b R
2) f (f (f (0))) = 0
Chng minh rng f (0) = 0.

Li gii: Ta s dng nhn xt sau: f k (x) = f (f (...f (x) ...))


| {z }
k ln f

T (1) ta c:





|f (0)| = |f (0) 0| > f 2 (0) f (0) > f 3 (0) f 2 (0) = f 2 (0)

v |f 2 (0)| = |f 2 (0) 0| > |f 3 (0) f (0)| = |f (0)|


ta suy ra |f (0)| = |f 2 (0)|

*) Trng hp 1: f (0) = f 2 (0)


Khi : f (0) = f 2 (0) = f 3 (0) = 0

.c

*) Trng hp 2: f (0) = f 2 (0)


Ta c: |f (0)| = |f (0) 0| > |f 2 (0) f (0)| = 2 |f (0)|
|f (0)| 6 0 f (0) = 0

Vy f (0) = 0.

.5.20.ng phn gic ca cc gc A, B, C ca tam gic ABC ct cc cnh

ca tam gic ti cc im A1 , B1 , C1 . Chng minh rng:

BC
AC
AB
=
=
AC + AB
BA + BC
CA + CB
Li gii: Gi s ng trn ngoi tip t gic BA1 B1 C1 ct li
ng thng AC ti X. Ta cn chng minh X phi thuc on AC.
Trc ht, do A nm trn ng BC1 nhng khng thuc on BC1
nn A phi nm ngoi ng trn .
Tng t, C nm ngoi ng trn .
Ta c mi im nm trong on B1 X u nm trong ng trn , do
B1 X khng cha A cng khng cha C.
V B1 nm trn cnh AC do X nm trn cnh AC.
t BC = a; CA = b; AB = c. p dng phng tch ca im A i
vi ng trn ta c: AB.AC1 = AX.AB1

28

Nguyn Hu in, HKHTN H Ni


T nh l v ng phn gic trong tam gic ta c:
AC1 =

bc
bc
; AB1 =
a+b
a+c

Do ta c:
AX =

AC1 .AB
bc
a+c
(a + c) c
=
.c.
=
AB1
a+b
bc
a+b

Chng minh tng t ta cng c: CX =


Hn na, do X thuc cnh AC nn:

(a+c)a
b+c

c
a
b = AC = AX + CX = (a + c)
+
a+b b+c

hay

c
a
a
b
c
b
=
+

a+c
a+b b+c
b+c
a+c a+b

.c

BC
AC
AB
=
=
AC + AB
BA + BC
CA + CB

.5.21.Nhng s nguyn no c th biu din c di dng


(x + y + z)2
xyz

vi x, y, z l cc s nguyn dng.

Li gii: *) Nhn xt: Ta c cc s sau tha mn yu cu bi ton:

(9 + 9 + 9)2
(4 + 4 + 8)2
(3 + 3 + 3)2
1=
; 2=
; 3=
9.9.9
4.4.8
3.3.3
4=

(2 + 2 + 4)2
(1 + 4 + 5)2
(1 + 2 + 3)2
; 5=
; 6=
2.2.4
1.4.5
1.2.3
(1 + 1 + 2)2
(1 + 1 + 1)2
8=
; 9=
1.1.2
1.1.1

Ta s chng minh khng c cc kt qu khc tha mn b sau:


B : Nu n c th biu din c di dng
(x + y + z)2
xyz

29

thi olympic Mng c


th n c th vit di dng:
(x0 + y 0 + z 0 )2
x0 y 0 z 0
vi x0 6 y 0 + z 0 ; y 0 6 x0 + z 0 ; z 0 6 x0 + y 0
Chng minh b :
Gi x, y, z l cc s nguyn dng tha mn n =
l nh nht.

(x+y+z)2
xyz

v x + y + z

V n l mt s nguyn nn x l c ca (x + y + z)2 do x l c ca
(y + z)2
t x0 =

(y+z)2
,
x

khi ta c

2
(y + z)2 y+z
+1
x
(x0 + y + z)2
x
=
=
2
(y+z)
x0 yz
yz

y+z
x

+1
yz

(x + y + z)2
=
=n
xyz

2

.c

Do x+y+z l nh nht nn x+y+z 6 x0 +y+z Suy ra x 6 x0 = (y+z)

x
x 6 y + z Chng minh tng t th ta cng c: y 6 x + z ; z 6 x + y
(B c chng minh)
2

Ta xt cc trng hp sau:
*) Trng hp 1: x = y > z = 1

. p dng b , khng mt tnh tng qut, ta c


Gi s n = (x+y+z)
xyz
th gi s y + z > x > y > z

Khi n = (2x+1)
x l c ca 2x + 1 x = 1 v n = 9
x2
*) Trng hp 2: x = y + 1 > z = 1
2

(2x)
4x
= x1
x 1 l c ca 4x x 1 l c ca 4
Khi n = x(x1)
x {2; 3; 5} n {8; 6; 5}
*) Trng hp 3: y + z > x > y > z > 1

Khi : yz (y + z) = (y 1) (z 1) 1 > 0
yz > y + z > x

Do x > y > z ta c: xy > z ; xz > y


Do


(x + y + z)2
x
y
z
3
1 1 1
n=
+ + +
=2
+ +
6 2. +1+1+1 = 6
xyz
x y z
yz xz xy
2
n {1; 2; 3; 4; 5; 6}

Vy cc kt qu ca n cn tm l n {1; 2; 3; 4; 5; 6; 8; 9}

30

Nguyn Hu in, HKHTN H Ni

.5.22.Mt t nc c n thnh ph. Tng chi ph ca chuyn i t thnh ph i


n thnh ph j l xij . Gi s rng tng chi ph ca tuyn ng qua mi
thnh ph ng mt ln v kt thc ti im bt u khng ph thuc
vo vic chn tuyn ng. Chng minh rng tn ti cc s a1 , a2 , ..., an
v b1 , b2 , ..., bn sao cho xij = ai + bj vi mi s nguyn dng i, j tha
mn 1 6 i < j 6 n.
Li gii: t f (a, b) = xa1 + x1b xab vi a, b v 1 l ba s phn bit
*) B : f (a, b) khng ph thuc vo a, b
Chng minh b :
+) Vi n 6 2, iu ny l tm thng v khi f c xc nh khng
c a v b

+) Vi n = 3 ta cn ch ra f (2, 3) = f (3, 2)
hay x21 + x13 + x32 = x31 + x12 + x23

.c

Nhng nhng ng thc ny l tng cc chi ph ca 2 tuyn ng m


mi tuyn u i qua mi thnh ph ng 1 ln, do chng bng nhau.
+) Vi n > 4, tuyn ng:

a, 1, b, c, 2, 3, ..., a 1, a + 1, ..., b 1, b + 1, ..., c 1, c + 1, ..., n

v tuyn ng

a, b, 1, c, 2, 3, ..., a 1, a + 1, ..., b 1, b + 1, ..., c 1, c + 1, ..., n

phi c tng ton b chi ph bng nhau. Cc tuyn ng ny gn ng


nht, cho php ta dng tm c s khc nhau ca tng chi ph ca
2 tuyn ng l:
(xa1 + x1b + xbc ) (xab + xb1 + x1c )
Do , f (a, b) = f (b, c) vi mi a, b, c i mt khc nhau v khc 1
Hn na, tng ca ton b chi ph ca 3 tuyn ng:
1, a, b, 2, ..., n ; b, 1, a, 2, ..., n ; a, 1, b, 2, ..., n
phi bng tng ca ton b chi ph ca 3 tuyn ng:
1, b, a, 2, ..., n ; a, 1, b, 2, ..., n ; b, 1, a, 2, ..., n

31

thi olympic Mng c


Do 2 (x1a + xab + xb1 ) = 2 (x1b + xba + xa1 )
T suy ra f (a, b) = f (b, a)
Vi c, d khng bng a, b ta c: f (a, b) = f (b, c) = f (c, d) ;
f (a, b) = f (b, c) = f (c, b) ;
f (a, b) = f (b, a) = f (a, c) = f (c, a)
iu ny chng minh c kt qu ca b .

*) Vi mi a, b phn bit v khc 1 ta c: f (a, b) = F vi F l hng s


Cho a1 = 0 ; b1 = F v t bk = x1k ; ak = xk1 F
Vi mi i, j khng ng thi bng 1 ta c:
xij = xi1 xi1 x1j + xij + x1j = xi1 F + xij = ai + bj

.c

tc l tn ti cc s a1 , a2 , ..., an v b1 , b2 , ..., bn sao cho xij = ai + bj


vi mi s nguyn dng i, j tha mn 1 6 i < j 6 n

Chng 6

thi olympic Rumani


.6.23.Hm f : R2 R c gi l olympic nu n tha mn tnh cht: vi

.c

n 3 cc im ri rc A1 , A2 , , An Rn , nu f (A1 ) = f (A2 ) = =
f (An ) th cc im A1 , A2 , , An c gi l cc nh ca a gic li.

Cho P C[X] khc a thc hng. Chng mnh rng hm f : R2 R


c cho bi f (x, y) = |P (x + iy)|, l olympic khi v ch khi tt c cc
nghim ca P l bng nhau.

Li gii: Trc ht ta gi s rng tt c cc nghim ca P l bng


nhau, khi ta vit c di dng:

P (x) = a(z z0 )n vi a, z0 C v n N. Nu A1 , A2 , , An l
cc im ri rc trong R2 sao cho f (A1 ) = f (A2 ) = = f (An ) th
A1 , A2 , , An nm trn ng trn vi tm l (Re(z0 ), Im(z0 )) v bn
q

knh l n |f (A
, suy ra cc im l cc nh ca mt a gic li.
1 )|
Ngc li, ta gi s rng khng phi tt c cc nghim ca P l bng
nhau, khi P (x) c dng:

P (x) = (z z1 )(z z2 )Q(z) vi z1 v z2 l 2 nghim phn bit ca


P (x) sao cho |z1 z2 | l nh nht. Gi l l ng thng i qua hai

im Z1 v Z2 vi Z1 = (Re(z1 ), Im(z2 )),Z2 = (Re(z2 ), Im(z2 )), v t


z3 = 12 (z1 + z2 ) sao cho Z3 = (Re(z3 ), Im(z3 )) l trung im ca Z1 Z2 .
K hiu s1 , s2 ln lt l cc tia Z3 Z1 , Z3 Z2 , v r = f (Z3 ) 0. Ta phi

c r 0, bi v nu ngc li ta c z3 l mt nghim ca P sao cho:

33

thi olympic Rumani


|z1 z3 | |z1 z2 |, iu ny l mu thu vi |z1 z2 | l nh nht.
Do
lim f (Z) = lim f (Z) = +.
ZZ3
Zs1

ZZ3
Zs1

v f lin tc, tn ti Z4 s1 v Z5 s2 sao cho f (Z4 ) = f (Z5 ) = r. Do


vy f (Z3 ) = f (Z4 ) = f (Z5 ) v Z3 , Z4 , Z5 khng phi l cc nh ca a
gic li. Do vy, f khng phi l olympic.
.6.24.Vi n 2 l s nguyn dng. Tm s cc hm f : {1, 2, , n}
{1, 2, 3, 4, 5} tha mn tnh cht: |f (k + 1) f (k)| 3 vi k =
1, 2, , n 1

Li gii: Ta c n 2 bt k v tm s cc hm tng ng. Nu


f : {1, 2, , n} {1, 2, 3, 4, 5} phi tha mn cho th f (n) 6= 3
bi nu ngc li th f (n 1) 0 hoc f (n 1) 6, v l. K

hiu an , bn , dn , en l s cc hm f : {1, 2, , n} {1, 2, 3, 4, 5} tha


mn tnh cht cho sao cho f (n) tng ng bng 1, 2, 4, 5. Khi

.c

a2 = e2 = 2 v b1 = d2 = 1, v do vy vi n 2 :

an+1 = en + dn , bn+1 = en

en+1 = an + bn , dn+1 = an

Ta cn tm an + bn + dn + en vi n 2. Ta c: a2 = e2 v b2 = d2 ;

bng quy np ta c an = en v bn = dn n 2. Do vy vi n, ta c:
an+2 = en+1 + dn+1 = an+1 + bn+1 = an+1 + en = an+1 + an

do vy, {an }n2 tha mn nh dy Fibonaci {Fn }n0 , vi cc ch s c


chn sao cho: F1 = 0 v F1 = 1. Bi v a2 = 2 = F2 v a3 = e2 + d2 =
3 = F3 , vy suy ra an = Fn vi n. Do ,an +bn +dn +en = 2(an +bn ) =

2en+1 = 2an+1 = 2Fn+1 vi n 2 v 2Fn+1 tha mn tnh cht cho.

.6.25.Cho n 1 l mt s nguyn dng v x1 , x2 , , xn l cc s thc sao


cho: |xk+1 xk | leq1 vi k = 1, 2, , n 1. Chng mnh rng:


n
n
n2 1
X
X


xk
|xk |


4
k=1
k=1

34

Nguyn Hu in, HKHTN H Ni

Li gii: Nu s cc s xk m ln hn s cc s xk dng th (a1 , , an )


l mt hon v ca (x1 , , xn )(tng ng l (x1 , , xn )) sao cho
a1 , , an l mt dy khng gim. Do cch xy dng, lc lng P cc

s dng ak khng nhiu phn t hn lc lng N cc s mak , v do


n1
. V N l khc rng v a1 , , an l khng gim, cc
vy |P |
2
phn t ca P l ak0 +1 < ak0 +2 < < ak0 +l vi k0 > 0
Gi s rng 1 i n 1. Trong dy x1 , , xn phi c hai phn t
k nhau xj v xk sao cho xj ai v xk ai+1 suy ra 0 ai+1 ai

xk xj 1. Do vy, ak0 +1 ak0 + 1 1,ak0 +2 ak0 +1 + 1 2.


K hiu P v N ln lt l tng ca cc s trong P v N. Mt khc
ta c:

.c

|P N | |P N | |2P |


n1
)
2(1 + 2 + +
2

n1 n1
n2 1

.(
+ 1) =
2
2
4

.6.26.Cho n, k l cc s nguyn dng ty .Chng minh rng tn ti cc s

nguyn dng a1 > a2 > a3 > a4 > a5 > ksao cho:

n = Ca31 Ca32 Ca33 Ca34 Ca35

 
a(a 1)(a 2)
a
=

3
6

3
Li gii: Ta thy rng: n + Cm
> 2m + 1 vi m ln hn gi tr N, bi
v v tri l bc 3 vi h s cao nht dng trong khi v phi l tuyn

tnh vi m.
3
= m(m1)(m2)
Nu m 0(mod4), th Cm
l chn bi v t s chia ht
6
3
l
cho 4 cn mu s th khng.Nu m 3(mod4) th Cm
= m(m1)(m2)
6
l bi v c t s v mu s u chia ht cho 2 nhng khng chia ht
3
cho 4. Do vy ta chn m > maxk, N sao cho n + Cm
l s l.

35

thi olympic Rumani

3
3
3
3
Ta vit: 2a+1 = n+Cm
> 2m+1. Ta thy rng:(Ca+3
Ca+2
)(Ca+1

3
2
2
Ca ) = Ca+2 Ca = 2a + 1. Do vy
    
 
 
  
m
a
a+1
a+2
a+3
m
=
+

=
n = (2a + 1)
3
3
3
3
3
3

tha mn yu cu bi ton v a + 3 > a + 2 > a + 1 > a > m > k


.6.27.Cho P1 P2 Pn l mt a gic li trong mt phng. Gi s rng vi cp
nh Pi , Pj , tn ti nh V ca a gic sao cho Pi V Pj =
minh rng n = 3

.
3

Chng

Li gii: Trong li gii ny ta s dng kt qu sau:


Cho tam gic XYZ sao cho XY Z 3 th tam gic l u hoc

th tam gic XYZ

max{Y X, Y Z} > XZ. Tng t nu XY Z


u hoc min{Y X, Y Z} < XZ

.c

Chng ta ch ra rng tn ti cc nh A, B, C v A1 , B1 , C1 sao cho:(i)


tam gic ABC v A1 B1 C1 l tam gic u v (ii) AB(tng ng l A1 B1 )
l khong cch nh nht (ln nht) khc 0 gia 2 nh. Hn na, A, B

l 2 nh phn bit sao cho AB c di nh nht, v C l nh sao


cho ACB = 3 . Khi max{AC, CB} AC, tam gic ABC phi

l tam gic u. Tung t, ta chn A1 , B1 sao cho A1 B1 c di ln


nht, v nh C1 sao cho A1 C1 B1 = 3 , khi tam gic A1 B1 C1 l tam

gic u.
Ta ch ra rng 4ABC
= 4A1 B1 C1 . Cc ng thng AB,BC, CA chia
mt phng thnh 7 phn. Gi DA gm cc phn do tam gic chia m
nhn BC lm bin v cc phn c to ra ti phn to ra cc nh
B v C. Tng t ta nh ngha cho DB v DC . Bi v a gic cho
l li, nn mi hoc nm trong 1 phn hoc trng vi A, B, C.
Nu 2 im bt k trong A1 , B1 , C1 , gi s l A1 , B1 nm trong min
DX , th A1 XB1 < 3 . Do vy, max{A1 X, XB1 } > A1 B1 , mu thun
vi A1 B1 l ln nht.
Hn na, khng c hai im trong A1 , B1 , C1 trong cng 1 phn. By
gi ta gi s rng mt trong cc im A1 , B1 , C1 ( gi s l A1 ) nm
trn 1 phn(gi s l DA ). Bi v min{A1 B, A1 C} BC, ta c

BA1 C 3 . Ta c B1 khng nm trong DA . Bi v a gic cho l

36

Nguyn Hu in, HKHTN H Ni


li, B khng nm trong tam gic AA1 B1 , v tng t C khng nm trong
tam gic AA1 B1 . T c B1 nm trn min ng c bin l cc tia A1 B
v A1 C. Tng t, vi C1 . Hn na, 3 = B1 A1 C1 BA1 C = 3 , du
bng xy ra khi B1 v C1 ln lt nm trn tia A1 B v A1 C . Bi v a
gic cho l li,nn iu ny ch xy ra khi B1 v C1 ln lt bng B
v C -trong trng hp BC = B1 C1 , ta c tam gic ABC v A1 B1 C1 l
bng nhau.
Mt khc, khng c im no trong A1 , B1 , C1 nm trn DA DB DC ,
do chng ln lt trng vi A, B, C. Trong trng hp ny, tam gic
ABC v A1 B1 C1 l trng nhau.
Do vy hai nh bt k ca a gic c khong cch ging nhau, nh

AB = A1 B1 . iu ny l khng th xy ra nu c hn 3 im trong mt
phng th hon ton khng c tnh cht ny. Do vy n=3
.6.28.Chng minh rng tn ti v hn b gm 4 s nguyn dng(x, y, z, t)

.c

sao cho c chung ln nht ca 4 s l 1.v tha mn:

x3 + y 3 + z 2 = t4

Li gii: t a = k 3 vi k l s chn v k > 0 ta c:

tc l

(a + 1)4 (a 1)4 = 8a3 + 8a

(2k 3 )3 + (2k 3 )3 + [(k 3 1)2 ]2 = (k 3 + 1)4

Bi v k 3 + 1 l s l,nn (2k 3 , k 3 + 1) = (k 3 , k 3 + 1) = 1. Do vy, ta c


v hn b bn s dng (x, y, z, t) = (2k 3 , 2k, (k 3 1)2 , k 3 + 1) vi k > 0
l s chn, tha mn cc iu kin ca bi ton.
.6.29.Biu din nh phn ca mt s nguyn dng l a, c xc nh bng
thut ton n gin sau: xc nh mt s nguyn dng nh nht n sao
cho 22000 l c ca an 1

Li gii: Bi v a l s l nn (a, 2k ) = 1 vi k 0. Do vy, theo nh


k
k1
a(2 ) 1(mod2k ) vi k. Do vy bc n ca a
l Euler, ta c a2
modulo 22000 chia cho 22000 1 = 21999

37

thi olympic Rumani

Nu a 1(mod22000 ) suy ra n=1. Ta gi s rng a 6= 1(mod22000 ) . Vi


mi m 1 , ta vit:
m

m1

a2 1 = (a 1)(a + 1) (a2 + 1)(a2 + 1) (a2


{z
|

+ 1)
}

().

Biu din nh phn ca a c kt thc l 2 ch s 01 hoc 11. Ta c


k
a 1(mod4) v do vy a2 1(mod4) vi k 1. Do vy phn tch

(*)vi m c nh (m 1), 21 l s m cao nht ca 2 m chia ht cho


m 1 biu thc mi ngoc n pha trn .
Nu a 1(mod4), khi a 6= 1, biu din nh phn ca a kt thc l:
1 |00 {z
01}

ch s

vi s l s nguyn ln nht sao cho 2s |(a 1). Trong trng hp ny, s

.c

m cao nht ca 2 chia cho a 1 l 2s trong khi s m cao nht ca 2


chia ht cho a + 1 l 2

Nu thay th l a 1(mod4) , khi bi v a 6= 1 , biu din nh phn


ca a kt thc l:

1 |011{z
1}

ch s

vi s l s ngueyen ln nht sao cho 2s |(a + 1). Trong trng hp ny,

s m cao nht ca 2 chia cho (a + 1) c 2s trong khi s m cao


nht ca 2 chia cho (a 1) c 2.
Trong mi trng hp, ta s dng (*) v kt qu s m cao nht ca 2

chia cho (a2 1 c 2s+m . T c m 1 nh nht sao cho a2 1


chia ht cho 22000 l 2000 s (nu s<2000) hoc 1 ( nu s 2000).
m

Trong cc trng hp tng ng ta c n = 21999s hoc n = 2. Bi v ta


c th d dng s dng biu din nh phn ca a suy ra hai trng
hp v gi tr ca s l g, ta c th s dng biu din nh phn ca a
tm n.
.6.30.Cho tam gic nhn ABC v im M l trung im ca BC. Tn ti duy
nht mt im trong N sao cho ABN = BAM v ACN = CAM.
Chng minh rng BAN = CAM.

38

Nguyn Hu in, HKHTN H Ni


Li gii: Cho B l im nm trn tia AC sao cho ABB = BAM,
0
0
cho C l im nm trn tia AB sao cho ACC = CAM. Khi N
0

l giao im ca hai ng thng BB v CC


ng thng i xng vi AM qua ng phn gic ca gc BAC, ct
0

BB ti P. Gi D l im i xng ca A qua M, t gic ABCD


l hnh bnh hnh. Bi v P AB = CAM = CAD v ABP =
0

MAB = DAB = ADC, tam gic ABP v ADC l ng dng. Do


AP
AB
=
. Bi v BAD = P AC, nn tam gic BAD v tam
vy,
AD
AC
gic PAC ng dng. Hn na, ACP = ADB = CAM. T suy
ra P nm trn ng thng CC cng nh nm trn BB , v do vy
N P . Hn na c BAN = BAP = CAM nh yu cu bi ton.
0

.c

Chng 7
thi olympic Nc Nga
m

.7.31.Sasha th xc nh vi s nguyn dng x 6 100. Anh ta chn hai s

.c

nguyn dng bt k M v N m nh hn 100 v c cu hi "S no


ln nht trong cc c s chung ca x + M v N?" Chng minh rng
Sasha c th xc nh c gi tr ca x sau 7 cu hi.

Li gii: +) Vi n = 0, 1, 2, ..., 6, t an l s nguyn duy nht trong


[0; 2n ) tha mn 2n |(x an ). R rng a0 = 0.

+) Vi n 6 5, an+1 bng an hoc an + 2n m khi kt qu c vn c


nu v ch nu gcd (x + 2n an , 2n+1 ) = 2n
V 2n an < 2n+1 < 100 ta suy ra nu Sasha bit c gi tr ca an ,

anh ta c th xc nh c an+1 vi mt cu hi iu kin bng cch


t (M, N) = (2n an , 2n+1 ). Do sau 6 cu hi, Sasha c th xc

nh c a1 , a2 , ..., a6 v kt lun x bng a6 hoc a6 + 64


Bi v a6 6= a6 + 64 (mod 3), Sasha c th xc nh c x nu anh ta
pht hin ra liu c hay khng x a6 (mod 3) vi cc cu hi ca anh
ta.
Tht vy, anh ta c th nu t N = 3 v M {1, 2, 3} nn 3 |(a6 + M ) ,

anh ta s thu c cu tr li "3" nu v ch nu x a6 (mod 3).

.7.32.Cho O l tm ng trn ngoi tip tam gic nhn ABC. ng trn


1 vi tm K i qua cc im A, O, C m ct cc cnh bn AB v
BC ti M v N. t L l im i xng vi K qua ng thng MN.

40

Nguyn Hu in, HKHTN H Ni


Chng minh rng BLAC.
Li gii: Gi , , l cc gc A, B, C ca tam gic ABC. V t gic
\ = , BMN
\ = nn MKC
\ = 2 ; NKA
\ = 2
ACNM ni tip, BNM
V ng thng AC l trc ng phng ca ng trn v 1 nn
n vung gc vi OK. Khi ta c:
\ = COK
\ = OAK
\ = OCK
\=
AOK
\ = 2 4
Do AKC
Kt hp cc iu trn, ta c:
\ = 2 + 2 (2 4) = 2
MKN

\ =
V L l im i xng ca K qua ng thng MN nn ta c: MLN
2 v LM = LN. Do LMN OCA

.c

Mt khc ta cng c: MBN CBA bi v t gic ACNM ni tip.


Do v O l tm ng trn ngoi tip tam gic ABC v L l tm
\ =
ng trn ngoi tip tam gic MBN nn ta suy ra MLB
[ = , ta d dng suy ra BLAC.
\ = . Bi v BAC
V khi : MBL
2

.7.33.C vi thnh ph trong mt quc gia v mt cch t tn ng. Ni

m mi con ng ni hai thnh ph v khng c hai con ng no ni


hai thnh ph c tn ging nhau. N c hiu rng trong mi thnh
ph u c ti thiu 3 con ng i ra. Chng minh rng tn ti mt

con ng tun hon (c ngha l ni kt thc l ni bt u) nh th


s con ng trong nhng con ng khng th chia ht cho 3.
Li gii: Ta s dng s dch chuyn trong l thuyt th. Trong mt
th, tt c mi nh u c bc t nht l 3. Ta chng minh tn ti
mt chu trnh m di ca m khng chia ht cho 3.
Thc hin thut ton sau:
+) C nh mt im u v1 sau cho v1 , v2 , ..., vi nu tn ti mt
im ring t i nh v ti nh gn k th vi+1 l mt nh. Bi v
th c gii hn v tt c cc nh t c bng thut ton ny l
r rng.
Qu trnh ny kt thc ti vi nh vn . Ta bit mi nh c t nht bc

41

thi olympic Nc Nga

3 v bi s tha nhn mi nh gn k vi vn lp thnh mt dy. Nh


vy, vn l nh k vi va , vb , vn1 vi a < b < n 1.

Ta c 3 chu trnh:

va va+1 va+2 ... vn1 vn va

vb vb+1 vb+2 ... vn1 vn vb

va va+1 va+2 ... vb1 vb vn va

Nhng chu trnh trn c di n a + 1 ; n b + 1 ; b a + 2 theo


th t. Bi v (n a + 1) (n b + 1) (b a + 2) = 2 khng th

.c

chia ht cho 3
Do tn ti 1 trong 3 chu trnh trn c di khng chia ht cho 3.

.7.34.Cho x1 , x2 , ..., xn l cc s thc (n > 2) tha mn iu kin 1 < x1 <

13
13
x2 < ... < xn < 1 v x13
1 + x2 + ... + xn = x1 + x2 + ... + xn . Chng
13
13
minh rng: x13
1 y1 + x2 y2 + ... + xn yn < x1 y1 + x2 y2 + ... + xn yn vi mi

s y1 < y2 < ... < yn .

Li gii: Vi 1 < x < 1, t f (x) = x x13


Ta phi c x1 < 0 v nu khng f (x1 ) > 0 v f (x2 ) > 0, f (x3 ) >

0, ..., f (xn ) > 0.


iu ny v l v f (x1 ) + f (x2 ) + ... + f (xn ) = 0 c cho bi phng
trnh trn.
Chng minh tng t ta cng suy ra xn > 0
Gi s rng 2 6 i 6 n
+) Nu xi 6 0 th khi ta c
x1 < x2 < ... < xi1 < xi 6 0 v

n
P

j=1

f (xj ) =

i1
P

f (xj ) > 0

j=1

+) Nu thay th xi > 0 th khi ta c: 0 < xi < xi+1 < ... < xn v ta


n
P
cng chng minh c
f (xj ) > 0
j=1

+) S dng cng thc ly tng Abel v kt qu trn ta c:

42

Nguyn Hu in, HKHTN H Ni


n
X
i=1

= y1

n
P

i=1

n
P

i=2

f (xi ) +

n
P

i=2

xi yi

n
X

x13
i yi

i=1

n
X

yi f (xi )

i=1

(yi yi1 ) (f (xi ) + f (xi+1 ) + ... + f (xn ))

(yi yi1 ) (f (xi ) + f (xi+1 ) + ... + f (xn )) > 0

T ta d dng suy ra iu phi chng minh.


.7.35.Gi AA1 , CC1 l cc ng cao ca tam gic nhn ABC. ng phn
gic ca gc nhn gia hai ng thng AA1 , CC1 ct cc cnh AB v
BC ti P, Q tng ng. Gi H l trc tm tam gic ABC v M l
[ ct on HM ti
trung im ca cnh AC, ng phn gic ca ABC
R. Chng minh rng t gic P BQR ni tip c mt ng trn.

Li gii: H ng vung gc vi cnh AB v BC ti P , Q tng ng,


chng ct nhau ti R00 . Gi S l giao im ca R0 P v HA, T l giao

.c

im ca R0 Q v HC.
H ng vung gc t M ti AB, ct HA ti U v h ng vung

gc t M ti BC ct HC ti V .
\
V P SH, HT Q c cc cnh tng ng song song v P[
SH = HT
Q

Do P Q l ng phn gic ca gc nhn to bi hai ng thng


\.
AA1 , CC1 nn P[
HS = QHT
Do P HS QHT
\ = ABC
[ = QCH
\ v P
\
\ nn
Mt khc, v HAP
HA = QHC
2

Do :

P HA QHC

HP
HC
2MU
MU
HV
HT
=
=
=
=
=
HS
HQ
HA
2MV
MV
HU

Khi , php v t tm H bin ng thng P S thnh ng thng


MU v cng bin ng thng QT thnh ng thng MV . Do n
bin R0 = P S QT thnh M = MU MV . V th 3 im H, R0 , M

thng hng
\
\ l ng
Ta li s dng gi thit P HA QHC, ta c HP
B, HQB
\
\
d v chng cng ph vi hai gc HP
A, HQC.
\
\0 .
Nh vy, BP = BQ v BR0 P BR0 Q nn P
BR0 = QBR

43

thi olympic Nc Nga

Do R0 nm trn c hai ng thng HM v ng phn gic ca


[ suy ra R0 R
ABC,
[
T d dng suy ra t gic P BQR ni tip v BP
R=

[
= BQR

.7.36.C 5 vin ngc c trng lng khc nhau. Oleg bit c trng lng
ca tng vin. Vi mi vin ngc x k hiu m (x) l trng lng ca
n. Dmitrii c gng xc nh trng lng ln nht ca cc vin ngc
. Anh ta c php chn 3 vin A, B, C v hi Oleg rng: "C phi
m (A) < m (B) < m (C) khng?". Oleg ch tr li "ng" hoc "Sai".
Hi Dimitrii c th xc nh c trng lng ln nht sau 9 cu hi
hay khng?
Li gii: Chng ta s ch ra rng Dimitrii s khng th xc nh c
khi lng vin ngc ln nht sau 9 cu hi.

Gi s Dimitrii c mt phng php xc nh c vin ngc c khi


lng ln nht sau 9 cu hi hoc t hn. Gi s rng sau khi Oleg tr li

.c

cu hi th i ca Dimitrii c chnh xc xi vin ngc c trng lng ln




nht tha mn cu hi th i. Ta ch ra rng xi+1 > max xi 20, 12 xi

vi i = 1, 2, ..., 8
rng c 5! = 120 cch ly c vin ngc c trng lng ln
nht. Sau vi 3 vin ngc A, B, C bt k, ng 16 vin ngc c th c

m (A) < m (B) < m (C).


Nh vy, nu Dimitrii hi liu m (A) < m (B) < m (C) v Oleg tr li

"Sai" th Dimitrii c th loi b ti a 20 trong s 120 kh nng c th.


Trong trng hp xi+1 > xi 20 vi mi i. Vi mi xi m ph hp i cu

hi u tin, mt phn s1 ca nhng kh nng c loi b nu Oleg


tr li "ng" ti cu hi th i + 1; trong khi phn b ca mt phn
ca s2 s c loi b nu Oleg tr li "Sai".
Nu |S1 | 6 x2i v Oleg tr li "ng" th ta c:
xi
xi+1 = xi |S1 | >
2
Mt khc ta c: |S2 | 6 x2i
Nu Oleg tr li "Sai" ta li c xi+1 > x2i
Do , nu x1 = 120; x2 > 80; x3 > 60; x4 > 40; x5 > 20;
x6 > 10; x7 > 5; x8 > 3; x9 > 2

44

Nguyn Hu in, HKHTN H Ni

.c

T , Dimitrii khng th chc chn rng anh ta tm thy c kt qu


sau 9 cu hi.

Chng 8

thi olympic i Loan


.8.37.Cho tam gic nhn ABC, AC > BC v M l trung im AB. Cc ng

.c

cao AP v BQ gp nhau H, ng thng AB v BQ ct nhau R.


CMR: RH CM.

Li gii: Gi S l chn ng cao h t C xung AB v X l chn

ng vung gc t H xung CM.


\
\=HXC
\ = nn H, P, Q, X v C cng nm trn mt
V HP
C = HQC
2
ng trn.
\ = HSM
\ = nn cc im H, X, S v M cng
Tng t, v HXM
2
thuc mt ng trn.

Hn na, P, Q, S v M thuc cng mt ng trn v chng nm trn


ng trn 9 im ca tam gic ABC. Theo tnh cht i xng, hai
trong cc trc i xng ca 3 ng trn ny l AB, PQ, HX phi
trng nhau. V R = AB PQ nu R phi thng hng vi H v X.
Do : RH CM.
.8.38.Gi (h) l s cc s nguyn dng n tho mn UCLN ( n, k) = 1 v
n k . Gi s (5m 1) = 5n 1 vi m, n nguyn dng no . CMR:
UCLN (m, n) > 1.
Li gii: Trong li gii ny, chng ta s dng nhng l thuyt v hm
sau:

46

Nguyn Hu in, HKHTN H Ni

(ab) = (a) (b)(1)


(p ) = p p1 (2)

(1) Vi a, b nguyn t cng nhau


(2)Nu p nguyn t v l s nguyn dng
Gi s phn chng rng UCLN (m, n) = 1
u tin ta ch ra m l s l
Ta c: 5n 1(mod8) nu x l chn
.
Nu m l chn th 5m 1..8 nhng 5m 1 6= 8
.
V 5m 1 6= 8, 5n 1 = (5m 1)..(16) = 8
hoc (8).(P ) = 8(P ) 0(mod8)

Vi P > 1 l lu tha ca s nguyn t l P Do n phi chn, UCLN


( m,n) = 1 theo nh gi thit phn chng. Tip theo ta gi s rng

P 2/(5m 1) vi P nguyn t l R rng PX5, v 5 c............... modulop,


gi d l ..................... nn ta c d/m

.c

Li c :d/(p2 ) v (5m 1) = 5n 1 do d/n.


Nhng d>1 v 5 6= 1(modp) vUCLN(m,n)6=1-a (theo gi thit phn
chng)
Q
Do ,5m 1 = 4
p vi S l tp cc s nguyn t l.

pS

Cho P l phn t bt k thuc S. V 1 = ( p) = ( 5p ) = ( m5 )n v m l l,


( 5p ) = 1

Li c, theo lut tng h : ( p5 )( p5 ) = (1) (51)(p1)


=1
4
dn n( p5 ) = 1m P 1 hoc 4 (mod5).

Hn na, ta khng th c P 1(mod5) v 5 chia ht (5m 1) = 5n1 v


P 1 = (p) , iu ny khng xy ra. Do P 4 (mod 5)
Q
Suy ra 1 5m 1 = 4
p 4 4|s| (mod5) v1 = 5n 1 =
pS

(4).

pS

(p) = 2

pS

(p 1) = 2 3|s| (mod5)

T phng trnh u ny ta c |S| phi chn. Nhng t phng trnh


th hai ta c:|S| 3(mod4)iu ny mu thun

Do gt ca ta l sai, vy UCLN (m, n) > 1.

.8.39.Cho A ={1, 2,......, n} . Vi n N . Mt tp hp con ca A c gi l

kt ni nu n l s nguyn ln nht sao cho A cha k tp i mt

47

thi olympic i Loan

khc nhau: Sao cho giao ca hai tp bt k Ai v Aj l mt tp kt


ni
Li gii: Gi A1 , A2 , ...Ak l cc tp con khc nhau ca A trong gi
thit
t
m = max (min Ai )
1in

v gi thit rng min Ai0 = m


Mi tp Ai c phn t b nht nh hoc bng m, do cch xc nh m v
mi tp Ai c phn t ln nht ln hn hoc bng m hocAi Ai0 =
l tp khng kt ni.
Do , mi cp k(min Ai , max Ai ) bng mt trong m(n+1m)cp

( r,s) m 1 n m s n vi mi cp (r,s) ta ch ra rng t nht


mt tp Ai c (MinAi , MaxAi ) = (r, s)

.c

Nu c hai tp khc nhau th giao ca chng l mt tp kt ni cha r


v s v cha c r, r+1,. . . ,s

.Gi tr ln nht ny t c nu Ai l tp con kt ni ca A


 

n2 +2n
4

iu ch ra rng c hai tp u bng {r, r + 1, ..., s } iu ny mu


thun.
 n 2  n 2
2 =
Do

k
ln
nht
l
bng
k
=
m(n
+
1

m)

(n
+
1)
2
j
k

cha m0 m m0 =  2 
n

.8.40.Cho
hm F : N N tho mn

f (1) = 0

f (n) = maxn {f (j) + f (n j) + j} n 2


0jb 2 c

Tnh f(2000).

Li gii: Vi mi n Z + ta ch n biu din ca n trong h nh


phn Ch rng c s ca biu din l s thay i t nht 1 k t bn
tri ca biu din , v th c s c bt u bi a1
Chng ta gi gi tr thp phn ca c s ny l gi tr ui ca n vi
mi mt xut hin trong biu din nh phn ca n, nu n i din cho
s 2k , 2k

k
2

l mt gi tr place ca n.

48

Nguyn Hu in, HKHTN H Ni


t g(n) l tng gi tr tail v place ca n
Ta CMR: F(n) = g(n)
t g(0) = 0 r rng g(1) =0. . .
 
u tin ta CMR: g(n) g(j) + g(n j) + jn, j tho mn 0 j n2
iu ny l hin nhin vi j=0 v g(0) =0
By gi ta b sung s k t ca n-j

Vi trng hp c bn ( khi n-j c k t 1) ta ch c th thay n-j. Trong


trng hp (n,j) = (2,1) hoc ( n, j ) = (1;0)
Trng hp (1) d c l ng.
TH1: n-j v j c cng s cc k t l k +1
t a v b ln lt l s cc ch s 1 ngoi cng bn tri ca n-j v j.

ta s ch ra rng g(n) = g(a + b) 2k+1 g(2k + a) + g(2k + b) + (2k + b)


rng: g(a + b) g(a) + g(b) + b( iu ny ng do gi thit)

.c

Do ta c: g(a + b + 2k+1) g(a + b) g(2k + a) g(a) + g(2k +


b) g(b) + 2k (2)
g(2k + a) = g(a) + 2k k + a
2
v phi ta c
g(2k + b) = g(b) + 2k k + b
2
k
2

Do v phi bng: 2k k2 + a + 2k
Cn i vi v tri: v a < 2k , b < 2k

+ b + 2k = 2k+1

k+1
2

+a+b

Biu din nh phn ca a + b + 2k+1 ging nh biu din nh phnca


a+b vi vic thm 1 vo 2k+1 v tr
Do g(a + b + 2k+1)bng g(a+b)cngvi gi tr ui ca a+b v

(2)

.
cng2k+1 k+1
2
V th g(a + b + 2k+1 ) g(a + b) bng v phi, ta chng minh c
TH2: n-j c s k t nhiu hn j
Gi s n-j c k +1 k t v a = n j 2k . Ta cn CM rng: g(a +
j + 2k ) g(a + 2k ) + g(j) + j. Ta bit theo gi thit:g(a + j)
g(a) + g(j) + min {a, j} .

Do ta cn chng minh iu kin l:g?(a + j + 2k ) g(a + j)


g(a + 2k ) g(a) + j min {a, i} (3)

Theo TH1 v phi: g(a + 2k ) g(a) = 2k k2 + a


Do ,v phi bng: 2k k2 + a + j min {a, j} = 2k k2 + max {a, j}

49

thi olympic i Loan


Vi v tri ca (3): nu a + j < 2k
chng hn nh 2k k t khng c trong tng a + j + 2k
th g = (a + j + 2k ) = g(a + j) + 2k
do VT(3) VP(3)

k
2

+a+j

Mt khc nu c2k k t khng c trong tng a + j + 2k th g(a +


j + 2k ) = g(a + j) + 2k+1 k+1
2k k2
2

V th, v tri = 2k+1 k+1


2k k2 = 2k
2
Do (3) ng.
Gi thit c CM hon ton

k
2

+ 2k > 2 k

k
2

+ max {a, j}

Ta CMR: Du bng xy ra khi g(n) = g(j) + g(n j) + jvi 1 s


gi tr j

t 2k l lu tha ln nht ca 2 nh hn n v t j = n 2k th
g(n) = g(n 2k) + g(2k ) = g(n j) + n 2k = j

iu cho thy f (n) = g(n)n


Vy vi vic tm gi tr place v gi tr ui ca 2000 (vi biu din

.c

nh phn 11111010000) ta c:f (2000) = 10864.

Chng 9
thi olympic Th Nh K

.c

.9.41.Tm cc b 4 s xp theo th t (x, y, z, w) ca cc s nguyn vi 0 x,


y, z, w 36
x2 + y 2 z 3 + w 3 (mod 37)

Li gii: Tt c cc ng d s l mod 37. Vi mi k trong khong 0

n 36 ta tm c cc cp s nguyn (x, y) vi 0 x, y 36 tha mn


x2 + y 2 k. Ch rng iu ny tng ng vi (x 6y)(x + 6y) k.

Trc ht ta xem xt trng hp k = 0. Vi mi y {0, 1, . . . , 36} ta


c (x 6y)(x + 6y) 0 nu v ch nu x 6y.. V vy c mt cp (x,

y) vi y = 0 x2 + y 2 0 ( l (x, y) = (0, 0)), v vi bt k y no


khc c 2 cp (x, y) nh vy. Do vy c tng cng 2.36 + 1 = 73 cp
(x, y) x2 + y 2 0.

Gi ta xt trng hp k 6= 0. a x + 6y, b x 6y. Vi bt k


gi tr a {1, 2, . . . , 36} c chnh xc mt gi tr b {1, 2, . . . , 36}

ab k. Vi mi cp (a, b) trong 36 cp tng ng vi mt nghim (x, y)


duy nht v ta phi c x (a + b)21 , y (a + b)121 . Do vy phng

trnh (x 6y)(x + 6y) k c chnh xc l 36 nghim (x, y) khi k 6= 0.


Ta xem xt s cp 4 (x, y, z, w) x2 + y 2 z 3 + w 3 0. C 3
cn bc 3 r1 , r2 , r3 ca 1 mod 37, l: Nu ta g l mt phn t
nguyn thy mod 37 th cn bc 3 l 1, g 12 , g 24 . Vi bt k z no, ta

51

thi olympic Th Nh K

c z 3 + w 3 0 nu v ch nu w bng r1 z, r2 z hay r3 z. Do vy c
109 cp (z, w) z 3 + w 3 0, mi cp z = 0 v 3 cp z = z0

i vi miz0 {1, 2, . . . , 36}. trn ta tm ra rng c chnh xc


73 cp (x, y) x2 + y 2 0.. Do vy c 109.73 b 4 (x, y, z, w)

x2 + y 2 z 3 + w 3 0.
Vi mi cp trong 372 109 cp (z, w) z 3 + w 3 0 c chnh xc 36

cp (x, y) x2 + y 2 z 3 + w 3. Vy nn c (372 109).36 b 4 (x, y, z,


w) x2 + y 2 z 3 + w 3 0. V vy, c 109.73 + (372 109).36 = 53317
b 4 (x, y, z, w) x2 + y 2 z 3 + w 3 .
.9.42.Cho mt vng trn tm O, 2 ng tim cn xut pht t im S nm
bn ngoi ng trn c tip im l P, Q. ng thng SO giao vi

ng trn ti A, B vi B gn S hn A. Cho X l mt im nm trong


cung nh PB v ng SO giao vi cc ng QX v PX ln lt ti C,
D. Chng minh rng:

.c

1
2
1
+
=
AC AD
AB

Li gii: Ko di tia PC cho ct vi cung QB ti Y. Bng php i


\
\
\
xng cung BX v BY, n ch ra rng \
CP B = Y
P B = BP
X = BP
D.

Do vy, P B l phn gic bn trong ca CPD. p dng nh l ng


phn gic trong v phn gic ngoi ta tm ra c:

PC
AC
BC
=
=
BD
PD
AD

Thay BC = AB - AC, BD = AD - AB v chia na bn tri, bn phi


bi ng AB, ta c
AD AB
AB AC
=
AB.AC
AD.AB
iu ny c ngha l
1
1
1
1

AC AB
AB AD
iu ny tng ng vi ng thc cn chng minh.

52

Nguyn Hu in, HKHTN H Ni

.9.43.Vi 2 s nguyn dng bt k n, p. Hy chng minh rng c chnh xc


(p + 1)n+1 pn+1 hm s
f : {1, 2, . . . , n} {p, p + 1, . . . , p}
|f (i) f (j)| p vi tt c i, j {1, 2, . . . , n}.

Li gii: Vi m {p, p + 1, . . . , p}, c (min{p + 1, p m + 1})n l


hm s tha mn iu kin cho bao gm nhng gi tr ch nm trong
{m, . . . , m + p}. D nhin, (min{p + 1, p m + 1})n l hm s c cc gi
tr ch nm trong {m + 1, . . . , m + p}. Do vy, chnh xc
(min{p + 1, p m + 1})n (min{p, p m})n
l hm s tha mn iu kin cho vi gi tr m nh nht.

Biu thc ny bng vi (p + 1)n pn i vi mi gi tr ca p+1, m 0


v bng (p + 1 m)n (p m)n khi m > 0. Do , tng ca biu thc

.c

khi m 0 l

(p + 1)((p + 1)n p)n

m=1

((p + 1 m)n (p m)n ) = pn

p
X

trong khi tng biu thc khi m > 0 l tng

Cng 2 tng ny li ta c tng cc hm s tha mn iu kin cho


l (p + 1)n+1 pn+1 .
.9.44.Trong tam gic nhn ABC c bn knh ng trong ngoi tip R. ng
cao AD, BE, CF ln lt c di l h1 , h2 , h3 . Nu t1 , t2 , t3 ln lt l
chiu di cc tip tuyn t A, B, C ti ng trn ngoi tip ca tam
gic DEF. Hy chng minh:
3
X
3
ti
( )2 R
2
hi
i=1

.
Li gii: Cho H l trc tm ca tam gic ABC v X, Y, Z ln lt l
trung im ca AH, BH, CH. Bi ng trn ngoi tip ca tam gic

53

thi olympic Th Nh K

DEF l ng trn 9 im ca tam gic ABC, n qua cc im X, Y


v Z. Do t21 = AX.AD = AX.h1 hoc ( th1 1 )2 = AX. Ta c th tm
c cc biu thc tng t vi BX v CX. Vy nn bt dbgr thc cn
chng minh tng ng vi
3
AX + BX + CX R
2
(nhn mi v vi 2):
AH + BH + CH 3R.
= , C = ta c:
Cho A = , B
AF
AC. cos
=
= 2R cos
sin
sin

AH =

Tng t, BH = 2R cos v CH = 2R cos , bt ng thc cn chng

.c

minh tng ng vi

3
cos + cos + cos .
2

Nh rng tam gic ABC l tam gic nhn v hm t cos t l lm

trong khong (0, 2 ). Do , theo bt ng thc Jensen ta c v tri ca


bt ng thc cui cng ny t cc i khi 3 gc u bng 3 , trong
trng hp v tri bng 23 . Vy bt dng thc cui cng ny l ng

ng thi bt ng thc cn chng minh cng l ng.


.9.45.(a) Chng minh rng vi mi s nguyn dng n, s cp ca s nguyn
xp theo th t tha mn x2 xy + y 2 = n l hu hn v chia ht cho
6.] [(b) Tm tt c cc cp s nguyn (x, y) xp theo th t tha mn
x2 xy + y 2 = 727.

Li gii: (a) Bt k nghim (x, y) u phi tha mn bt ng thc:


n = x2 xy + y 2 =

(x y)2 x2 + y 2
x2 + y 2
+

2
2
2

v rt nhiu cp hu hn (x, y) tha mn c iu kin ny. Do vy


c rt nhiu nghim hu hn.

54

Nguyn Hu in, HKHTN H Ni


Tip n ta chng minh rng s nghim chia ht cho 6. Nu (x, y) l
nghim th (y, y x) cng l nghim. Php bin i tuyn tnh ny l
kh nghch, do n hon v tt c cc nghim v ta c th chia cc
nghim ra thnh cc lp vi mi lp di dng:
(x, y), (y, y x), (y x, x), (x, y), (y, x y), (x y, x)
i vi mt s nghim(x, y) ban u. R rng chng minh khng c
2 trong 6 nghim mi lp bng nhau tr khi x = y = 0 l khng th.
Do vy mi lp c 6 phn t ring bit v kt qu l c chng minh.
(b) Bt k nghim no vi x2 xy+y 2 = 727 ta c th p dng c php

bin i (x, y) (y, y x) nh phn (a) v c th (x, y) 7 (y, x),


c c nghim (x, y) khc vi y 0 x |y|. Gi ta tm tt c

.c

cc nghim nh vy vi y 0 x |y|. Sp xp li ng thc cn tm


ta c x2 xy + y 2 727 = 0. Xem xt ng thc ny l mt ton
phng trong y, ta c th p dng phng trnh bc 2 tm ra rng:

x 2908 3x2
y=
2

Do vy, 2908 3x2 phi l s chnh phng, v n khng th chia ht


cho 3. Do 3x2 x2 xy + y 2 = 727 ta bit c thm rng 2181

2908 3x2 2908 vi 46 < 2908 3x2 < 54. Kim chng nhng kh

nng ta thy rng ch 2908 3x2 = 49 c nghim nguyn x, kt qu


l ta c nghim duy nht(13, 18) ca phng trnh.

Do bt k nghim no cng c th bin i thnh (13, 18) bng


vic p dng 2 s m t trn. Nh vy bt k nghim no trong

(13, 18) hoc (18, 13) di bin i (x, y) 7 (y, y x), c ngha tt
c cc nghim ti x2 xy + y 2 = 727 l:
(13, 18), (18, 31), (31, 13), (13, 18), (18, 31), (31, 13),
(18, 13), (13, 31), (31, 18), (18, 13), (13, 31), (31, 18).
.9.46.Cho tam gic ABC, cc ng phn gic trong v ngoi ca gc A ln
lt ct ng thng BC ti D v E. Cho F l mt im ( khc im
A) ng thng AC tip xc vi ng trn c ng knh DE.

55

thi olympic Th Nh K

V tip tuyn ti A vi ng trn ngoi tip ca tam gic ABF v giao


vi ng trn ti A v G. Chng minh rng AF = AG.
Li gii: Ta chng minh cho trng hp C, B v E thng hng. Theo
th t ta chng minh cho cc trng hp khc tng t. Gi O l
tm ca . Theo nh l v ng phn gic trong v ngoi ca tam
gic, ta c:
CD
CA
CE
=
=
DB
AB
BE
Do vy CD(CE CB) = CD.BE = CE.DB = CE(CB CD) hoc (
cng CD (CE + CB) cho c 2 v).

2CD.CE = CB(CD + CE)


Bi v CD + CE = 2CO, ta c: CD.CE = CB.CO Mt khc, theo

.c

nh l tch im p dng vi C v ta c: CD.CE = CA.CF , suy ra


CB.CO = CA.CF . Do vy, theo nh l tch im vi cc im A, B, O,
F nm trn ng trn 1 no . Ta thc hin php nghch o A vi

tia AO. l mt vng trn qua A, im trc giao vi AO v bao gm 1


im P nm trn AO vowis AP = 2AO. Do vy, nh ca n di php

nghch o l mt ng trc giao vi AO v bao gm 1 im P trn


tia AO vi AP 0 = AO/2. Ni cch khc, nh l1 ca l ng trung

trc ca AO. K tip, php nghch o a 1 (vng trn i qua A, qua


O v tip xc vi AG) ti l2 khng i qua A, qua O v song song vi

AG. Suy ra php nghch o a F, giao gia v 1 , ti giao F ca l1


v l2 ; hn na php nghch o (php bin i ngc) a G, giao ca
1 v AG, ti giao ca l1 v AG. Php di xng qua trung im ca AO
a l1 ti chnh n v l2 ti AG; Do vy, s phn chiu ny a OF 0 ti
AG0 , c ngha l OF = AG. Bi F nm trn trung trc ca AO, ta
cng c OF = AG. Vy nn AF = AG, iu ny ng ngha vi AF
= AG. Kt qu c chng minh.
.9.47.Hy ch ra rng c th ct bt k lng tr tam gic c chiu di v hn
bi 1 mt phng cho ra kt qu thit din l mt tam gic u.

56

Nguyn Hu in, HKHTN H Ni


Li gii: Gi s rng mt mt phng trc giao vi cc cnh ca lng
tr ti A, B, C v t a = BC, b = CA, c = AB v khng mt tnh
tng qut gi s a b c. Vi t 0, xc nh:
q

f (t) = a2 + (t + c2 b2 + t2 )2 c2 + t2
Khi :

f (0) = a2 + c2 b2 c2 0
p

Mt khc, ta c f (b) = a2 + (b + c)2 c2 + b2 > 0. Bi f l lin tc

BC =

a2

+ (t0 +

.c

nn tn ti t0 sao cho f (t0 ) = 0. By g ta cho B nm trn 1 cnh vi


B, cch B 1 khong l t. C nm cng cnh vi C v cch C 1 khong l
p
c2 b2 + t20 v nm trn gi i din ca mt phng (ABC) tnh t
B. Theo nh l Pitago ta c:
q
q
q
AB 0 = c2 + t20 ; AC 0 = b2 + (c2 b2 + t20 ) = c2 + t20 ;
c2 b2 + t20 )2 =

c2 + t20

V vy mt phng (ABC) p ng c yu cu bi.

.9.48.Cho hnh vung ABCD, cc im M, N, K, L ln lt nm trn cc cnh


AB, BC, CD, DA sao cho MN song song vi LK v khong cch gia
MN v LK bng AB. Hy ch ra rng cc ng trn ngoi tip ca tam

gic ALM v NCK l giao nhau cn cc ng trn ngoi tip ca tam


gic LDK v MBN th khng giao nhau.
Li gii: Hng t gic sao cho AB nm ngang v nm pha trn CD,
A nm pha ty so vi B. Trc ht ta cho l AL > BN, ni cch
khc N nm pha bc (mc d khng nht thit phi chnh bc) so vi
L. Gi s ngc li th c 1 on nm ngang vi im cui tri L v
im cui phi trn MN vi di AB. Mt khc, di ca on
ny ln hn khong cch gia LK v MN , iu ny c gi thit
l AB. Do vy ta c s tri ngc nhau v AL > BN. Tng t ta cng
c th rt ra c AM > DK.
Dng P v Q sao cho cc t gic BMPN v DKQL l cc hnh ch nht.
Ta bit t phn trn rng P nm ng bc so vi Q. Dng R v S sao

57

thi olympic Th Nh K

cho R pha ng nam so vi Q v sao cho t gic PRQS l hnh ch


nht c cc cch song song vi cc cch ca hnh vung ABCD.
ch ra rng cc ng trn ngoi tip ca tam gic ALM v NCK l
giao nhau, ta thy rng cc ng trn b chn bi cc ng trn ngoi
tip tam gic ALM v NCK ln lt bao hm cc hnh ch nht ALRM
v CKSN. V vy cc ng trn ny u cha hnh ch nht PRQS. Do
cc min trong ca cc ng trn ngoi tip tam gic ALM v NCK
l giao nhau vy nn cc ng trn ngoi tip cng phi giao nhau.
Gi ta chng minh rng ng trn ngoi tip 1 ca tam gic MBN v
ng trn ngoi tip 2 ca tam gic LDK l khng giao nhau. Ch
rng chng cng ln lt l ng trn ngoi tip ca hnh ch nht

BMPN v DKQL. Cho l1 l tip tuyn vi 1 ti P, l2 l tip tuyn vi


2 ti Q. V MN song song vi LK do vy BP song song vi QD. Do

l1 vung gc vi BP v l2 vung gc vi QD nn ta c l1 song song vi


l2 . Vy nn mi im ca 1 nm trn hoc gi bn phi ca l1 cng

.c

nm pha bn phi ca l2 ; mt khc mi im trn 2 nm trn hoc


bn tri ca l2 . Vy nn 1 v 2 khng th giao nhau.

.9.49.Cho f : R R l mt hm tha mn :

|f (x + y) f (x) f (y)| 1

x, y R.

vittcx, y R. Chng minh rng tn ti mt hm g :R R tha mn


|f (x) g(x)| 1 vi mi x R, v g(x + y) = g(x) + g(y) vi mi

Li gii: Ta cho hm
f (2n x)
g(x) = lim
n
2n
tha mn iu kin u bi. Vic trc tin cn lm l ch ra rng tn
ti vi mi x. Thc t ta c th chng minh c iu ny v ng thi
cng chng minh c
|f (x) g(x)| 1

58

Nguyn Hu in, HKHTN H Ni


i vi tt c x. Trc ht thy rng t x = y = 2m x0 trong bt ng
thc cho ca f ta c:
m+1

f (2
x0 ) 2f (2m x0 ) 1

Chia cho 2m+1 ta c:




f (2m+1 x0 ) f (2m x0 )
1

2m+1
2m 2m+1
Vi bt k x xc nh no, xem xt tng v hn:


X
f (2m+1 x) f (2m x)

m+1
2
2m
m=0

V cc tr tuyt i ca cc s hng b chn bi mt dy 12 , 14 , . . ., tng


ca chng tin ti 1, tng ny hi t tuyt i v cng b chn bi 1.

.c

Mt khc, theo nh ngha tng v hn bng



n 
X
f (2m+1 x) f (2m x)
lim

n
2m+1
2m
m=0
n+1

Tng v hn trong gii hn bng ( f (22n+1 x) f (x)), c ngha l




f (2n+1 x)
lim
f (x)
n
2n+1

Gi ta c th ly ra hng s f(x) t


f (2n+1x)
lim
f (x)
n
2n+1
Suy ra gii hn trong biu thc cui cng ny hi t v ngu nhin n
cng chnh l gii hn ta mun dng xc nh g(x). Hn na, ta thy
phn trn rng lng cui cng ln nht bng 1, v vy ta c:
|g(x) f (x)| 1
Tip tc thy rng g(x + y) = g(x) + g(y)vi mi x, y. Thy rng:
f (2n x)
2n y
f (2n (x + y))

lim

lim
n
n 2n
n
2n
2n
n
n
n
f (2 (x + y)) f (2 x) f (2 y)
= lim
n
2n

g(x + y) g(x) g(y) = lim

59

thi olympic Th Nh K
Theo iu kin cho,
|f (2n (x + y)) f (2n x) f (2n y)| 1

vi n bt k, hng s trong gi hn ca biu thc cui cng nm trong


khong 21n v 21n . V
1
=0
lim
n 2n

.c

suy ra gii hn trong biu thc cui l bng 0. Do vy g(x + y) =


g(x) + g(y).

Nguyn Hu in

OLYMPIC TON NM 2000

.c

(Tp 3)

33 THI V LI GII

NH XUT BN GIO DC

.c

Li ni u
th gi lnh lamdethi.sty ti bin son mt s ton thi Olympic, m
cc hc tr ca ti lm bi tp khi hc tp LATEX. ph v cc bn ham

hc ton ti thu thp v gom li thnh cc sch in t, cc bn c th tham


kho. Mi tp ti s gom khong 30 bi vi li gii. Tp ny c s ng gp
ca Nguyn Vn Hu, L Th Thu Hin, Nguyn Trung Hiu, Nguyn Th Mai

Hoa, Nguyn Vn Huy, Nguyn Thng Huyn

.c

Rt nhiu bi ton dch khng c chun, nhiu im khng hon ton

chnh xc vy mong bn c t ngm ngh v tm hiu ly. Nhng y l ngun


ti liu ting Vit v ch ny, ti c xem qua v ngi dch l chuyn v

ngnh Ton ph thng. Bn c th tham kho li trong [1].

H Ni, ngy 2 thng 1 nm 2010


Nguyn Hu in

Rt nhiu on v mi hc TeX nn cu trc v b tr cn xu, ti khng


c thi gian sa li, mong cc bn thng cm.

51
89/176-05
GD-05

M s: 8I092M5

Mc lc
3

Mc lc . . . . . . . . . . . . . . . . . . . . . . . . . . . . . . . . . . . . . . . . . . . . . . .

Chng 1. thi olympic Hoa K . . . . . . . . . . . . . . . . . . . . . . . . .

Chng 2. thi olympic Vit Nam . . . . . . . . . . . . . . . . . . . . . .

11

Chng 3. thi olympic Chu Thi Bnh Dng . . . . . .


Chng 4. thi olympic o - Balan . . . . . . . . . . . . . . . . . . . . .

26

20

Chng 5. thi olympic a Trung Hi . . . . . . . . . . . . . . . . .

30

Chng 6. thi olympic Petecbua . . . . . . . . . . . . . . . . . . . . . . .

34

Chng 7. thi olympic Anh . . . . . . . . . . . . . . . . . . . . . . . . . . . .

40

.c

Li ni u . . . . . . . . . . . . . . . . . . . . . . . . . . . . . . . . . . . . . . . . . . . .

Ti liu tham kho. . . . . . . . . . . . . . . . . . . . . . . . . . . . . . . . . . . .

42

Chng 1

thi olympic Hoa K

.1.1. Mt b bi c R qun , W qun trng v B qun xanh. Mt ngi chi


thc hin vic rt cc qun bi ra khi b bi. Vi mi lt, anh ta ch

.c

c php rt ng 1 l bi, v phi chu mt s tin pht cho lt rt


bi : - Nu l bi c rt c mu xanh, tin pht bng s qun trng

cn li trong b bi - Nu l bi c rt c mu trng, tin pht bng


hai ln s qun cn li trong b bi - Nu l bi c rt c mu ,

tin pht bng ba ln s qun xanh cn li trong b bi Hy xc nh


tng s tin pht ti thiu m ngi chi phi tr (ph thuc vo R, W,

B) v tm tt c cc cch chi c th t c s tin pht

Li gii: Ta s chng minh s tin pht ti thiu phi tr l min(BW,2


WR,3RB) D nhin s tin pht ny l t c, tng ng vi 1 trong
3 cch rt bi sau:(bb..bbrr..rr ww.. ww);(rr..rr ww.. wbb. . . )( ww..
wwbb..rr)Vi mi mt cch rt bi, ta nh ngha chui xanh l mt
on lin tip cc qun bi mu xanh c rt ra khi b bi (tc trong
mt s lt lien tip, ta ch rt qun xanh ra). Tng t, ta c nh
ngha chui , chui trng.
By gi ta s chng minh 3 b :
B 1: Vi mi cch rt bi cho trc, ta c th thc hin 1 cch
rt bi khc, trong 2 chui cng mu c gp vo nhau m khng
lm tng s tin pht

Nguyn Hu in, HKHTN H Ni

Ta s chng minh trong trng hp gp 2 chui , cc trng hp khc


hon ton tng t. Gi s gia 2 chui c w qun trng v b qun
xanh. By gi, nu ta chuyn mt qun t chui th nht sang chui
th 2, s tin pht s tng them 2w 3b (do mi lt rt qun trng
phi tng them tin pht l 2 bi s xut hin ca 1 qun mi, v
qun c chuyn i nm sau b qun xanh nn khng phi chu 3b
tin pht).
Nu , ta ch vic chuyn tt c cc qun t chui 1 sang chui 2.
Ngc li, ta s chuyn tt c cc qun t chui 2 sang chui 1.
Trong c 2 trng hp, 2 chui c gp vo nhau v s tin pht
khng b tng thm.

B 2: Cch chi ti u khng tn ti chui (tc khng xy ra trng


hp rt 1 qun ngay sau 1 qun trng)

iu ny l hin nhin, v nu xut hin lt rt bi nh vy ta thay


i chui bng chui , ta thu c 1 cch chi mi c s tin pht nh

.c

hn
B 3: Cch chi ti u (t tin pht nht) s c t hn 5 chui
Gi s tn ti 1 cch chi ti u c 5 chui tr ln. Gi thit rng qun

bi u tin c rt c mu (cc trng hp khc chng minh tng


t). By gi, ta gi s rng cc chui c rt c gi tr (theo th t,

chng minh tng t trong cc trng hp khc).


Theo b 1, ta c th gp 2 chui , hoc 2 chui trng li vi nhau

m khng lm tng s tin pht. Nhng cch chi hin ti l ti u, do


vy ta phi c: (1)
Gp 2 chui trng li v ta c 1 cch rt bi cng c s tin pht ti
thiu: . Theo b 1, ta c th gp 2 chui vi nhau m s tin pht
khng tng them. Nhng do cch chi ny l ti u, nn ta phi c ,
mu thun vi (1). Vy iu gi s l sai, b 2 c chng minh.
Tc l bt k cch chi ti u no cng ch c ti a 4 chui. Kt hp
vi b 2, cch chi ti u nu phi rt qun u tin l
Cch chi ny l ti u khi v ch khi v , Tng t, ta cng c cch
chi ging nh trn nu qun u tin uc rt l qun trng hoc qun
xanh./.

thi olympic Hoa K

c cc s thc m phng trnh




x2 2mx 4(m2 + 1) x2 4x 2m(m2 + 1) = 0

c ng ba nghim phn bit.

Li gii: p n: m = 3.
Cho hai tha s v tri ca phng trnh bng 0 ta nhn c hai
phng trnh a thc. t nht mt trong cc phng trnh ny phi
nghim ng vi gi tr x no x l nghim ca phng trnh ban
u. Nhng phng trnh ny c th vit di dng (x m)2 = 5m2 + 4

(1)v (x 2)2 = 2(m3 + m + 2) (2). Ta c ba trng hp m phng


trnh ban u c th c 3 nghim phn bit: Phng trnh (1) c nghim

kp hoc phng trnh (2) c nghim kp hoc hai phng trnh c mt


nghim chung. Tuy nhin, trng hp th nht khng xy ra v hin

.c

nhin 5m2 + 4 = 0 khng th tha mn vi mi gi tr thc m.


Trong trng hp th hai, ta phi c 2(m3 + m + 2) = 0; m3 + m + 2
phn tch thnh (m + 1)(m2 m + 2) v tha s th hai lun dng vi

mi gi tr thc m. V vy ta phi c m=-1 trng hp ny xy ra.


Khi nghim duy nht ca phng trnh ny l x=2 v phng trnh

(1) tr thnh (x + 1)2 = 9, tc l x=2, -4. Nhng iu ny c ngha l


phng trnh ban u ca ta ch c nghim l 2 v -4, tri vi yu cu

ca bi ton.
Xt trng hp th ba, gi r l nghim ca phng trnh th xr l mt

tha s ca c hai biu thc x2 2mx4(m2 +1) v x2 4x2m(m2 +1).


Tr hai biu thc ny cho nhau ta nhn c x r l mt tha s ca
(2m 4)x (2m3 4m2 + 2m 4), hay (2m 4)r = (2m 4)(m2 + 1). V

vy m = 2 hoc r = m2 +1. Tuy nhin, trong trng hp th nht th c


hai phng trnh bc hai ca ta tr thnh (x 2)2 = 24, v v vy, ta ch

thu c hai nghim phn bit. Vy ta phi c r = m2 + 1. Khi , thay


vo ng thc (r2)2 = 2(m3 +m+2), ta c (m2 1)2 = 2(m3 +m+2)

hay (m + 1)(m 3)(m2 + 1) = 0. Do m = 1 hoc 3. Trng hp


m=-1 c ch ra khng tha mn. V vy, ta ch c m=3. Khi
cc phng trnh ca ta tr thnh (x 3)2 = 49 v (x 2)2 = 64, chng

c cc nghim l x=-6, -4, 10, tha mn yu cu ca bi ton.

Nguyn Hu in, HKHTN H Ni

.1.2. Cho ABC l tam gic u c din tch bng 7. Gi M, N tng ng l


cc im trn cnh AB, AC sao cho AN=BM. Gi O l giao im ca
BN v CM. Bit tam gic BOC c din tch bng 2.
B
hoc bng 13 hoc bng 32 .
(a) Chng minh rng M
AB
[
(b) Tnh gc AOB.
Li gii: (a) Ly im L trn BC sao cho CL=AN v gi P, Q ln lt l
giao im ca CM v AL, AL v BN. Php quay vi gc quay 120o quanh
tm ca tam gic ABC bin A thnh B, B thnh C, C thnh A; php
quay ny cng bin M thnh L, L thnh N, N thnh M v bin O thnh
P, P thnh Q, Q thnh O. Do OPQ v MLN l cc tam gic u ng
\ 2 . V vy, O nm trn
tm vi tam gic ABC. Suy ra \
BOC=-MOC=
3

ng trn i xng vi ng trn ngoi tip tam gic ABC qua BC. C

nhiu nht hai im O trn ng trn ny v nm trong tam gic ABC


t l khong cch t O ti BC v t A ti BC bng 27 , t l ny cng

.c

l t l din tch ca cc tam gic OBC v ABC. V vy ta ch ra rng


MB 1
= 3 hoc 23 tng ng vi cc v tr ca im O, v khng c t l no
AB
B
= 13
khc (tc l khng c hai im M cho cng mt im O. Nu M
AB

= 31 , p dng nh l Menelaus cho tam gic ABN v ng thng


th AN
AC
[BOC]
BO
= 34 , do [BN
= BO = 37 . Suy ra [BOC]
= 3 CN = 72 v ta
CM, ta c ON
C] BN
[ABC] 7 CA

B 2
= 3 , theo nh l Menelaus
c iu phi chng minh. Tng t, nu M
AB
[BOC]
[BOC]
BO
B
BO
= 67 , do [BN C] = BN
= 67 . Suy ra [ABC] = 67 CN
= 72 . (b) M
= 31 th
ta c BN
CA
AB
pi v O
= P[
MONA l mt t gic ni tip do A=
OQ = 2
. Do
3
3
MB

[
\
\
\
[
\
= 1 v
AOB=AOM+MOB=ANM +P OQ = ANM + . Nhng
3

AN
= 31
AC

AB

nn d dng thy c N l hnh chiu ca M trn AC. V vy


[ = 5 . Lp lun tng t i vi trng hp cn li,
\
ANM = 2 v AOB
6

[
\
ta c ANM = 6 v AOB = 2 .
.1.3. Cho f (x) = x2 2ax a2 43 . Tm tt c cc gi tr ca a |f (x)| 1
vi mi x [0; 1].

Li gii: p n: 21 a 42 .
th ca f(x) l mt parabol c im cc tiu (c ngha l h s a
m) v nh l (a;f(a)). T f (0) = a2 43 ta c |a| 21 f (0) 1.

Gi s a 0 th parabol ca ta tng nghim ngt trong khong t 0

thi olympic Hoa K

n 1, do f (1) 1. Nhng ta c 12 a + 1 1, 41 (a + 1)2 1,


1
54 (a + 1)2 1. T 45 (a + 1)2 = f (1), ta c f tha mn iu kin
4

ca bi ra khi 21 a 0. Vi a > 0, f gim vi 0 x a v tng


vi a x 1. V vy ta cn ch ra gi tr nh nht ca f(a) nm trong

phm vi theo yu cu ca bi ton, tc l f(1) nm trong gii hn ny.


T a 12 ta c 1 < (a + 1)2 49 v v vy f (x) = 1 54 (a + 1)2 < 14 .

Mt khc, f (a) = 2a2 43 nn ta phi c a 42 f (a) 1. Ngc


li, nh gi f(0), f(a), f(1) ta ch ra c f tha mn iu kin ca bi

ra khi 0 < a 42 .

.1.4. K hiu u(k) l c l ln nht ca s t nhin k. Chng minh rng


n

2
1 X u(k)
2

.
n
2 k=1 k
3

.c

Li gii: t v(k) l c ln nht ca k c dng ly tha ca 2, nn


u(k)v(k) = k. Trong {1, 2, ..., 2n} c 2ni1 gi tr ca k sao cho v(k) = 2i

vi i n 1, v mt gi tr sao cho v(k) = 2n . Do , v tri bng


n

2
n1 ni1
X
2
1
1 X u(k)

= n+
.
n
2 k=1 k
4
2n+i
i=0

T tng ca chui hnh hc ta c


n

2
1 X u(k)
2
2

= 4n + (1 4n ) > .
n
2 k=1 k
3
3

.1.5. Tm tt c cc s thc tha mn h


x3 = 2y 1
y 3 = 2z 1

z 3 = 2x 1.

Li gii: Trc ht ta ch ra rng x = y = z. Gi s tri li rng x 6= y.


Nu x > y, th y =

(x3 +1)
2

>

(y 3 +1)
2

= z, nn y > z, v tng t z > x,

Nguyn Hu in, HKHTN H Ni

10

mu thun. Tng t, nu x < y th y < z v z < x, mu thun. Nn


cc nghim ca h phng trnh c dng x = y = z = t vi t l nghim
ca phng trnh t3 = 2t 1. Vy, nghim ca h phng trnh l



1 + 5 1 5
x = y = z = t, t 1,
.
,
2
2
.1.6. Tm s t nhin a nh nht phng trnh sau c mt nghim thc:
 x 
3x
+2=0
cos
+
cos2 (a x) 2 cos (a x) + cos
2a
2a 3
Li gii: Li gii: Gi tr nh nht ca a l 6. Phng trnh tha mn

.c

khi a=6, x=8. chng minh a l gi tr nh nht, ta vit phng trnh


di dng




 x 
3x
2
cos
+1 =0
+
(cos(a-x)-1) + cos
2a
2a
3

Do c hai s hng v tri u khng m nn ng thc xy ra


th chng phi cng bng 0. T cos(a-x)-1 = 0 ta c x phi l mt
s nguyn ng d vi a trong php chia cho 2. T s hng th hai

bng 0, ta thy cc gi tr cosin phi nhn gi tr bng 1 v -1. Nu




cos x
+
= 1 th x
+ 3 = 2k vi gi tr k nguyn v nhn hai v
2a
3
2a

ta c 3x 4a (mod12a). Khi th nu cos x
+ 3 = 1 th
vi 6a

2a
x
+ 3 = (2k + 1) v nhn hai v vi 6a
ta c 3x 4a (mod12a).
2a

Trong c hai trng hp ta u c 3x chia ht cho 2, v vy x phi chia


ht cho 2 v a cng phi tha mn iu . Hn na, c hai trng
hp ta cng u c -2a v 4a cng phi chia ht cho 3, v th a phi chia

ht cho 3. Tm li ta c 6 phi l c ca a v a=6 l gi tr nh nht


cn tm.

Chng 2

.c

thi olympic Vit Nam

l bng nhau.

.2.7. Trn mt phng cho hai ng trn 1 , 1 theo th t c tm l O1 v


0
0
0
O2 . Cho M1 v M2 l hai im ln lt nm trn 1 , 1 sao cho O1 M1
0
v O2 M2 ct nhau. Cho M1 v M2 ln lt l hai im trn 1 , 1 sao
0
0
\
cho khi quay theo chiu kim ng h s o ca gc M\
1 OM1 v M2 OM2

(a) Xc nh qu tch trung im ca on thng M1 M2


0
0
(b) Gi P l giao im ca cc ng thng O1 M1 v O2 M2 . ng
trn ngoi tip M1 P M2 ct ng trn ngoi tip O1 P O2 ti P v mt
im khc l Q.Chng minh rng Q l im c nh khng ph thuc
vo v tr ca M1 v M2 .

Li gii:

Nguyn Hu in, HKHTN H Ni

12
Q
0

M1
M0
O1

M 02
M2

O
M

M1

O2

(a)Chng ta dng cc s phc, mi im c k hiu l ch in hoa ta t


tng ng vi mt s phc c k hiu l ch in thng. Gi M 0 , M v

.c

O ln lt l trung im ca cc on thng: M1 M2 , M1 M2 v O1 O2 .
m2 o2
1 o1
Ta cng t z = m
= m
sao cho php nhn bi z l php quay
0 o
m0 o1
2
1
1
m1 + m2
= (o1 + z(m01 o1 )) + (o2 + z(m02 o2 )) = o + z(m0 o)
2
2
2

m=

quanh 1 im qua mt s gc. Khi :

t suy ra qu tch im M l ng trn tm O bn knh OM 0 .


(b)Chng ta s dng trc tip cc gc c modun . Ch rng:

QM1 M2 = QP M2 = QP O2 = QO1 O2

Tng t: QM2 M1 = QO2 O1 , suy ra tam gic QM1 M2 ng dng


vi tam gic QO1 O2 . Do :
q o1
q m1
=
q o2
q m2
Hay tng ng:

q o1
(q m1 ) (q o1 )
o1 m1
o1 m01
=
=
=
q o2
(q m2 ) (q o2 )
o2 m2
o2 m02

V hai ng thng O1 M1 vO2 M2 ct nhau, o1 m01 6= o2 m02 v ta


c th gii phng trnh ny tm c gi tr duy nht ca q, suy ra
0

Q l im c nh khng ph thuc vo v tr ca M1 v M2 .

thi olympic Vit Nam

13

.2.8. Gi s rng tt c ng trn ngoi tip ca bn mt ca mt t din


c bn knh bng nhau. Hy ch ra rng hai cnh i bt k ca mt t
din l bng nhau.
Li gii: Trc ht ta chng minh rng vi bn im khng ng phng
bt k X, Y , Z, W ta c:
XY Z + Y ZW + ZW X + W XY < 2

.c

Z
Tht vy: p dng bt ng thc tam gic cho cc gc ta thy rng:

XY Z + Y ZW + ZW X + W XY
< (ZY W +W Y X)+Y ZW +(XW Y +Y W Z)+W XY

= (ZY W +Y W Z+Y ZW )+(XW Y +W Y X+W XY )


= + = 2.

Gi R l bn knh chung ca cc ng trn ngoi tip bn mt t din.


Ta nh ngha rng hai gc ca t din ABCD gi l i din vi cng
mt cnh l hai gc v d nh ABCv ADC, ta c:
sinABC =

AC
= sinADC
2R

( do nh l hm sin).
Do hai gc bt k i din vi cng mt cnh hoc bng nhau hoc
b nhau.
Hn na, rng nu XZ v Y W l hai cnh i ca t din XYZW
th suy ra (XY Z + ZW X + (Y ZW W XY < 2)) nn khng
th c trng hp hai gc i din vi cng mt cnh XZ v vi cnh

Nguyn Hu in, HKHTN H Ni

14

Y W u l b nhau. Ni cch khc, nu cc gc i din ca cnh


XZ l b nhau th hai gc i din vi cng mt cnh Y W l bng nhau.

B
D

.c

By gi ta gi s ngc li rng c hai gc i din vi cng mt cnh

v d nh ABCv CDA l b nhau cn tt c cc cp gc i din


vi cng mt cnh khc l bng nhau, ta c:

BCD + DAB = ( CDB DBC) + ( ADB DBA)

= ( pi CAB DAC) + ( ACB DCA)

= ( CAB ACB) + ( DAC DCA)

= ABC + CDA
=

iu ny tri vi gi s. Do bn cnh cc gc i din vi cnh AC


th cn mt s cp gc i din vi cng mt cnh khc l b nhau.Theo
phn l lun trn th cp gc i din vi cnh BD khng th b nhau
nn chng phi bng nhau. Do cp gc i din vi cng mt cnh
trong s cc cnh AB, AD, CB v CD l b nhau. Khng gim tnh
tng qut, gi s cp gc i din vi cnh AB l b nhau suy ra cp

thi olympic Vit Nam

15

gc i din vi cnh CD l bng nhau. Hn na:


CDB = DCB DBC
= DAB DAC
= ( ABD ADB) ( ACD ADC)
= ABD + ACD + ADB + ADC
= ABD + ACD + ( ACB) + ( ABC)
= ABD + ACD + ( ACB ABC)
= ABD + ACD + CAB

Suy ra CDB < CAB. V cc CDB v CAB l khng


bng nhau nn chng phi b nhau. By gi ta li c cc gc

.c

ADB, BDC v CDA l cc gc nh ca mt t din nn


(ADB + BDC) + CDA < ADC + CDA < 2. Nhng:

ADB + BDC + CDA = ( ACB) + ( BCA) + ( CBA)

= 3

= 2.

iu ny v l. Do cc gc i din vi cng mt cnh ca t din l


bng nhau. Nh chng ta l lun trn th trong trng hp ny ta c:
BCD + DAB = ABC + CDA
Suy ra 2DAB = 2ABC hay DAB = ABC.
Cho nn, DB = 2RsinDAB = 2RsinABC = AC. Tng t ta cng
c: DA = BC, DC = BA.
l iu phi chng minh.
.2.9.

Cho hai ng trn (C1 ), (C2 ) ct nhau ti P v Q. Tip tuyn


chung ca hai ng trn( gn P hn Q) tip xc vi (C1 ), (C2 ) ln
lt ti A v B. Tip tuyn ca(C1 ) ti P ct (C2 ) ti E ( khc P ),
tip tuyn ca (C2 ) ti P ct (C1 ) ti F ( khc P ). Gi H, K ln lt l

Nguyn Hu in, HKHTN H Ni

16

hai im nm trn hai tia AF v BE sao cho AH = AP , BK = BP .


Chng minh rng nm im A, H, Q, K, B nm trn cng mt ng
trn.
Li gii:
B
T

R
P
H

Q
E
F
V iu kin c tnh cht i xng nn ta ch cn chng minh ABKQ

.c

l ni tip ng trn.
Chng ta s dng gc c hng modulo . Gi R l giao im ca

ng thng AP v tia BE. ng thng AB v BE ct nhau ti T .


S dng tnh cht ca tip tuyn v t gic ni tip ta c:

QAR = QAP = QP C = QBC = QBR

Nn t gic ABRQ l ni tip ng trn. Chng ta s chng minh

rng K = R. Tht vy:


S dng gc ngoi tam gic ABP v CP R; tip tuyn AB v

P T ; t gic ni tip ng trn ta c:

BP R = BAP + P BA = AQP + P QB
= AP T + P EB = RP E + P ER
= P RB
Do tam gic BP R l tam gic cn vi BP = P R suy ra R = K.
l iu chng ta cn chng minh.
.2.10.Cho a, b, c l cc s nguyn dng i mt ngyn t cng nhau. Mt
s nguyn n c gi l stubborn nu n khng c biu din di

thi olympic Vit Nam

17

dng:
n = bcx + cay + abz
vi x, y, z l cc s ngyun dng bt k. Hy xc nh qua hm s ca
a, b v c s lng cc s nguyn stubborn .
Li gii: Chng ta c th khng nh c rng bt k s ngyun n no
u c th biu din di dng n = bcx + cay + abz trong x, y, z l
cc s nguyn v 0 < y b; 0 < z c cn x c th m. Tht vy,v a
v bc nguyn t cng nhau nn ta c th vit: n = an0 + bcx0 vi n0 , x0
l cc s nguyn. v b v c l nguyn t cng nhau nn n0 = cy0 + bz0 vi
y0 , z0 l cc s nguyn. Do n = bcx0 = cay0 + abz0 . Chn s nguyn
, sao cho 0 < y0 + b b v 0 < z0 + c c, khi ta c :

n = bc(x0 a a) + ca(y0 + b) + ab(z0 + c)

.c

y chnh l dng biu din ca n m ta mun.


Ch rng bt k s nguyn dng no nh hn bc + ca + ab u l

stubborn. Mt khc ta cng khng nh rng mi s nguyn n > 2abc


khng l stubborn. Chng han, biu din n = bcx + cay + abz vi cc

s nguyn v 0 < y b, 0 < z c. Khi :

2abc < bcx + cay + abz bcx + cab + abc = bcx + 2abc

Suy ra x > 0 nh vy n khng l stubborn khi n > 2abc.

Tip theo chng ta chng minh rng ng mt na cc s nguyn


dng trong S = [bc + ca + ab; 2abc]. lm c iu ny ta i chng
minh n S l stubborn khi v ch khi f (n) = (2abc + bc + ca + ab) n

khng l stubborn.
iu kin cn: Gi s rng n l stubborn v biu din f (n) =

bcx + cay + abz vi 0 < y b, 0 < z c. Nu x khng dng th chng


ta c th vit n = bc(1 x) + ca(b + 1 y) + ab(c + 1 z), vi 1 x0 ,

b + 1 y0 v c + 1 z0 l cc s nguyn dng, nhng iu ny l khng


th v n l stubborn. Do , x > 0 v f (n) khng l stubborn.
iu kin : Gi s ngc li f (n) khng l stubborn v n
cng khng l stubborn. Biu din f (n) = bcx0 + cay0 + abz0 v n =

Nguyn Hu in, HKHTN H Ni

18

bcx1 + cay1 + abz1 vi xi , yi, zi l cc s nguyn dng. Khi :


2abc = bc(x0 + x1 1) + ca(y0 + y1 1) + ab(z0 + z1 1)
t x = x0 + x1 1 v cng ty,z tng t. T ng thc trn chng

t 0 bcx( mod a). V bc nguyn t cng nhau vi a nn x phi chia ht


cho a suy ra x a.
Tng t y b, z c, khi 2abc = bcx + cay + abz 3abc ( v l).

Tm li: c bc+ca+ab1 cc s nguyn dng nh hn bc+ca+ab


l stubborn, mi s nguyn ln hn 2abc khng l stubborn, v mt
na ca 2abc(bc+ca+ab)+1 cc s nguyn dng cn li l stubborn.
Kt qu ca bi l tng ca:
2abc + bc + ca + ab 1
2abc (bc + ca + ab) + 1
=
2
2

Gi R+ l tp cc s thc dng v a, r > 1 l cc s thc. Gi

.c

.2.11.

cc s nguyn dng stubborn.

bc + ca + ab 1 +

s rng f : R+ R l mt hm s sao cho: (f (x))2 axr f ( xa ) vi mi


x > 0.

1
(a) Nu f (x) < 22000 vi mi x < 22000
, chng minh rng f (x)
r 1r
xa
vi mi x > 0.
(b) Xy dng mt hm f : R+ R ( khng cn tho mn iu

kin trong cu (a)) sao chof (x) > xr a1r vi mi x > 0.

Li gii: Ch rng ta c th vit li bt ng thc di dng:


2

f (x/a)
f (x)

xr a1r
(x/a)r a1r
Gi s ngc li tc l tn ti x0 sao cho f (x0 ) > xo r a1r . t xn =

(*)
x0
an

n)
2
v n = xfr(x
1r vi n 0, suy ra 0 > 1. T (*) ta c:n+1 n vi
na
n
n 0, v bng quy np ta chng minh c rng: n 20 vi n 0.
n
Chng ta s s dng iu ny ngay sau y, rng mi n 20 l

mt s dng v do f (xn ) cng l s dng. Chng ta gn x = xn


vo bt ng thc v sp xp cc bt ng thc li ta c:
n xnr a1r
n
f (xn )
f (xn+1 )
=
=

f (xn )
axrn
axrn
ar

thi olympic Vit Nam

19

Vi mi n 0.
n
Ta thy lun tn ti N sao cho 2ar < 20 n vi mi n > N.

)
2 hay tng ng (
Vi tt c cc gi tr n nh vy th ta c: f f(x(xn+1
n)
do f (xn ) l dng) f (xn+1 ) 2f (xn ). Cho nn f (x) 22000 ( vi n

1
ln), nhng cng vi n ln th ta li c xn = axn0 < 22000
. iu ny tri
r 1r
vi gi thit tc iu gi s l sai. Do f (x) x a
vi mi x > 0.

(b) Vi mi s thc xlun tn ti duy nht mt gi tr x0 (1; a]


n
sao cho xx0 = an vi n l s nguyn. Gi (x) = x20 v t f (x) =
(x)xr a1r . Bng php co li, chng ta cng c (x)2 = (x/a) vi mi
x, ni cch khc (*) cng ng vi mi x. Ta cng c (x) > 1 vi mi
x hay ni cch khc f (x) > xr a1r vi mi x > 0.

.c

l iu cn chng minh.

Chng 3
thi olympic Chu Thi

100
P

i=0

x3i
13xi +3x2i

.c

.3.12.Tnh tng S =

Bnh Dng
vi xi =

i
100


i = 1, 101

Li gii: V 1 3x + 3x2 = x3 (x 1)3 6= 0 x


3

x
x
Ta c th t f (x) = 13x+3x
x
2 = x3 +(1x)3
Cho x = xi , x = 1 xi = x101i v thm 2 phng trnh h qu ta tm
ra:f (xi ) = f (x101i ) = 1. V th

101
X

S=

i=0

f (xi ) =

50
X

(f (xi ) + f (x101i )) = 51

i=0

.3.13.Cho mt s b tr vng trn quanh ba cnh mt tam gic, mt vng


mi gc, hai vng mi cnh, mi s t 1 n 9 c vit vo mt trong
nhng vng trn ny sao cho
i. Tng ca 4 s mi cnh tam gic l bng nhau.
ii. Tng ca bnh phng ca 4 s trn mi cnh ca tam gic l bng
nhau.
Tm tt c cc cch tho mn yu cu ny.
Li gii: Ly bt k mt s b tr cc con s, gi x, y, z l s trong
gc v S1 , S2 ln lt l tng ca bn s, tng ca bnh phng bn s

thi olympic Chu Thi Bnh Dng

21

trn mt cnh bt k. Do iu kin cho ta c:


3S1 = x + y + z +

9
X

k = x + y + z + 45

k=1

3S2 = x + y + z +

9
X

k 2 = x2 + y 2 + z 2 + 285

k=1

T ng thc th hai ta suy ra x, y, z hoc tt c chia ht cho 3 hoc


khng c s no chia ht cho 3. Bi nguyn l Pigeouhole c hai s l
ng d mod3. Ly phng trnh th nht theo mod3 ta cng suy ra
3| (x + y + z). Do x y z(mod3)

Nu (x, y, z) = (3, 6, 9) hay (1, 4, 7) th S2 = 137 hoc 17. Nu S2 = 137


th S2 1(mod3) suy ra ch c mt s trn ba cnh l l. iu ny

khng th v 5 > 3 s l c vit trong mi khe


V th, (x, y, z) = (2, 5, 8) v S2 = 126. V 92 + 82 > 126 nn 9 khng

.c

th nm cng cnh vi 8, tc l n nm trn cnh cha 2 hoc 5. V



Min 72 + 92 , 72 + 52 + 82 > 126

nn s 7 phi nm trn cnh cha s 2 hoc 8. Nh vy 4 ln cc s trn


3 cnh phi l (2, 4, 9, 5); (5, 1, 6, 8); (8, 7, 3, 2) cho tng bnh phng

cc s trn mi cnh l 126. Cui cng, ta thy cc b s trn u tho


mn.

.3.14.Cho tam gic ABC, trung tuyn AM v phn gic AN . V ng


vung gc qua N ct MA, BA ti P, Q. Gi O l im m ng vung
gc qua P vi BA ct AN. Chng minh QOBC
Li gii: Cch 1. Nu AB = AC th QO l trung trc ca BC v yu
cu phi chng minh c tho mn. Gi s AB 6= AC, s dng to

Castesian:A(0; 0), N(1; 0). t dc ca AB l m th ca AC l 1.


Vit B = (b; mb), C = (c; mc), b 6= c v c b, c l dng. dc

ca BC l m(b+c)
.
bc
V P NAN v x-to ca P l 1, v P thuc ng thng AB(ng
thng y = mx) nn ta c P = (1, m). Do phng trnh ca OP

l y = (x 1)/m + m, suy ra x interceptO l (m2 + 1, 0), M l

Nguyn Hu in, HKHTN H Ni

22

((b + c)/2; m(b c)/2) v n l trung im ca BC. Do phng trnh


x. Bi v Q l giao im ca AM v
ca ng thng AM l y = m(bc)
b+c


m(bc)
bc
b+c
.
V
th

dc
ca
P
Q
l
= m(b+c)
. M
P N, ta c Q 1; m(bc)
b+c
m2 +11

1 l dc ca BC nn QOBC.
Cch 2. Gi , , l s o cc gc CAB, ABC, BCA v y =

BAM, z = MAC, x = MAN = |yz|


2
Nu = th QO l trung trc ca BC. Ngc li, gi s 6= th

y 6= z , vi cch lm ny, ta bin i phng trnh bng cch nhn v


chia vi biu thc lng gic khc 0 v 6= , y 6= z; , , y, z (0; )
S dng quan h lng gic vo cc ABC, BNO, ONQ ta c




 
tan OQN. tan QAN = ON/QN QN/AN = ON/BN BN/AN = tan OBN

Suy ra rng BAN = /2, OBN = /2, QAN = x


Do : tan OQN.tanx = tan (/2) tan (/2) ()

.c

p dng lut hm sin cho ABC, ACM, ta c:

sin y+sin z
sin ysin z

sin sin
.
sin +sin

Suy ra:

sin .BM/AM
sin y
sin
=
=
sin z
sin
sin .CM/AM
Cho u, v trong khong (0; /2), ch rng

sin(u + v)cos(u v)
sin(2u) + sin(2v)
tan(u + v)
=
=
tan(u v)
sin(u v)cos(u + v)
sin(2u) sin(2v)


Cho (u, v) = (y/2, z/2) ; (u, v) = /2, /2 trong ng thc ny ta tm
ra:



tan /2 + /2
tan (/2)
tan (y/2 + z/2)
cot (/2)




=
=
=
tan (y/2 z/2)
tan (y/2 z/2)

tan /2 /2
tan /2 /2
Nu > th x = y/2
so snh biu
 z/2 
 thc cui cng vi
(*), ta c: tan OQN = tan /2 /2 /2 . ta c: OQN, /2


/ /
(0; /2) v t 7 tan t l n nh, v th OQN =
2  2 
/ / /
2
2
2 OQAB .
Chng minh tng t nu >

thi olympic Chu Thi Bnh Dng

23

.3.15.Cho n, k l cc s nguyn dng, n > k. Chng minh rng:


1
nn
nn
n!
.
<
<
n + 1 k k (n k)nk
k! (n k)!
k k (n k)nk
Li gii: S dng khaitrinnh thc,ta vit: nn = (k + (n k))n
n
n
P
k m (n k)nm > 0 vi mi m
dng
am , vi am =
m=0
m
Bt ng thc cho tng ng vi:

nn
< ak < nn
n+1
n
P
Bt ng thc bn phi tho mn do nn =
am > ak . chng minh
m=0

m=0

k=0

.c

bt ng thc cn li ta ch cn ch ra ak > a0 , .....am1 , am+1 , .....am .


Bi v
n
n
X
X
n
ak = (n + 1) ak
n =
am <

k m (n k)nm

am
=
am+1

Tht vy,
minh
ta chng

rng amtng vi m k, gim vi m k, ch


n
n
= m+1
. V th:
rng
nm
m
m+1

m+1

k m+1 (n k)nm1

nk m+1
.
nm k

Biu thc ny nh hn 1 khi m < k v ln hn hay bng 1 khi m k.

Do ta c iu phi chng minh.

.3.16.Cho mt hon v (a0 , a1 , .....an ) ca dy 0, 1, ....., n. Mt chuyn v ca


ai , aj gi l hp php nu ai = 0, i > 0 v ai1 + 1 = aj . Hon v
(a0 , a1 , .....an ) gi l chnh quy nu sau hu hn cc bc chuyn v hp
php n tr thnh (1, 2, ...., n, 0). Vi n no th (1, n, n 1, .....3, 2, 0) l

chnh quy ?.

Nguyn Hu in, HKHTN H Ni

24

Li gii: Vi n c nh, 0 v 1 l ci hon v (1, n, n 1, .....3, 2, 0)


v (1, 2, .....n, 0). Ta ni 0 l chm dt trong hon v 1, nu sau mt
s chuyn v hp php ca 0 ta thu c 1, v nu khng mt chuyn
v hp php no c th p dng cho chuyn v 1, . V khng c chuyn
v hp php no c th c p dng cho 1 , nu 0 chnh quy th n
chm dt trong 1 .Nh khi p dng chuyn v hp php cho 0 nhiu
nht mt chuyn v hp php c th p dng cho mi hon v thu c.
V th 0 chm dt trong nhiu nht mt hon v.
Nu n = 1, 2, d dng kim tra (1, n, n 1, .....3, 2, 0) l chnh quy. Nu
n > 2, chn, ta i hi rng 0 khng chm dt trong 1 v v th khng


p dng k chuyn v hp php cho 0 suy
chnh quy. Cho k 0; n2
2

ra hon v m bt u vi 1, n, n 1, ....2k + 2, 0. Do 0 chm dt


chuyn v hp
trong mi hon v bt u bi 1, n, 0 thu c sau n2
2

php.
By gi ta gi s rng n > 2, xt trng hp ny ta s a ra vi k

.c

hiu. Vi mi s nguyn s > 0, t 0 sao cho s + t chia ht cho n + 1,


ta xy dng php hon v gi l(s, t) _bc thang ti mt thi im nh
sau: p dng (1) mt ln v lp li (2) v (3) mt cch xen k:

(1) Cho s s u tin 1, 2, ....s 1, 0


(2) Cho t s tip theo l t s ln nht trong 1, 2, ..., n cha phn nh

vo mc no, sp xp theo trt t tng.


(3) Cho s s tip theo l s s ln nht trong 1, 2, ..., n cha phn nh

vo mc no, sp xp theo trt t tng.


Nu s + 1| n + 1 v t > 0 th p dng n/(s + t) chuyn v hp php cho
(s, t)_bc thang, qu trnh ta m ch nh s la chn bc thang
Tip theo ta gi s rng s|(n + 1). Nu 2s khng chia ht cho n + 1 th
p dng n/(s 2) chuyn v hp php cho (s, 0)_bc thang suy ra mt
php chuyn v khc 1 m khng c thm chuyn v hp php no c
th c p dng. Nu thay v 2s|(n + 1) th (s, 0)_bc thang thc s
l (s, s)_bc thang ci m c th chn t (2s, 0)_bc thang
Gi ta chng minh rng nu n > 2 v n l th 0 chnh quy nu v ch
nu n + 1 l lu tha ca 2. V n + 1 chn nn ta vit n + 1 = 2q r,
q l s t nhin v r l s t nhin l. p dng (n 1)/2 chuyn v hp

thi olympic Chu Thi Bnh Dng

25

php cho 0 dn n (2, 0)_bc thang


Nu 2q > 2 th 2s|(n + 1) vi s = 21 , ......2q1 ta c th lp li dn
n (2q , 0)_bc thang
Nu r = 1 suy ra ta thu c 1 v 0 chnh quy. Ngc li, p dng
r 2 chuyn v hp php dn n mt hon v 0 l ln cn tri
ca n v th khng c mt chuyn v hp php no l c th. Tuy nhin

.c

hon v cui bt u bi 1, 2, ....2q hn 1, n. Suy ra 0 khng chm dt


trong 1 v th 0 khng chnh quy
Vy 0 l chnh quy khi v ch khi n = 2 hoc n + 1 l lu tha ca 2.

Chng 4

thi olympic o - Balan

.4.17.Tm tt c cc s nguyn dng N sao cho s ch chia ht cho 2 v


5 v N + 25 l s chnh phng.

.c

Li gii: Ta c th biu din N di dng l 2a .5b ,vi a v b l cc

s m nguyn. Vi s nguyn x > 5,ta c x2 = N + 25,iu ny tng


ng vi (x + 5)(x 5) = N. V vy N c biu din bng tch ca

hai s t nhin hn km nhau 10. Ta xt hai trng hp:


Trng hp 1: b = 0. Khi vi 2a = (x + 5)(x 5) ta c x + 5 v x 5

l c ca 2. Nhng khng c hai s l c ca 2 m hai s li hn


km nhau 10, do khng c s tho mn trng hp ny.

Trng hp 2: b 1 .Trong trng hp ny, x2 chia ht cho 5, v vy


x2 cng chia ht cho 25. iu chng t b 2. Ly x = 5y, cho
y > 1 v (y 1)(y + 1) = 2a .5b2 . Ta c y 1 v y + 1 l cc s chn, v

p = 21 .(y 1) v q = 12 .(y +1) l cc s nguyn dng m p.q = 2a2 .5b2 .


V vy p v q bng 2m v 5n vi m, n l cc s nguyn. Ta xt hai trng

hp nh nh sau:
1) 5n 2m = 1; v 5n , 2m 6= 0(mod3),ta c : 2m 1(mod3) v 5n

2(mod3). V vy n l s t nhin v 5n 5(mod8). Ta c 2m (5m 1)


4(mod8),t suy ra m = 2. T ta c N = 2000.
2) 2m 5n = 1. Ta c 2m = 5n + 1 2(mod4) t suy ra m = 1 v

n = 0. T ta c N = 200. V vy, tt c cc s tho mn bi ton l

thi olympic o - Balan

27

N = 200 ; 2000.
.4.18.Tm cc s nguyn n 5,sao cho ta c th s dng mu t nh a gic
n-nh bi 6 mu m 5 nh lin tip nhau c mu khc nhau ?
Li gii: Ta gi cc mu l a, b, c, d, e, f . Biu th S1 bi dy a, b, c, d, e
v S2 bi dy a, b, c, d, e, f . Nu n > 0, ta c th biu din di dng
5x + 6y vi x; y 0, khi n tho mn cc iu kin sau: x lin vi

dy S1 ko theo y lin vi dy S2 trn hnh a gic. Ta c: y c th


bng 0,1,2,3 hoc 4. Khi n c th bng 1 trong cc s c dng 5x ;

5x + 6; 5x + 12; 5x + 18 hoc 5x + 24. Tt c cc s ln hn 4 khng tho


mn cc dng trn l 7,8,9,13,14,19.
Xt tt c cc s n, tr cc trng hp bng 7,8,9,13,14,19. Ta c tn
ti s k sao cho 6k < n < 6(k + 1). Theo nh l Pigeonhole, c t nht

k + 1 nh ca n cnh c mu ging nhau. Gia 2 hoc 3 nh c t nht


4 nh khc na, bi v 5 nh lin tip c mu khc nhau. V vy, c t
nht 5k + 5 nh, v n 5k + 5. Do vy, ngoi tr n = 7, 8, 9, 13, 14, 19,

.c

cc trng hp cn li u tho mn bi ton.


Vy tt c cc s n 5 tr 7,8,9,13,14,19 tho mn bi ton.

.4.19.Trong khng gian 3 chiu, cho hnh lp phng n v cng vi 6 hnh

lp phng n v khc, to thnh 7 hnh lp phng n v vi cc


trng tm l (0, 0, 0), (1, 0, 0), (0, 1, 0), (0, 0, 1). Chng minh hoc

trong.

phn chng(trong khng gian 3 chiu) khng gian c th chia thnh 3


chiu sao cho khng c 2 trong chng c th chia cc im nm bn
Li gii: Ta c th chia khng gian thnh cc khng gian 3 chiu, m
trng tm ca cc hnh lp phng n v trng vi cc li im. Xt
cc im c to (x, y, z). Xt x + 2y + 3z. Ta gi 2 li im gn k
nhau nu v ch nu chng khc nhau duy nht 1 to . Ta xem xt
khng gian 3 chiu cha 7 hnh lp phng n v m trng tm ca
chng ng vi cc s t 0 n 6. Do vy, khng gian c th chia thnh
cc khng gian 3 chiu tho mn bi ton.
.4.20.Trong mt phng, cho tam gic Ao Bo Co . Xt tt c cc tam gic ABC

tho mn cc iu kin sau:(i)Co ; Ao vBo ln lt nm trn AB, BC v

Nguyn Hu in, HKHTN H Ni

28

[ = A\
[
\
[
\
CA.(ii)ABC
o Bo Co , BCA = Bo Co Ao v CAB = Co Ao Bo . Tm qu
tch tm ng trn ngoi tip tam gic ABC.
Li gii: Ta c t nht mt tam gic ABC tn ti, v d ta xt tam
gic Ao Bo Co gia tam gic ABC.
Gi tam gic Ao Bo Co c ng trn ngoi tip w, tm O. Gi s tam
gic ABC tho mn cc iu kin ca bi ton. V A v Ao nm 2
[
pha ca cnh BC, v C\
o Ao Bo = CAB. V vy, ng trn ngoi tip
tm O1 ca tam gic Bo Co A phi i xng vi O qua Bo Co . Tng t
ta xt vi ng trn ngoi tip tm O2 v O3 ca tam gic Ao Co B v
Ao Bo C. Ta c t gic OBo O1 Co l hnh thoi sao cho O1 = bo + co.Tng
t,ta c:O2 = ao + co

Ly M l trung im ca O1 O2 . Xt A(a, 0), Co(co , 0), B(b, 0). Ta c


honh ca im O1 v O2 l 12 (a + co ) v 21 (b + co ). V vy im M

c honh 12 (a + b) + co . V vy, ly im H 0 c honh 21 (a + b) th


c H 0 nm trn AB. Ch rng h0 = m + (m co ) = ao + bo + co . Ta

.c

c H 0 nm trn OG vi G l trng tm Ao Bo Co , m OH 0 = 3OG.


Do vy H 0 l trc tm H ca tam gic Ao Bo Co . Qu tch cn tm l duy

nht mt im l trc tm ca tam gic Ao Bo Co .

.4.21.Cho 27 im phn bit trn mt phng khng c 3 im no thng hng.


4 im trong chng lp thnh cc nh ca hnh vung n v; 23 nh
cn li nm trong hnh vung trn. Chng minh rng tn ti 3 im
1
.
48

ring bit X; Y ; Z sao cho [XY Z]

Li gii: Ta chng minh bng quy np. Ly n 1 im nm trong hnh


vung (khng k 3 ng thng), hnh vung c th chia thnh 2n + 2
tam gic m cc nh ca cc tam gic cng l 1 trong n im hoc cc
tam gic nm bn trong hnh vung.
Vi n = 1, vi hnh vung l hnh li nn ta c th chia hnh vung thnh
4 tam gic bi cc ng nm trong hnh vung, do cc tam gic u
nm trong hnh vung.
Gi s bi ton ng vi n = k 1. Ta s chng minh bi ton ng
vi n = k + 1.
Tht vy, vi n = k, ta c th chia hnh vung thnh 2k + 2 tam

thi olympic o - Balan

29

gic m cc nh ca cc tam gic cng l mt trong k im hoc


cc tam gic nm bn trong hnh vung. Vi n = k + 1, ta xt thm
im P1 . V khng c 3 im no thng hng nn P nm trong s
2n+2 tam gic, v d ABC. Nh vy ABC chia thnh cc tam
gic nh l AP B; BP CvCP A. Nh vy hnh vung c chia thnh
2(n + 1) + 2 = 2n + 4 tam gic.Vy bi ton c chng minh bng quy
np.
Trng hp c bit n = 23, hnh vung c th chia thnh 48 tam gic
vi tng din tch bng 1. Vy 1 trong cc tam gic trn c din tch ti

.c

tho mn yu cu bi ton.

1
48

a l

Chng 5

thi olympic a Trung Hi

.5.22.Cho n s dng a1 , a2 , ..., an v tp hp {1 , 2 , ..., n }, vi mi i


{1, 1}. Chng minh rng tn ti mt php hon v (b1 , b2 , ..., bn ) ca

.c

a1 , a2 , ..., an v tp hp {1 , 2 , ..., n } vi mi i {1, 1} sao cho du


P
hiu ca ij=1 j bj bng vi du hiu ca i , 1 i n.

Li gii: Ta xy dng mt dy cc s khc khng x1 , x2 , ..., xn vi nhng


thuc tnh sau y:

i) Vi 1 i n, x1 , x2 , ..., xn c gi tr tuyt i khc nhau


ii) Khi c sp xp theo th t gi tr tuyt i tng dn, du hiu

ca chng xen k nhau


iii) Vi 1 i n, du hiu ca cc s trong x1 , x2 , ..., xn c gi tr tuyt
i ln nht bng i .
lm c nh vy, ta ch n gin xy dng x1 , x2 , ..., xn trong trt
t, ti mi bc chn xi0 theo quy tc du hiu ng vi thuc tnh (ii),
vi i = i0 v t hoc |xi0 | > max {|x1 | , |x2 | , ..., |xi0 1 |} hoc
|xi0 | < min {|x1 | , |x2 | , ..., |xi0 1 |}. V vy thuc tnh (iii) ng vi i = i0 .

Chn bi v i sao cho bj1 < bj2 |xj1 | < |xj2 | v j xj > 0, j, j1 , j2 . Gi
thit rng 1 i n. Sp xp b1 , b2 , ..., bi theo bc tng dn thu c
bk1 , bk2 , ..., bki . Bng cch xy dng, dy cc du hiu k1 , k2 , ..., ki xen
k nhau, v ki = i . V vy :

thi olympic a Trung Hi


i
X
j=1

31


j bj = i bki bki1 + bki2 bki3 + ... bk1 .

Biu thc trong ngoc l tng ca bk/2c biu thc dng ca cc hnh
thc bkj+1 bkj v c th cng thm vo mt s hng bk1 . V vy,
Pi
j=1 j bj c cng du hiu vi i vi mi i, pcm.

.5.23.Cho t gic li ABCD. Dng ra pha ngoi cc cnh ca t gic cc


tam gic u W AB, XBC, Y CD, ZDA vi S1 , S2 , S3 , S4 ln lt l trng
tm. Chng minh rng S1 S3 S2 S4 AC = BD.
Li gii: Chn O l mt im bt k. Gi a, b, c, d theo th t biu th
cho cc vect t O n A, B, C, v D. M1 , M2 , M3 , M4 theo th t l
trung im ca AB, BC, CD, DA, v si biu th cho cc vect t Mi ti
Si vi i = 1, 2, 3, 4.
C 2 vect x v y, cho (x, y) l gc gia chng thun theo chiu kim

.c

ng h. (Tt c cc gc u mod 2). Khng mt tnh tng qut, gi


s rng ABCD l nh hng theo chiu kim ng h, v cho l php

bin i m quay bt k vect no /2 ngc chiu kim ng h v


nhn ln ca n ln bi 21 3 . Khi :

(x) (y) = | (x)| | (y)| ( (x) , (y))





|y|
|x|

(x, y)
=
2 3
2 3
1
= xy
12

Tch v hng ca vect S3 S1 vi vect S4 S2 bng :




(b d) + (a c)
0
0
+ s1 s3
2

 

(b d) (a c)
0
0
+ s2 s4 ,

v bng tng ca 4 biu thc sau y :


2

|b d| |a c|
4

 0
  0

0
0
, s1 s3 s2 s4 ,

Nguyn Hu in, HKHTN H Ni

32

i
1h 0
0
0
0
s1 (b a) s3 (c d) + (b c) s2 (a d) s4 ,
2
i
1h 0
0
0
0
s1 (c d) s3 (b a) + (a d) s2 (b c) s4 .
2

Biu thc u tin bng

(BD2 AC 2 ). Bn s hng trong biu thc

1
4

th 3 u bng 0: MS1 AB ng rng s1 (b a) = 0, v. v. . .


Trong biu thc th 2, ta thy :
0

s1 s3 = ((b a) (d c)) = ((c a) + (b d))


v

s2 s4 = ((c b) (a d)) = ((c a) (b d))

1
(CA2 BD2 ).
hay 12
i vi biu thc th t,

.c

Do , tch v hng ca chng bng 1/12 ca :


((c a) + (b d)) ((c a) (b d)) = |c a|2 + |b d|2 ,

trong khi

. 
s1 (c d) = AB 2 3 (CD) cos (/2 + (a b, c d))



. 
0
0
s3 (b a) = s3 (a b) = CD 2 3 (AB) cos (/2 + (c d, a b)) .
Tng ca cc argument ca 2 csin l :
+ ( (a b, c d) + (c d, a b)) = 3,
Vi ng rng gi tr ca mi csin l ph nh ca ci khc. Do , s1
0
v s3 trong biu thc 4 trit tiu ln nhau bn ngoi. Tng t, cng
0

lm nh vy vi s2 v s4 .

1
V th, ton b tch v hng bng 14 12
(BD2 AC 2 ). V
S1 S3 S2 S4 khi v ch khi tch v hng ny bng 0, S1 S3 S2 S4
0

BD = AC, pcm.

thi olympic a Trung Hi

33

.5.24.P, Q, R, S theo th t l trung im ca cc cnh BC, CD, DA, AB ca


t gic li ABCD. Chng minh rng :


4 AP 2 + BQ2 + CR2 + DS 2 5 AB 2 + BC 2 + CD2 + DA2
Li gii: Ta bit cng thc : XM l trung tuyn ca tam gic
XY Z, th XM 2 = 12 XY 2 + 12 XZ 2 41 Y Z 2 . Ta thay (X, Y, Z, M) bng

(A, B, C, P ), (B, C, D, Q), (C, D, A, R) v (D, A, B, S) vo trong cng


thc ny v cng 4 cng thc li vi nhau thu c 1 cng thc th
5. Nhn c 2 v ca cng thc th 5 vi 4, ta tm thy v tri ca bt
ng thc s bng

AB 2 + BC 2 + CD2 + DA2 + 4 AC 2 + BD2

Do , ta ch cn chng minh AC 2 + BD 2 AB 2 + BC 2 + CD2 + DA2 .

.c

y l cng thc bt ng thc hnh bnh hnh. chng minh n,


gi O l mt im bt k trong mt phng v vi mi im X, gi x

biu th vect t O ti X. Ta c th khai trin ton b cc s hng trong


AB 2 + BC 2 + CD2 + DA2 AC 2 BD 2 , v d vit AB 2 = |a b|2 =
|a|2 2a b + |b|2 , thy rng biu thc ny bng

= |a + c b d|2 0

|a|2 + |b|2 + |c|2 + |d|2 + 2 (a b + b c + c d + d a a c b d)

vi du = xy ra khi v ch khi a + c = b + d(hay khi v ch khi ABCD


l hnh bnh hnh). iu phi chng minh.

Chng 6
thi olympic Petecbua
m

.6.25.Cho AA1 , BB1 , CC1 l ng cao ca tam gic nhn ABC. Hai im

.c

A2 v C2 nm trn ng thng A1 C1 sao cho ng thng CC1 chia


i on thng A2 B1 v ng thng AA1 chia i on thng C2 B1 .

ng thng A2 B1 v AA1 gp nhau ti im K, v ng thng C2 B1


v CC1 gp nhau ti im L. Chng minh rng ng thng KL v AC
song song vi nhau.

Li gii: Gi im K1 v L1 l im gia ca C2 B1 v A2 B1 do K1
nm trn ng AA1 v L1 nm trn ng thng CC1 . D dng chng
minh c ng cao AA1 ca tam gic ABC l ng phn gic ca

tam gic A1 B1 C1 t cho thy rng A1 K1 va l ng phn gic


va l ng trung bnh ca tam gic A1 C2 B1 , v vy A1 C2 = A1 B1 v
A1 K1 cng l ng cao ca tam gic A1 C2 B1 do A1 K1 vung gc
vi B1 C2 tng t C1 L1 vung gc vi A2 B1 .
T suy ra ng thng KK1 v LL1 l ng cao ca tam gic KLB1
a n chng ng quy ti vi ng cao l t B1 trong tam gic ny.
T ng thng KK1 v LL1 gp nhau ti trc tm H ca tam gic
ABC, l phi i qua im B1 v H. Do l vung gc vi AC. Bi v l l
ng cao trong tam gic KLB1 i qua B1 , n cng vung gc vi KL.
Chng ta kt lun rng KL//AC, nh bi.
.6.26.Mt trm im c chn trong mt phng to . Hy ch ra rng ti

thi olympic Petecbua

35

a 2025 = 452 cc hnh ch nht vi cc nh trong s cc im ny c


cnh song song vi cc trc.
Li gii: Li gii th nht: Gi O l mt trong 100 im, v gi hnh
ch nht c gi tr nu cc nh ca n l O v 03 im khc c la
chn. Chng ta khng nh rng c t nht 81 hnh ch nht c gi tr.
V qua O ng thng I1 v I2 song song vi trc to , ti m c
chn l cc im nm trn I2 {O}. Cho im c nh bt k chn

P khng nm trn I1 hoc I2 , ti a mt hnh ch nht c gi tr c P


nh l mt nh, hn th na, tt c cc hnh ch nht c gi tr ca
dng ny cho im P no . Do c 99 m n nh im P, c ti a
nhiu tam gic c gi tr kiu nh vy. Nu m + n > 17, chng ta lm

c. Mt khc, cho mt cp (P, Q) l cc im chn, y P thuc


I1 {O} v Q thuc I2 {O}, t nht c mt hnh ch nht gi tr c

.c

P v Q nh l cc nh; hn na tt c cc hnh ch nht c gi tr ca


dng ny i vi cp (P, Q) nh vy. V c mn < m(17 m) < 8.9 = 72
cc cp nh vy, c t nht 72 < 8 hnh ch nht c gi tr.

Chng ta kt lun rng trong bt k trng hp no, c t nht 81 hnh


ch nht m gi tr ca n l O v c 03 im khc c la chn. Bin

i O trn tt c 100 im, m s hnh ch nht cho mi O. Tng cng


s im t nht l 8100, v chng ta tam gic bt k m cc nh ca

n c la chn cc im l 4 ln. V vy, c t nht 8100/4 = 2025


hnh ch nht, nh bi yu cu.

Li gii th hai: Gi mt hnh ch nht thch hp nu 4 nh ca n


c la chn l cc im. V tt c cc ng thng ng I1 ....., In i
qua t nht mt trong nhng im la chn. Gi thit rng Ii cha xi
P
l cc im la chn, s := ni=1 xi = 100. S tam gic thch hp
vi cc cnh bn trn ng thng ith v j th l ti a min{Cx2i , Cx2j }.

i vi cc s nguyn
Quan st thy rng min{Cx2i , Cx2j } 6 2xyxxy
4
dng l x v y, bi v nu x 6 y th cnh bn tay tri ti a l
x(x1)
2

1
[x(y 1) + y(x 1)]. Do
4
P
P
x +x
2x x )
l 16i6j6n i4 j 16i6j6n i 4 j
P
2525 14 ( ni=1 x2i + 100n)

vy s tam gic thch hp ti a


P
= 14 (s2 ni=1 x2i ) 41 (n 1)s =

p dng bt ng thc bnh phng trung bnh nghim v bt ng thc

Nguyn Hu in, HKHTN H Ni

36

trung bnh hnh s hc, ta thy rng biu thc t cui cngq
ny ti a
100
1 s2
n) =
l bng 2525 4 ( n +100n) = 252525( n +n) > 252525.2 ( 100
n

2025 Nh bi yu cu

.6.27.a, Tm tt c cc cp ca cc s nguyn khc bit a, b


l lu tha ca mt s nguyn t.

b2 +a
a2 +b

v b2 + a

. Chng
b, Cho a v b l cc s nguyn dng ln hn 1tho mn ab 2+a1
+b1
2
minh rng b + a 1 c t nht hai tha s nguyn t khc nhau.
Li gii:
a, Chng ta chng minh rng ch c cp nh vy l (a, b) = (5, 2). Nu

a+1
(b = 1), th aa+1
2 +1 dn n a(a+1)(a2 +1) = a 1. Do vy a=1 khng a
ra li gii khi a, b c a ra yu cu khc bit. V vy tng b>1, v

vit b2 + a = pm khi p l nguyn t v m > 1.


Quan st thy b(b3 + 1) (b2 )2 + b a2 + b 0(mod(b2 + a)), b2 + a

.c

chia cho b(b3 + 1) nhng gcd(b, b3 + 1)v b2 + a l lu tha ca mt


nguyn t, v vy mt trong s b hoc b3 + 1 c th chia ht cho b2 + a.
Trng hp u r rng l khng th c, trong trng hp th 2,
2

b +a
2
2
chng ta c (b+1)(b
2 b+1) . Mi b+1 v b b + 1 t hn b + a, v vy c
hai u khng chia ht cho b2 + a. V b2 + a = pm l lu tha ca p,

chng ta kt lun rng p chia ht cho c b+1 v b2 b + 1, th n cng


phi chia ht cho (b2 b + 1) (b + 1)(b 2) dn n p=3.

Khng c li gii cho m=1. Nu m=2, th chng ta c b2 + a = 9, li gi


d dng l (a,b)=(5,2). Mt khc, gi s rng m > 3, mt trong b+1 v

b2 b + 1 chia ht cho 3 v chia ht cho 3m1 nhng b2 < b2 + a = 3m


dn n rng b+1 nhiu hn 3m/2 + 1 < 3m1 v khng th chia ht cho
3m1 . Chng ta kt lun rng 3m1 chia ht cho b2 b + 1 v do vy 9

chia ht cho 4.(b2 b + 1) = (2b 1).2 + 3. iu ny khng th v khng


c bnh phng ng dng vi 6 modun 9. V vy khng c li gii ngoi
(5,2)
b, Gi thit, v s mu thun, rng b2 1+a l lu tha ca mt nguyn

t. V (b2 1)2 a2 chia ht cho b2 1 + a, khi a2 + b 1 l gi thit


tng ca chng l (b2 1)2 + b + 1 = b(b 1)(b2 + b 1). Quan st
thy rng b, b-1, v b2 + b 1 = b(b + 1) 1 = (b + 2)(b 1) + 1 l
nguyn t cng nhau tng cp. V vy, mt trong b, b-1, v b2 + b 1

thi olympic Petecbua

37

phi chia ht cho lu tha nguyn t b2 1 + a. V b v b-1 nh hn


b2 + a 1 chng ta phi c rng b2 + a 1 chia cho b2 + b 1 v v vy
2

chng ta phi c
a 6 b v a 6= b l gi thit a<b. Mt khc, v ba2+a1
+b1
2
2
0 6 (a + b 1) (b + a 1) = (a b)(a + b 1). V vy a > b nh l
mt mu thun .

.6.28.Trong mt t nc c 2000sn bay, ban u khng c mt chuyn bay


no ca hng hng khng. Hai hng hng khng ln lt gii thiu nhng
chuyn bay thng kh hi mi (gia hai thnh ph bt k, ch c mt
chuyn bay thng c gii thiu). C quan vn ti mun t c mc
tiu l nu sn bay no b ng ca th mi ngi vn c th du lch gia
hai sn bay bt k khc, c th bng chuyn ti. Hng hng khng to

ra mc ch t c s b thua l. Hng hng khng no s chin


thng cuc chi mt cch hon ho?

.c

Li gii:
Cng ty hng khng th hai chin thng. Xem xt tnh hung khi

m mc tiu khng t c, nhng vic b sung thm chuyn bay n


bt k t c mc ch nu ra. Do mc tiu khng t c, c

sn bay A bt k khi ng ca chia cc thnh ph ra lm hai nhm


khng lin lc c l G1 v G2. Khi hai thnh ph bt k nm

trong G1 hoc trong G2 th phi gia nhp, bi v nu khng b sung


chuyn bay gia hai thnh ph th mc tiu a ra khng t c.
Tng t nh vy, tt c cc thnh ph phi gia nhp vo A, nhng

khng c thnh ph trong G1 c th gia nhp vi bt k thnh ph no


chuyn bay
ca G2. Do nu h s k thnh ph trong G1, th c k1
2
gia hai thnh ph trong G1, (1999k)(1998k)
chuyn bay gia hai thnh
2
ph trong G2, v 1999 chuyn bay gia A v thnh ph khc. Nh vy
tng s chuyn bay l k(k-1999)+1999000 l u nhau. Ni c th, cha
bao gi ti lt ca hng hng khng th hai b sung thm mt chuyn
bay mi. Do vy, cng ty hng khng th hai lun trnh c s thua
l.
.6.29.Chng ta c phng trnh bc hai a thc li, tt c c cng bit s.
Tng ca hai a thc bt k c nghim thc ring bit. Ch ra rng tng

Nguyn Hu in, HKHTN H Ni

38

s ca tt c cc a thc cng u c nghm thc ring bit.


Li gii:
Bit s chung phi l dng, bi v nu khng mi a thc ch c gi tr
dng, v vy tng ca hai a thc bt kk s khng c nghim thc.
Cho bit s chung l 4D, mi a thc l dng (x c)2 D i c bt
k. mi a thc, c xem l khong cch trong a thc c cha

gi tr m, khong cch c di l 2 D . Nu hai trong s khong

cch ny (c1 D, c1 + D v (c2 D, c2 + D khng giao nhau,

th |c2 c1| > D v 12 (c1 + c2) khng nm trong mt trong hai khong
cch. Do vy c hai a thc u khng m ti 21 (c1 + c2) nhng im
ny khi tng s a thc p t c gi tr nh nht ca n - mu thun

vi gi thit l c p nghim thc phn bit Do vy, hai khong cch

giao nhau bt k. Chn mt khong cch (c D, c + D c l cc

tiu. V tt c cc khong cch giao nhau bt k khc iu ny, chng

ta thy rng tt c cc khong cch u cha (c + D thuc i vi

.c

thuc bt k. Ti im ny, tng s ca tt c cc a thc cha gi tr


m a, do vy tng ny phi c nghim thc ring bit.

.6.30.Trn mt bn c am v hn t 111 gc khng chng nhau. Cc hnh


c ch L c t lm 3 n v hnh vung. Tp hp c tnh cht sau:

i vi gc bt k, cha 2.2 hnh vung bao ph ton b cc gc. Chng


minh rng mt gc c th dch chuyn gia mt v 110 ca cc gc
tnh cht c bo v.
Li gii: Nu 2.3 hnh ch nht bt k c bao ph bi hai gc, th
chng ta c th d chuyn tt c cc gc ngoi tr hai gc . Do vy,
chng ta c th tng kt rng khng c hnh ch nht nh vy tn ti.
Chng ta xy dng mt th trc tip m cc nh ca n l cc gc,
nh sau: i vi mi gc, v 2.2 hnh vung c cha gc , v thm
mt ng ni t gc ny ti gc kia bao ph s d ca 2.2 hnh vung.
Nu mt gc khng c im ni v phi n, chng ta c th dch chuyn
gc , v vy chng ta c th tng kt rng, khng c gc nh vy tn
ti. Do , mi ng ni ca th nm trong chu trnh no . Nu
c hn mt chu trnh, th chng ta c th dch chuyn tt c cc gc

thi olympic Petecbua

39

ngoi tr cc gc nm trong chu trnh ca chiu di di cc tiu, v yu


cu c tnh c tn ti. V vy, n tho mn ch ra rng khng th
tn ti mt chu trnh n bao gm tt c 111 nh.
Theo im gia ca mt gc chng ta hng theo im ti im gia ca
2.3 hnh vung cha gc . Nh li rng chng ta tng kt khng
c hai gc bao ph 2.3 hnh vung, mt iu d dng kim tra l nu
c mt im ni t mt gc ti mt gc khc, th im gia ca nhng
gc ny khc 1 trong c hai to ca chng l x v y. Do , trong
chu trnh bt k, to x ca cc nh nn trong s bin i ca chu

.c

trnh, do s ca cc nh nm trong chu trnh l chn. Do vy, khng


c mt chu trnh cha tt c 111nh, nh bi a ra.

Chng 7

thi olympic Anh

.7.31.Cho hai ng trn ct nhau (C1 ) v (C2 )c mt tip tuyn chung tip
xc (C1 ) ti P, tip xc (C2 ) ti Q. hai ng trn ny ct nhau ti M

.c

v N. Chng minh rng tam gic MNP v tam gic MNQ c cng din
tch.

Li gii: Gi X l giao im ca MN v PQ. V MN l trc ng


phng ca (C1 ) v (C2 ), X c cng mi lin quan ny vi hai ng
\ =
trn vy XP 2 = XQ2 hay XP = XQ Cng v P\
XM + MXQ
\ cho nn [MNP ] = 1 MN(XP sinP\
XM ) =
ta c sinP\
XM = sinMXQ

1
\
MN(XQsinMXQ)
2

\ ta c iu phi chng minh.


= MNQ.

.7.32.Cho x, y, z l nhng s thc dng tho mn xyz = 32. Tm gi tr nh


nht ca x2 + 4xy + 4y 2 + 2z 2 .
Li gii: p dng bt ng thc trung bnh hai ln ta c:
p
x2 + 4xy + 4y 2 + 2z 2 = (x2 + 4y 2)2 + 4xy + 2z 2 2 x2 4y 2 + 4xy + 2z 2 =
p
p
4xy + 4xy + 2z 2 3 3 4xy4xy2z 2 = 3 3 32(xyz)2 = 96.
Du "=" xy ra khi v ch khi x2 = 4y 2 v 4xy = 2z 2 , tc l
(x,y,z)=(4,2,4)

.7.33.a, Tm mt tp A ca 10 s thc nguyn m khng c 6 phn t phn


bit no ca A c tng chia ht cho 6
b, C th tm c mt tp nu 10 c thay bi 7 hay khng?

thi olympic Anh

41

Li gii: a, Mt v d ca tp A l A = {6j + k|1 j 5, 1 k 2}


Trong bt k 6 phn t trong tp con ca A nu c t s ng d 1
modun 6 th t {1, 2, ..., 5}. nhng phn t khc trong tp con l ng
d 0 mdun 6. V vy tng ca nhng phn t trong tp con l ng
d t 6= 0 (moun6)
b, Khng th cho bt k tp no c 7 s thc nguyn, chng ta c 6
phn t phn bit ca tp ny c tng chia ht cho 6. Bi v c hn 2
s nguyn trong tp ny, chng ta c th chn hn 2 m l chn. Cng
cch ny lm nh th ta c th tm thy 5 tp con c 2 phn t ri rc
ci m c tng l ng d ca 0, 2, 4 moun 6. Nu tt c tng xut
hin, 6 phn t trong tp con tng ng c tng ng d 0 + 2 + 4 =

6 (moun 6). Cch khc, ch tng xut hin . Bi Pizenhle principle,


3 tp con s c tng nh nhau. Do nhng phn t trong 3 cp s c

.c

tng chia ht cho 6.

Ti liu tham kho


[1] Titu Andreescu, Zuming Feng, and George Lee, Jr. Mathematical
Olympiads 20002001, Problems and Solutions From Around the World,
The Mathematical Association of America, 2002.

[2] Nguyn Hu in, Phng php irichle v ng dng, NXBKHKT, 1999.

[3] Nguyn Hu in, Phng php Quy np ton hc, NXBGD, 2000.

.c

[4] Nguyn Hu in, Nhng phng php in hnh trong gii ton ph

thng, NXBGD, 2001.

hc, NXBKHKT, 2001.

[5] Nguyn Hu in, Nhng phng php gii bi ton cc tr trong hnh

[6] Nguyn Hu in, Sng to trong gii ton ph thng, NXBGD, 2002.

[7] Nguyn Hu in, a thc v ng dng, NXBGD, 2003.


[8] Nguyn Hu in, Gii phng trnh v nh nghim nguyn,
NXBHQG, 2004.
[9] Nguyn Hu in, Gii ton bng phng php i lng bt bin,
NXBGD, 2004.

Nguyn Hu in

OLYMPIC TON NM 1997-1998

.c

(Tp 4)

51 THI V LI GII

NH XUT BN GIO DC

.c

Li ni u
th gi lnh lamdethi.sty ti bin son mt s ton thi Olympic, m
cc hc tr ca ti lm bi tp khi hc tp LATEX. ph v cc bn ham
hc ton ti thu thp v gom li thnh cc sch in t, cc bn c th tham
kho. Mi tp ti s gom khong 51 bi vi li gii.

Rt nhiu bi ton dch khng c chun, nhiu im khng hon ton

.c

chnh xc vy mong bn c t ngm ngh v tm hiu ly. Nhng y l ngun


ti liu ting Vit v ch ny, ti c xem qua v ngi dch l chuyn v
ngnh Ton ph thng. Bn c th tham kho li trong [1].

H Ni, ngy 2 thng 1 nm 2010


Nguyn Hu in

Rt nhiu on v mi hc TeX nn cu trc v b tr cn xu, ti khng


c thi gian sa li, mong cc bn thng cm.

51
89/176-05
GD-05

M s: 8I092M5

Mc lc
3

Mc lc . . . . . . . . . . . . . . . . . . . . . . . . . . . . . . . . . . . . . . . . . . . . . . .

Chng 1. thi olympic Austria. . . . . . . . . . . . . . . . . . . . . . . . . .

Chng 2. thi olympic Bungari . . . . . . . . . . . . . . . . . . . . . . . . .

Chng 3. thi olympic Canada . . . . . . . . . . . . . . . . . . . . . . . .

13

Chng 4. thi olympic Chine . . . . . . . . . . . . . . . . . . . . . . . . . .

17

Chng 5. thi olympic Colombia . . . . . . . . . . . . . . . . . . . . . .

21

Chng 6. thi olympic Czech v Slovak Repubulick . . . .

24

Chng 7. thi olympic Php . . . . . . . . . . . . . . . . . . . . . . . . . . .

28

.c

Li ni u . . . . . . . . . . . . . . . . . . . . . . . . . . . . . . . . . . . . . . . . . . . .

Chng 8. thi olympic c . . . . . . . . . . . . . . . . . . . . . . . . . . . .

31

Chng 9. thi olympic Irland . . . . . . . . . . . . . . . . . . . . . . . . . .

37

Chng 1
thi olympic Austria

.c

(x 1)(y 2 + 6) = y(x2 + 1),


(y 1)(x2 + 6) = x(y 2 + 1)

.1.1. Gii h phng trnh vi x, y l s thc

Li gii: Ta cng hai phng trnh trn cho nhau. Sau khi rt gn v a v

bnh phng ca mt hiu ta c phng trnh sau


(x 25 )2 + (y 25 )2 = 21

Chng ta li tr hai phng trnh cho nhau, tr phng trnh th hai cho
phng trnh th nht v nhm li, ta c:
xy(y x) + 6(x y) + (x + y)(x y) =

xy(x y) + (y x)

(x y)(xy + 6 + (x + y) xy + 1) =

(x y)(x + y 2xy + 7) =

Do vy, hoc xy = 0 hoc x+ y 2xy + 7 = 0. Cch duy nht c xy = 0

l vi x = y = 2 hoc x = y = 3 (tm c bng cch gii phng trnh (1))


vi php th x = y
By gi, ta xt trng hp x 6= y s c gii x + y 2xy + 7 = 0. Phng

trnh ny l tng ng vi phng trnh sau(c suy ra t cch sp xp

Nguyn Hu in, HKHTN H Ni

li cc s hng v tha s)
(x 12 )(y 21 ) = 15
4

Gi s, chng ta c th gii phng trnh (1) v (2) mt cch ng thi. t


a = x 52 v b = y 52 . Do , phng trnh (1)) tng ng vi
a2 + b2 = 21
v phng trnh (2) tng ng vi:
15
1
1
(a + 2)(b + 2) =
ab + 2(a + b) = 2ab + 4(a + b) =
4
4
2
Cng phng trnh (4) v (3) chng ta thy:
(a + b)2 + 4(a + b) = 0 a + b = 0, 4
Ly phng trnh (4) tr (3) ta thy:
(a b)2 4(a + b) = 1

Nhng by gi chng ta thy rng ,nu a + b = 4 th phng trnh (6) s b


sai; Do , a + b = 0. Th a + b = 0 vo phng trnh (6) chng ta thu c:

.c

(a b)2 = 1 a b = 1
V t phng trnh (5) chng ta c a+b = 0,v cng vi phng trnh (7)by gi

ta c th tm c tt c cc cp c th t (a, b). Chng l ( 12 , 21 ) v ( 12 , 21 )


. Do vy, cc nghim (x, y)ca h phng trnh cho l(2, 2),(3, 3),(2, 3) v

(3, 2).

.1.2. Cho dy s nguyn dng tha mn an = a2n1 + a2n2 + a2n3 vi n 3.


Chng minh rng nu ak = 1997 th k 3.

Li gii: Chng ta gii trc tip: Gi s vi k > 3, ak = 1997. Khi ,

c t nht mt s trong 4 s ak1 , ak2 , ak3 vak4 phi tn ti. t w =


ak1 , x = ak2 , y = ak3 v z = ak4 .By gi, iu kin ca chng ta l:

1997 = w 2 + x2 + y 2. Do , w 1997 < 45 v v w l mt s nguyn dng


nn w 44. Nhng do x2 + y 2 1997 4462 = 61.

By gi, (vi) w = x2 + yx2 + z 2 . V x2 + y 2 > 61 v z 2 > 0, x2 + y 2 + z 2 > 61.


Nhng w 6 44. Do , chng ta c mu thun v gi thit ca chng ta l
khng ng.
Vy, nu ak = 1997 th k 6 3.
.1.3. Cho k l mt s nguyn dng. Dy a n c xc nh bi a 1 = 1
v an l n s nguyn dng ln hn an1 l ng d n modulo k. Tm an
trong dy trn.

thi olympic Austria

. Nu k = 2 th an = n2 . Trc tin,
Li gii: Chng ta c an = n(2+(n1)k)
2
ch rng a1 1(modk). Do , vi tt c n, an n(modk), v s nguyn

u tin ln hn an1 m l ng d n modulo k phi l an1 + 1.


n - th s nguyn dng ln hn an1 l ng d n modul k l n gin (n - 1)k
hn s nguyn dng u tin ln hn an1 m tha mn iu kin . Do
vy, an = an1 + 1 + (n 1)k. Li gii bng php quy ny a ra cu tr
li ca bi ton trn.

\ v
.1.4. Cho hnh bnh hnh ABCD, mt ng trn ni tip trong gc BAD
nm hon ton trong hnh bnh hnh. Tng t, mt ng trn ni tip trong
\ nm hon ton trong hnh bnh hnh sao cho 2 ng trn tip
gc BCD
xc. Hy tm qu tch cc tip im ca 2 ng trn khi chng thay i.
\ sao cho
Li gii: Gi s K1 l ng trn ln nht ni tip trong gc BAD

n nm hon ton trong hnh bnh hnh. N ct ng thng AC ti 2 im


v gi s im xa A hn l P1 . Tng t Gi s K2 l ng trn ln nht
\ sao cho n nm hon ton trong hnh bnh hnh. N
ni tip trong gc BCD

.c

ct ng thng AC ti 2 im v gi s im xa C hn l P2 . Khi , qu

tch l giao ca 2 on AP1 v AP2 .


Chng ta bt u chng minh im tip xc phi nm trn ng AC. Gi s
\ v I2 l tm ng trn ni tip gc
I1 l tm ng trn ni tip gc BAD
\ Gi s X l im tip xc ca 2 ng trn. V cc ng trn tm I1
BCD.

v I2 l ni tip trong cc gc nn cc tm ny phi nm trn cc ng phn

gic ca cc gc. Mt khc v AI1 v CI2 l cc ng phn gic ca cc gc


i hnh bnh hnhneen chng song sonh vi nhau. Do vy I1 I2 l ng nm
ngang.
Gi s T1 l chn ng vung gc h t I1 ti AB v T2 l chn ng vung
gc h t I2 ti CD. Ch rng:
I2 T2
I1 T1
= sin Ib1 AB = sin Ib2 CD =
AI1
CI2

Nhng I1 X = I1 T1 v I2 X = I2 T2 . Do vy
I1 X
I2 X
=
AI1
CT2

V th tam gic CI2 X v tam gic AI1 X l ng dng v cc gc vung


\
I\
1 XA, I2 XC l bng nhau. V cc gc ny bng nhau nn cc im A, X v C

Nguyn Hu in, HKHTN H Ni

phi cng tuyn. Do vy, im tip xc X phi nm trn ng cho AC (


l iu phi chng minh).
Nh vy, chng ta bit rng X s lun nm trn AC, by gi ta s chng minh
bt k im no thuc qu tch u l im tip xc. Cho X bt k nm
trn qu tch , gi s I1 l ng trn b hn ng trn qua X, ni tip
\
trong gc BAD.
N s nm hon ton bn trong hnh bnh hnh bi v X l im gia A v P1 .
Tng t, ta v mt ng trn tip xc vi ng trn I1 v ni tip trong
\ t chng minh trn ta bit rng n phi tip xc vi ng trn I1
gc BCD,
ti X, hn na n s hon ton xc nh bn trong hnh bnh hnh bi v X
l im gia ca C v P2 .

V vy, bt c im no thuc qu tch s chy qua X. chng minh rng


bt k im no khc s khng chy qua. Ch rng bt k im no s hoc

khng nm trn ng thng AC hoc s khng cho 1 trong 2 ng trn I1


hoc I2 c cha bn trong hnh bnh hnh. Do vy, qu tch thc s l giao

.c

ca cc on AP1 v CP2 .

Chng 2

.2.5. Tm tt c cc s thc m phng trnh

thi olympic Bungari

.c



x2 2mx 4(m2 + 1) x2 4x 2m(m2 + 1) = 0

c ng ba nghim phn bit.

Li gii: p n: m = 3.

Cho hai tha s v tri ca phng trnh bng 0 ta nhn c hai phng
trnh a thc. t nht mt trong cc phng trnh ny phi nghim ng vi

gi tr x no x l nghim ca phng trnh ban u. Nhng phng trnh


ny c th vit di dng (x m)2 = 5m2 + 4 (1)v (x 2)2 = 2(m3 + m + 2)

(2). Ta c ba trng hp m phng trnh ban u c th c 3 nghim phn


bit: Phng trnh (1) c nghim kp hoc phng trnh (2) c nghim kp
hoc hai phng trnh c mt nghim chung. Tuy nhin, trng hp th nht

khng xy ra v hin nhin 5m2 + 4 = 0 khng th tha mn vi mi gi tr


thc m.
Trong trng hp th hai, ta phi c 2(m3 + m + 2) = 0; m3 + m + 2 phn
tch thnh (m + 1)(m2 m + 2) v tha s th hai lun dng vi mi gi tr

thc m. V vy ta phi c m=-1 trng hp ny xy ra. Khi nghim duy


nht ca phng trnh ny l x=2 v phng trnh (1) tr thnh (x + 1)2 = 9,
tc l x=2, -4. Nhng iu ny c ngha l phng trnh ban u ca ta ch
c nghim l 2 v -4, tri vi yu cu ca bi ton.

10

Nguyn Hu in, HKHTN H Ni

Xt trng hp th ba, gi r l nghim ca phng trnh th x r l mt


tha s ca c hai biu thc x2 2mx 4(m2 + 1) v x2 4x 2m(m2 + 1).
Tr hai biu thc ny cho nhau ta nhn c x r l mt tha s ca
(2m 4)x (2m3 4m2 + 2m 4), hay (2m 4)r = (2m 4)(m2 + 1). V

vy m = 2 hoc r = m2 + 1. Tuy nhin, trong trng hp th nht th c


hai phng trnh bc hai ca ta tr thnh (x 2)2 = 24, v v vy, ta ch

thu c hai nghim phn bit. Vy ta phi c r = m2 + 1. Khi , thay vo


ng thc (r 2)2 = 2(m3 + m + 2), ta c (m2 1)2 = 2(m3 + m + 2) hay
(m + 1)(m 3)(m2 + 1) = 0. Do m = 1 hoc 3. Trng hp m=-1

c ch ra khng tha mn. V vy, ta ch c m=3. Khi cc phng trnh


ca ta tr thnh (x 3)2 = 49 v (x 2)2 = 64, chng c cc nghim l x=-6,
-4, 10, tha mn yu cu ca bi ton.

.2.6. Cho ABC l tam gic u c din tch bng 7. Gi M, N tng ng l


cc im trn cnh AB, AC sao cho AN=BM. Gi O l giao im ca BN v

.c

CM. Bit tam gic BOC c din tch bng 2.


B
hoc bng 31 hoc bng 23 .
(a) Chng minh rng M
AB
[
(b) Tnh gc AOB

Li gii: (a) Ly im L trn BC sao cho CL=AN v gi P, Q ln lt l


giao im ca CM v AL, AL v BN. Php quay vi gc quay 120o quanh tm

ca tam gic ABC bin A thnh B, B thnh C, C thnh A; php quay ny


cng bin M thnh L, L thnh N, N thnh M v bin O thnh P, P thnh Q,
Q thnh O. Do OPQ v MLN l cc tam gic u ng tm vi tam gic
\
\ 2 . V vy, O nm trn ng trn i xng
ABC. Suy ra BOC=MOC=
3

vi ng trn ngoi tip tam gic ABC qua BC. C nhiu nht hai im O
trn ng trn ny v nm trong tam gic ABC t l khong cch t O
ti BC v t A ti BC bng 27 , t l ny cng l t l din tch ca cc tam
B 1
gic OBC v ABC. V vy ta ch ra rng M
= 3 hoc 32 tng ng vi cc
AB
v tr ca im O, v khng c t l no khc (tc l khng c hai im M cho
B
= 31 th AN
= 13 , p dng nh l Menelaus cho
cng mt im O. Nu M
AB
AC
[BOC]
BO
= 43 , do [BN
= BO = 73 . Suy
tam gic ABN v ng thng CM, ta c ON
C] BN
B 2
= 3 CN = 72 v ta c iu phi chng minh. Tng t, nu M
= 3 , theo
ra [BOC]
[ABC] 7 CA
AB

nh l Menelaus ta c

BO
= 67 ,
BN

do

[BOC]
= BO = 67 .
[BN C] BN

Suy ra

[BOC] 6 CN
=
= 72 .
[ABC] 7 CA

(b)

11

thi olympic Bungari

pi v O
= P[
th MONA l mt t gic ni tip do A=
OQ = 2
. Do
3
3
M
B
1
AN

[ AOM+
\ MOB=
\ ANM
\ +P[
\ + . Nhng
= 3 v AC = 31
AOB=
OQ = ANM
3
AB
\ = v
nn d dng thy c N l hnh chiu ca M trn AC. V vy ANM
MB 1
=3
AB

[ = 5 . Lp lun tng t i vi trng hp cn li, ta c ANM


\ =
AOB
6
[ = .
v AOB

.2.7. Cho f (x) = x2 2ax a2 34 . Tm tt c cc gi tr ca a |f (x)| 1


vi mi x [0; 1].

Li gii: p n: 12 a 42 .
th ca f(x) l mt parabol c im cc tiu (c ngha l h s a m) v

nh l (a;f(a)). T f (0) = a2 34 ta c |a| 21 f (0) 1. Gi s a 0


th parabol ca ta tng nghim ngt trong khong t 0 n 1, do f (1) 1.

Nhng ta c 12 a + 1 1, 41 (a + 1)2 1, 14 54 (a + 1)2 1. T


5
(a + 1)2 = f (1), ta c f tha mn iu kin ca bi ra khi 12 a 0.
4
Vi a > 0, f gim vi 0 x a v tng vi a x 1. V vy ta cn ch ra

.c

gi tr nh nht ca f(a) nm trong phm vi theo yu cu ca bi ton, tc


l f(1) nm trong gii hn ny. T a 12 ta c 1 < (a + 1)2 49 v v vy

2
.
4

mn iu kin ca bi ra khi 0 < a

f (x) = 1 54 (a + 1)2 < 41 . Mt khc, f (a) = 2a2 43 nn ta phi c

a 42 f (a) 1. Ngc li, nh gi f(0), f(a), f(1) ta ch ra c f tha

.2.8. K hiu u(k) l c l ln nht ca s t nhin k. Chng minh rng


n

2
1 X u(k)
2

.
n
2 k=1 k
3

Li gii: t v(k) l c ln nht ca k c dng ly tha ca 2, nn u(k)v(k) =


k. Trong {1, 2, ..., 2n} c 2ni1 gi tr ca k sao cho v(k) = 2i vi i n 1,
v mt gi tr sao cho v(k) = 2n . Do , v tri bng
n

2
n1 ni1
X
1 X u(k)
2
1

= n+
.
n
2 k=1 k
4
2n+i
i=0

T tng ca chui hnh hc ta c


n

2
1 X u(k)
2
2

= 4n + (1 4n ) > .
n
2 k=1 k
3
3

12

Nguyn Hu in, HKHTN H Ni

.2.9. Tm tt c cc s thc tha mn h


x3 = 2y 1
y 3 = 2z 1

z 3 = 2x 1.

Li gii: Trc ht ta ch ra rng x = y = z. Gi s tri li rng x 6= y.


3
3
Nu x > y, th y = (x 2+1) > (y 2+1) = z, nn y > z, v tng t z > x, mu
thun. Tng t, nu x < y th y < z v z < x, mu thun. Nn cc nghim
ca h phng trnh c dng x = y = z = t vi t l nghim ca phng trnh

t3 = 2t 1. Vy, nghim ca h phng trnh l





1 + 5 1 5
.
,
x = y = z = t, t 1,
2
2

.c

.2.10.Tm s t nhin a nh nht phng trnh sau c mt nghim thc:


 x 
3x
2
cos (a x) 2 cos (a x) + cos
+2=0
cos
+
2a
2a 3

Li gii: Gi tr nh nht ca a l 6. Phng trnh tha mn khi a=6, x=8.

chng minh a l gi tr nh nht, ta vit phng trnh di dng






 x 
3x
2
(cos(a-x)-1) + cos
cos
+1 =0
+
2a
2a
3

Do c hai s hng v tri u khng m nn ng thc xy ra th chng


phi cng bng 0. T cos(a-x)-1 = 0 ta c x phi l mt s nguyn ng d
vi a trong php chia cho 2. T s hng th hai bng 0, ta thy cc gi tr

+ 3 = 1 th x
+ 3 = 2k
cosin phi nhn gi tr bng 1 v -1. Nu cos x
2a
2a
ta c 3x 4a (mod12a). Khi
vi gi tr k nguyn v nhn hai v vi 6a


x

th nu cos 2a + 3 = 1 th 2a + 3 = (2k + 1) v nhn hai v vi 6a


ta

c 3x 4a (mod12a). Trong c hai trng hp ta u c 3x chia ht cho

2, v vy x phi chia ht cho 2 v a cng phi tha mn iu . Hn na,


c hai trng hp ta cng u c -2a v 4a cng phi chia ht cho 3, v th a
phi chia ht cho 3. Tm li ta c 6 phi l c ca a v a=6 l gi tr nh
nht cn tm.

Chng 3
thi olympic Canada

.3.11.C bao nhiu cp s (x; y) nguyn dng vi x y tho mn gcd(x, y) =


5! v lcm(x, y) = 50! ?

.c

Li gii: Trc ht, ch l c 15 s nguyn t t 1 n 50:


(2, 3, 5, 7, 11, 13, 17, 19, 23, 29, 31, 37, 41, 43, 47).

lm cho bi ton n gin hn, ta xc nh f (a, b) l m ln nht ca b


chia cho a. (Ch rng g(50!, b) > g(5!, b) vi mi b < 50.)

Do , vi mi s nguyn t p, ta c

f (x, p) = f (5!, p)

f (y, p) = f (50!, p)

f (y, p) = f (5!, p)

f (x, p) = f (50!, p)

V ta c 15 s nguyn t nn c 215 cp, v trong bt k cp no cng hin


nhin c x 6= y ( do gcd v lcm khc nhau), do c 214 cp vi x y.
.3.12.Cho trc mt s hu hn cc khong ng c di bng 1 sao cho hp
ca chng l khong ng [0, 50], chng minh rng tn ti mt tp con ca
cc khong khng giao vi tt c cc khong khc.
Li gii: Xt
I1 = [1 + e, 2 + e] , I2 = [3 + 2e, 4 + 2e] , ..., I24 = [47 + 24e, 48 + 24e]

14

Nguyn Hu in, HKHTN H Ni

trong e nh 48 + 24e < 50. hp cc on cha 2k + ke, ta phi c


mt on m phn t nh nht nm trong Ik. Tuy nhin, s khc nhau gia
mt phn t trong tp Ik v Ik + 1 lun ln hn 1, v vy cc tp ny khng
chng ln nhau. T 24 khong ban u v [0, 1] ( phi tn ti v hp l [0, 50]
) ta c 25 khong ri nhau m tng di tt nhin bng 25.
.3.13.Chng minh rng:
1
1999

< 12 . 43 . ... . 1997


<
1998

1
44

1997
1998
Li gii: t p = 21 . 34 . ... . 1998
v q = 23 . 54 . ... . 1999
. Ch rng p < q, v vy

=
p2 < pq = 12 . 32 . ... . 1998
1999

1
.
1999

Do ,
p<

1
1999

1
2

1998
1998!
1998

2 = 2
999
(999!.2 )
999

p>

1998
1998

< 1999

1998
999

1
1999

Do

+ ... +

1998

21998 =

ng thi

.c

p=

1
44

Li c

<

[ + COD
\ = .
.3.14.Cho O l mt im nm trong t gic ABCD sao cho AOB
\ = ODC.
\
Chng minh rng OBC

Li gii: Tnh tin ABCD theo vect AD th A v D nh nhau, v v vy


0 D = COD
0 O 0 D 0 = 1800 nn t gic
\ + CO
\
\ + A\
B v C nh nhau. Ta c COD
0 C.
\ = OO
\
OC 0O 0 D 0 ni tip. Do ODC

n
n
P
(1)k
v dng p(n)/q(n), trong
.3.15.Biu din tng sau
k 3 +9k 2 +26k+24
k=0
k
p, q l cc a thc vi cc h s nguyn.

15

thi olympic Canada


Li gii: Ta c

n
P

k=0

k=0

k=1

(1)k k

(1)k k

n+4
k

n+4

1
n+4
1
n+4

(1 1)n+3 = 0

k=1

k+4

n+4
k

n+4
P
k=0

(1)k

n+4
k

3 (1 1)n+4

n+3

n+4
P

(1)k

k1

k=4

n+4

n+4
P

(1)k (k 3)

n+4

(1)k (k 3)

n+4
P

n+4
P

k=0

v
n+4
P

k+1

(1)k (n+1)(n+2)(n+3)(n+4)

1
(n+1)(n+2)(n+3)(n+4)

k=0

(1)
(k+2)(k+3)(k+4)

.c

n
P

k=0

(1)k
k 3 +9k 2 +26k+24

n
P

Do
n+4
P
k=4

(1)k (k 3)

3
P

k=0

= 3

n+4
k

(1)k (k 3)

n+4
0

n+4
k

n+4
1

n+4
2

(n+1)(n+2)
2

16

Nguyn Hu in, HKHTN H Ni

v tng cho bng

.c

1
2 (n + 3) (n + 4)

Chng 4

thi olympic Chine

.c

.4.16. Cho x1 , x2 , ..., x1997 l cc s thc tha mn iu kin

(a) 13 xi 3 vi mi i = 1; 2; ...; 1997


(b) x1 + x2 + ... + x1997 = x12
1997

Li gii: Do x12 l mt hm li ca x nn tng cc ly tha bc 12 ca xi l


cc i nu mi gi tr xi l u mt ca khong quy nh.

Gi s c n gi tr xi bng 13 , 1996 n c gi tr bng 3 v cc gi tr


cui cng bng

n
318 3 + 3(1996 n)
3
h
i
Do gi tr cui cng ny phi nm trong min 13 ; 3 nn
1 318x3 + n 3x(1996 n) 3
tng ng vi
1 4n 6942 3
c duy nht mt s nguyn n = 1736 tha mn.
Khi , gi tr cui cng l

2
3

v gi tr ln nht cn tm l:

4
1736x36 + 260x36 + ( )6
3

18

Nguyn Hu in, HKHTN H Ni

.4.17. Cho t gic li A1 B1 C1 D1 v mt im P nm trong t gic li . Gi


s cc gc P\
A1 B1 v P\
A1 D1 l cc gc nhn, tng t cho ba nh cn li.
Xc dnh Ak , Bk , Ck , Dk l hnh chiu ca P ln cc ng thng Ak1 Bk1 ,
Bk1 Ck1, Ck1 Dk1 (k = 2, 3, ...)
a) Trong cc t gic Ak Bk Ck Dk (k = 1, 2, 3, ..., 12) th t gic no ng dng
vi t gic th 1997
b) Gi s rng t gic th 1997 l ni tip. Hi trong 12 t gic u tin th
t gic no cng ni tip ng trn
Li gii: Ta c Ak chnh l chn ca cc ng vung gc t P n Ak1 Bk1
v tng t nh vy cho cc im cn li. Do cc t gic ni tip vi cc ng
knh P Ak , P Bk , P Ck , P Dk ta c

\
P\
Ak Bk = P D\
k+1 Ak+1 = P Ck+2 Dk+2

.c

\
P B\
k+3 Ck+3 = P Ak+4 Bk+4

Mt khc, ta cng c P\
Bk Ak =P B\
k+1 Ak+1 v tng t nh vy cho cc gc
cn li.

Do vy, cc t gic th 1, 5, 9 ng dng vi t gic th 1997.


Nu t gic th 1997 l ni tip th cc t gic th 3, 7 v 11 cng vy.

c gn cho

.4.18. Ch ra tn ti v s s nguyn dng n sao cho cc s1, 2, 3, ..., 3n c th


a1 , a2 , ..., an , b1 , b2 , ..., bn , c1 , c2 , ..., cn

theo th t ny tha mn iu kin sau:


a) a1 + b1 + c1 = an + bn + cn
b) a1 + a2 + ... + an = b1 + b2 + ... + bn = c1 + c2 + ... + cn l bi ca 6
Li gii: Tng cc s nguyn t 1 n 3n l

3n(3n+1)
2

trong ta i hi phi

va l bi ca 6n v 9.
V th, n phi l bi ca 3 ng d vi 1 theo mddun4.
Ta s ch ra rng tn ti s sp xp cho n = 9m .
Vi n = 9 ta c s sp xp sau:

19

thi olympic Chine


8

16 17 10 15 26 19

24

21 23

25

12 14

16

13 18

11 22 27 20

(trong , dng u tin l a1 , a2 , ..., an v tip tc). iu ny chng t t s


sp xp cho m v n dn n s sp xp cho mn
00

ai+(j1)m = aj + (m 1)aj (1 j m, 1 j n)
v tng t cho bi , ci .
.4.19. Cho ABCD l mt t gic ni tip. Cc ng thng AB v CD ct
nhau ti P . Cc ng thng AD v BC ct nhau ti Q. Gi E v F l giao

.c

im tip tuyn t Q vi ng trn ngoi tip t gic ABCD. Chng minh


rng P, E, F thng hng.

C
Li gii: Gi X 0 l tip tuyn ca ng trn ti im X nm trn ng
trn.
chng minh P, E, F thng hng ta chng minh cc im cc ca n trng
nhau.
Cc tip tuyn E 0 v F 0 ti cc im E v F ct nhau ti Q. Do P l giao ca
AB v CD nn im cc ca P l ng thng i qua giao im ca A0 giao
vi B 0 v C 0 giao vi D 0 .

20

Nguyn Hu in, HKHTN H Ni

Ta s chng minh nhng im ny thng hng vi Q. Tuy nhin theo nh l


Pascal cho lc gic suy bin AADBBC th Q v giao im ca AC vi BD s
cng tuyn.
Tng t, p dng nh l Pascal cho lc gic suy bin ADDBCC ta cng c
kt qu tng t.
.4.20. Cho A = {1, 2, ..., 17} v hm f : A A tho mn
K hiu f [1] = f (x) v f [k+1] (x) = f (f [k] (x)) vi k N

Tm s t nhin ln nht M sao cho tn ti song nh f : A A tha mn


iu kin sau:
a) Nu m < M v 1 i 17 th f [m] (i + 1) f [m] (i) khng ng d vi 1
theo mun 17 b) Vi 1 i 17 th

f [m] (i + 1)f [m] (i) 1(mod17)


( y f [k] (18)) c xc nh bng f [k] (1)))

.c

Li gii: nh x f (x) = 3x(mod17) tha mn yu cu cho M = 8 v ta s


ch ra rng n l gi tr ln nht.

Ch rng bng cch sp xp vi chu trnh chuyn ta c th gi s rng


f (17) = 17, do M l s nguyn u tin sao cho f [M ] (1)) bng 1 hoc bng
16. Cng nh vy cho 16.

Nu 1 v 16 cng trn mt qu o ca hon v f th qu o ny c di


ln nht l 16 v 1 hoc 16 phi nh x vi nhng gi tr khc nhau sau 8

bc. Suy ra, M 8.


Nu c mt qu o khc, mt hoc thm ch hai qu o c di ln nht
l 8 v nh vy M 8.

.4.21. Cho a1 , a2 , ... l cc s khng m tha mn


am+n am +an (m, nN).

n
1)am vi mi nm.
Chng minh rng: an ma1 + ( m

Li gii: Bng phng php quy np vi k


an kam +anmk vi k < m
n

t n = mk + r vi r{1, 2, ..., m} th
a +ar ma1
an kam +ar = nr
m m
(Do am ma1 v ar ra1 )

Chng 5
thi olympic Colombia
m

.5.22. Cho mt bng k kch thc n n v 3 mu. Ta s t mu mi on

.c

ca li bi mt trong 3 mu trn sao cho mi vung n v c 2 cnh cng


mu v 2 cnh cn li cng mu khc. Hi c bao nhiu cch t mu c th?

Li gii: Gi 3 mu trn l A, B, C.
Gi an l s cch t mu ca 1 n dng u tin ca bng.

Vi n = 1, gi s WLOG on trn cng ca bng c t mu A, khi c


3 cnh chn on khc c t mu A v c 2 cch chn mu cn li v
nh vy c tt c a1 = 6 cch t mu.

By gi ta tm an+1 t an . Cho bt k mu no ca dng 1 n, gi thit


WLOG rng on bn phi nht c t mu A. By gi ta tng tng thm
mt vung n v kp vo cnh phi ca dng c dng mi c kch
thc 1 (n + 1), mu trn ca vung mi bit. Nu on trn

mi c t mu A th s c 2 cch chn mu cho 2 on cn li. Do vy:


a( n + 1) = 2an v an = 3.2n .
Tr li yu cu ban u, c 3n cch t mu gc trn v 3.2n cch t mu
mi dng. Nh vy c tt c 3m+n .2m.n cch t mu.

.5.23. Ta chi mt tr chi vi tam gic u ca n.(n+1)


ng xu (vi n ng
2
xu trn mi cnh). u tin, tt c cc ng xu u t sp. Trong mi ln
lt ta c th lt 3 ng xu lin tip lin k. Mc ch l tt c cc ng xu bi

22

Nguyn Hu in, HKHTN H Ni

lt nga. Hi n bng bao nhiu hon thnh vic ?


Li gii: Tr chi ny s c hon thnh vi cc gi tr ca n m chia 3 d
0 hoc 2.
R rng thy ngay trng hp n gin nht. Bi ton ng vi n = 2 v
n = 3 (mi trng hp c bn kh nng lt).
Vi cc gi tr n ln hn, chn mi ln lt 3 ng xu, s ng xu cn d c
lt mt ln, v nhng ng xu dc theo cc cnh ca tam gic c th c
lt 3 ln. V vy tt c cc ng xu u nga. Trong khi , mi ng xu bn
trong tam gic c lt 6 ln, v chng lp thnh mt tam gic c s ng xu
trn mi cnh l n 3.
Theo phng php quy np, cc gi tr n nh trn u tha mn.

Nu n chia 3 d 1, ta t cc ng xu bi cc mu vng, v xanh sao cho


bt k 3 ng xu no cnh nhau cng c mu khc nhau. Cng vy, 3 ng

xu lin tip bt k trn mt hng cng c mu khc nhau.


Nu cc ng xu gc u c mu vng th s ng xu mu vng nhiu hn

.c

s ng xu mu xanh hoc mu l 1 ng.


Lc ny tnh chn, l ca s ng xu nga mu vng khc vi tnh chn, l

ca s ng xu nga mu .
T s khc nhau v tnh chn, l ca s ng xu nga ca mi mu, chng ta
khng th kt thc nu khng c s nh nhau v tnh chn, l ca s ng xu

nga mu vng v mu . c th l mt trng hp nu tt c cc ng


xu u lt nga. Vy cc ng xu khng th c sp xp.

.5.24. Cho ABCD l mt hnh vung c nh. Xc nh tt c cc v tr c


th ca S hnh vung P QRS vi P v R nm trn 2 cnh khc nhau ca
ABCD; Q nm trn ng cho ca ABCD. Xc nh tt c cc v tr c th
ca im S.
Li gii: Cc v tr to thnh cc hnh vung khc nhau, quay 450 vi tm l
giao ca hai ng cho ca hnh vung.
Gi s ta a ra cc s phc sao cho A = 0; B = 1; C = 1 + i v D = i.
Trc tin, gi s P v R nm trn 2 cnh lin nhau ca ABCD. Khng mt
tnh tng qut, gi s P nm trn AB v R nm trn BC. Trong trng hp
ny Q phi nm trn AC. (Vi bt c im no nm trn BD khng trng

23

thi olympic Colombia

vi tm ca hnh vung, php quay vi gc quay 900 m AB khng trng vi


AD.) Nu P = x; Q = y + yi th R = (2y x)i v S = (x y) + (y x)i,
trong cc bin dc theo hnh vung cho.
Nu P v R nm trn 2 cnh i din ca ABCD, khng mt tnh tng qut,

ta gi s P nm trn AB, R nm trn CD v Q nm trn AC. Hn na, ta gi


thit Q = y + yi vi 21 y 1. Quay cnh AB mt gc 900 ta c Q trng vi

mt im duy nht trn CD. Nh vy P = 2y 1; R = i v S = y 1+(1y)i


vi cc bin dc theo hnh vung cho.
.5.25. Chng minh rng tp hp cc s nguyn dng c th chia thnh v hn
cc tp c v hn s A1 , A2 , . . . , (cc tp ri nhau) sao cho nu x, y, z, w thuc
Ak vi k no , khi x y v z w cng thuc tp Ai (trong i khng
=

z
.
w

.c

x
y

nht thit bng k) khi v ch khi

Li gii: Gi Ak l tp bao gm tt c cc s c dng (2k 1)2n v cch phn

chia ny s tha mn yu cu ra.


Tht vy, gi s x, y, z, w Ak vi x > y v z > w Ta c:

x = (2k 1)(2( a + b)), y = (2k 1)2a , z = (2k 1)2( c + d), w = (2k 1)2c .
Khi
x y = (2k 1)(2b 1)(2a ), z w = (2k 1)(2d 1)(2c )

Do xy = 2b ; wz = 2d , xy = wz khi v ch khi b = d khi v ch khi x y v z w


c c s l chung ln nht.

Chng 6
thi olympic Czech v Slovak

Repubulick
.c

.6.26. Cho tam gic ABC c ba cnh ln lt l a, b, c v ba gc tng ng

, , . Chng minh rng: nu = 3 th (a2 b2 ) (a b) = bc2 , xt xem


chiu ngc li c ng khng.

Li gii: Theo h qu ca nh l Sin ta c a = 2R sin , b = 2R sin , c =


2R sin , vi R l tm ng trn ngoi tip tam gic ABC. V vy:



a2 b2 (a b) = 8R3 sin2 sin2 (sin sin )

= 8R3 sin2 3 sin2 (sin 3 sin )
= 8R3 (sin 3 sin )2 (sin 3 + sin )

= 8R3 8 cos2 2 sin2 sin2 cos

= 8R3 sin2 1800 4 (sin )

= 8R3 sin2 (sin )

= bc2

Ni chung, chiu ngc li sai; ta c th ly = 3 360o , v d nh =

15o , = 125o , = 40o

25

thi olympic Czech v Slovak Repubulick

.6.27. Mi cnh v ng cho ca mt n-gic u (n 3) c t mu


hoc mu xanh. Ta chn mt nh v thay i mu ca cc on thng nhn
im lm u mt t mu thnh mu xanh v ngc li. Chng minh
rng, vi bt k cch t mu lc u th no, ta vn c th bin s cnh mu
xanh xut pht t mi nh l s chn. Chng minh rng, kt qu cui cng
ca vic t mu c quy nh da trn cch t mu ban u.
Li gii: Nhn thy, th t chn cc nh khng nh hng n kt qu t
mu cui cng. V vic chn mt nh hai ln khng nh hng n kt qu t
mu. V th, vic chn mt tp hp cc nh cng cho kt qu nh vic chn
cc nh cn li: Qu trnh sau cng tng t nh vic chn mt tp hp cc
nh u tin, sau chn tt c cc nh ( y, trong tp hp cc nh cn
li, nhng nh ban u c chn s l ln by gi c chn theo s chn

.c

ln v ngc li).
t tn cc nh l 1, 2, ..., 2n + 1. Gi ai l s cc on mu xanh xut pht
P
t nh th i, gi bi l s ln mi nh c chn v B = bi . Khi chn nh
k th ak tr thnh 2n a ak ; mt khc, mi on t nh k ti mt nh

khc i mu nn ai cn li thay i tnh chn l.


Tnh tng s ai th cho ra kt qu l hai ln tng s cc on mu xanh, v
th c mt s chn cc nh vi ai l s l -gi l 2x cc nh. Chn cc nh

ny. Tnh chn l ca cc s ai thay i 2x 1 ln thnh s chn. Tnh


chn l ca cc s ai cn li th thay i 2x ln gi nguyn l s chn. Do

, tt c cc nh u c mt s chn cc on mu xanh. Vy ta chng


minh c kt qu t mu cui cng l duy nht.
Ta xt mt cch t mu vi kt qu nh mong mun. Cui cng, s on mu
xanh ai xut pht t nh th i l ai + B bi (mod2). Khi , s on mu
xanh xut pht t cc nh l bng nhau, do , bj bk khi v ch khi lc u
aj ak . V vy, hoc bi 1 khi v ch khi ai 1 hoc l bi 1 khi v ch khi
ai 0, ta c kt qu t mu nh trn. Do kt qu t mu l duy nht.

Bi ton c chng minh


Ch : vi mt 2n-gic (n 2), th vic chn mt nh s lm thay i tnh
chn l ca tt c cc ai . V th, ta khng th c c kt qu tt c cc ai l
s chn, nu cc ai ban u khng cng tnh chn l. V nu c tt c cc ai
l s chn th kt qu t mu cui cng l khng duy nht.

26

Nguyn Hu in, HKHTN H Ni

.6.28. Cho t din ABCD c chia thnh 5 khi a din li sao cho mi mt
ca t din ABCD l mt mt ca khi a din (khng c mt no b chia),
v hai khi a din bt k trong 5 khi a din hoc c mt nh chung, hoc
c mt cnh chung hoc c mt mt chung. Hi 5 khi a din c tng s mt
t nht l bao nhiu?
Li gii: Tng s mt nh nht l 22. Khng c khi a din no c chung
hai mt vi t din ABCD, nu khng, do tnh li ca khi a din nn n s
l ABCD. Do , c mt khi a din P khng c chung mt mt vi ABCD
v cc mt ca n nm bn trong t din ABCD. Do , mi mt ca P phi
l mt chung ca P vi mt khi a din khc, c ngha l P c chung t nht
3 mt vi mt trong nhng khi a din cn li. ng thi, bt k mt no

ca khi a din khng l mt mt ca t din ABCD th phi l mt mt


ca khi a din khc. Tc l, tng s cc mt ca 5 khi a din l s chn.

Do mi khi a din phi c t nht 4 mt, nn tng s mt t nht l 20. Gi


thit, y l tng. Khi mi khi a din l mt t din c 4 nh v P c t

.c

nht 2 nh chung vi ABCD. V nu n c 2 im chung vi ABCD, gi s


l A v B, khi , n s c ti a 2 nh chung vi t din m 3 trong 4 nh

l A, C, D. iu ny l v l. Do , tng ca cc mt ln hn hoc bng 22.


Ta s ch ra trng hp du bng xy ra. Ln lt ly P v Q gn vi A v
B. Khi 5 khi a din AP CD, P QCD, BQCD, ABDP Q v ABCP Q tha

mn cc iu kin bi m tng s mt ca 5 khi a din ny bng 22.

.6.29. Ch ra rng tn ti mt dy cc s t nhin tng dn {an }


n=1 vi mi

k 0, m dy {k + an } ch cha mt s hu hn s nguyn t.

Li gii: Ly pk l s nguyn t th k, k 1 . Chn a1 = 2. Vi n 1 ,


ly an+1 l s nguyn nh nht ln hn an m an+1 p (modpk+1 ) vi mi
k n. Nhng s nguyn ny tn ti theo nh l Thng d Trung Hoa. V
vy, vi mi k + an 0 (modpk+1 ) vi n k + 1 . Do , trong dy {k + an },
gi tr ln nht trong k + 1 c th l s nguyn t; t s hng th k = 2 tr

i, cc s hng l bi ca pk+1 v phi l hp s .Ta c iu phi chng minh.


.6.30. Vi mi s t nhin n 2, hy tm gi tr ln nht ca biu th sau:
Vn = sin x1 cos x2 + sin x2 cos x3 + + sin xn cos x1

27

thi olympic Czech v Slovak Repubulick


vi x1 , x2 , , xn l cc s thc bt k.

Li gii: p dng bt ng thc 2ab a2 + b2 ta c:


Vn 6

sin2 xn + cos2 x1
n
sin2 x1 + cos2 x2 sin2 x2 + cos2 x3
+
++
=
2
2
2
2

Du bng xy ra khi x1 = x2 = = xn =

\ = .
.6.31. Cho hnh bnh hnh ABCD m ABD l tam gic nhn, v BAD
4
Trn cc cnh ca hnh bnh hnh, ly cc im K thuc AB, L thuc BC, M
thuc CD, N thuc DA sao cho KLMN l t gic ni tip c bn knh bng
bn knh ng trn ngoi tip cc tam gic ANK v CLM. Tm qu tch cc
giao im ca ng cho ca t gic KLMN.

\ , KMN
\ , LKM
\ , LNM
\ trn ng trn
Li gii: Do cc cung cha cc KLN
\ , LCM
\ ln lt trn ng
ngoi tip t gic KLMN v cc cung cha KAN

.c

trn ngoi tip tam gic AKN v CLM c cng s o, cc gc u


bng nhau v c cng s o l 45o . Cc tam gic SKL v SMN vi S l

giao im ca KM v NL, l cc tam gic vung cn ti S v ng dng


vi nhau. Khi , qua php ng dng, K bin thnh M, L bin thnh N,

AB bin thnh CD v BC bin thnh DA, v vy S nm trn on BD.

Chng 7

thi olympic Php

.7.32. Ti mi nh ca 1997- gic c gn mt s nguyn, sao cho tng ca


chng bng 1. Bt u t mt nh no , ta gn theo chiu ngc kim ng

.c

h quanh a gic. Hi c th chn mt nh bt u m tng ca k s nguyn


u tin l dng vi k = 1, 2, .., 1997
Li gii: C. Gi bk l tng ca k s nguyn u tin, ta c b1997 = 1. Gi x

l gi tr nh nht ca bk ta tm mt s k ln nht m bk1 = x. Sau ta


bt u t k nh th mi tng s s l s dng.
.7.33. Tm th tch ln nht ca mt hnh tr c cha trong phn chung ca

mt hnh cu tm O bn knh R v mt hnh nn nh O ct hnh cu theo


mt ng trn bn knh r, c cng trc vi hnh nn.
Li gii: Ta c hnh tr ct hnh cu theo mt ng trn bn knh s < r.
Khong cch t tm ca hnh cu n mt phng cha ng trn ny l

R2 s2 . Li c hnh tr cng ct hnh nn theo mt ng trn bn knh


s, khong cch t tm hnh cu n mt phng cha ng trn bng
p
s R2 /r 2 1. (V khong cch t tm ca hnh cu n mt phng cha

ng trn l R2 r 2 ). V vy, th tch ca hnh tr l:




p
2
2
2
2
2
R s s R /r 1
s
Chng ta tm gi tr ln nht ca biu thc trn bng cch cho o hm theo

29

thi olympic Php


s bng 0:

s3
0 = 2s R2 s2
3s2 R2 /r 2 1
R 2 s2

Chuyn v v bnh phng ta c:

s4 4R2 s2 + 4R4
9s2 R2 s2 r 2
=
R 2 s2
r2
Gii phng trnh ta c:
s2 =

3R2 + r 2 +

(9R2 r 2 ) (R2 r 2 )
6

V thay s2 vo cng thc th tch trn cho ta th tch ln nht.


.7.34. Tm din tch ln nht ca hnh chiu vung gc ca hnh lp phng

n v ln mt mt phng.

Li gii: Nhn thy hnh chiu ca hnh lp phng l tng hnh chiu ca

.c

3 mt ca hnh lp phng i mt vung gc vi nhau. Din tch hnh chiu


ca mi mt bng gi tr tuyt i ca tch hai vect n v ln lt vung

gc vi mt v mt phng chiu.
Nh vy nu cc tch l x, y, z th gi tr ln nht ca din tch hnh ca
chiu hnh lp phng bng gi tr ln nht ca tng x+y +z vi iu kin x2 +
p
y 2 +z 2 = 1. Mt khc, theo bt ng thc Cauchy-Shwarz 3 x2 + y 2 + z 2
(x + y + z). Du bng xy ra khi v ch khi x = y = z. Khi , gi tr ln nht

ca din tch bng 3

.7.35. Cho tam gic ABC vi a, b, c l di ca cc cnh v m, n, p l di


ca cc ng trung tuyn ca n. Vi mi s thc dng , gi () l s
thc c xc nh bi :
a + b + c = () (m + n + p )
(a). Tnh (2)
(b). Tnh gii hn ca () khi dn ti 0.
(c). Vi iu kin no ca tam gic ABC th () khng ph thuc vo .
Li gii: (a). Gi m, n, p l di ca cc ng trung tuyn tng ng vi
cc cnh a, b, c v gi s a b c. D dng tnh c m2 = (2b2 + 2c2 c2 )/4

30

Nguyn Hu in, HKHTN H Ni

v tng t vi hai trung tuyn cn li, v vy (2) =


(b). Nu x y z v 0 th

2
3

x 6 (x + y + z )1/ 6 31/ x
v do (x + y + z )1/ dn ti x. Vy chng ta c lim () =
0

(c). () khng ph thuc vo ta phi c

c 23 , n

a2
p2

= 34 , dn n a2 + c2 = 2b2 .

.c

= b 23 , p = a 23 .
Kt hp vi gi thit ta c m =
Vy () l hng s khi tam gic ABC tha mn iu kin trn.

a
p

Chng 8

thi olympic c

.c

p + 1 = 2x2

.8.36. Xc nh tt c cc s nguyn t p sao cho h

p2 + 1 = 2y 2

c nghim x, y nguyn.

Li gii: Ch c duy nht s nguyn t p = 7 tha mn bi ton. Khng mt

tnh tng qut c th gi s rng x, y 0. Ch rng p + 1 = 2x2 l chn,


nn p 6= 2. Hn na, 2x2 1 2y 2 (mod p) suy ra x y (mod p) v p l l.
T x < y < p, ta c x + y = p. Do
p2 + 1 = 2(p x)2 = 2p2 4xp + p + 1,
nn p = 4x 1, 2x2 = 4x, x l 0 hoc 2 v p l 1 hoc 7. Hin nhin 1
khng l s nguyn t, v p = 7, (x, y) = (2, 5) l nghim ca bi ton.

.8.37. Mt hnh vung Sa ni tip mt tam gic nhn ABC vi hai nh nm


trn cnh BC v mt nh nm trn AB, mt nh nm trn AC. Cc hnh
vung Sb , Sc c xy dng tng t. Vi nhng trng hp no ca tam gic
ABC th cc hnh vung Sa , Sb , Sc l bng nhau.
Li gii:

32

Nguyn Hu in, HKHTN H Ni

xa

t R l bn knh ng trn ngoi tip tam gic ABC v t xa , xb , xc


l di cc cnh ca Sa , Sb , Sc tng ng. K hiu , , l cc gc

BAC, CBA, ACB.


Gi s rng Sa c cc nh P, Q nm trn cnh BC trong P gn B hn.

Khi

= xa cot + xa + xa cot .
2R sin
=
1 + cot + cot
2R sin sin sin
=
sin sin + cos sin + cos sin
2R sin sin sin
=
sin sin + sin

xa

.c

2R sin = BC = BP + P Q + QC

v tng t cho xb , xc . T xa = xb suy ra


sin sin + sin = sin sin + sin
0 = (sin sin )(sin 1).
T tam gic ABC l nhn, ta c sin = sin , suy ra = v trong trng
hp tri li th + = l khng th xy ra trong tam gic. Tng t nh
vy = , nn ABC l tam gic u.
.8.38. Trong mt cng vin, 10000 cy c trng theo mt hnh li vung.
Xc nh s ln nht cc cy c th n h m sao cho t mi gc cy n,
bn khng th nhn thy mt gc bt k khc. (Gi s rng bn knh ca cc
cy l khng ng k so vi khong cch ca cc cy k nhau.)

33

thi olympic c

Li gii: Ta nhn thy rng trong mt hnh vung gm bn cy k nhau ta


ch b i c nhiu nht mt cy. T li 100 100 ta c th chia 10000

nh vo 2500 hnh vung ri nhau thc s, do , c th n h nhiu nht


2500 cy.

Ta s ch ra mt cch n 2500 cy tha mn bi ton. ng nht cc cy


vi ta (x, y) trn li, 0 x, y 99, v n h tt c cc cy c cc ta

chn. Xt hai gc bt k (a, b) v (c, d) vi a, b, c, d l chn. Xt p/q l mt


biu din ca (d b)/(c a) vi cc hng t l b nht ( y p, q c cng
du vi d b, c a, tng ng), khi , t nht mt trong hai s a + p v b + q

phi l l, do , cy (a + p, b + q) s chn tm nhn gia cc cy (a, b) v (c, d).

khng ph thuc vo v tr im P trn cung AB.

.8.39. Cho mt hnh vin phn AMB vi gc trung tm AMB nh hn 90o .


T mt im P bt k trn cung AB h cc ng vung gc P C v P D xung
MA v MB (C MA, D MB). Chng minh rng di on thng CD

.c

Li gii:

T P CM = P DM = /2, nn t gic P CMD ni tip ng trn ng


knh MP . Do , p dng nh l hm s sin cho tam gic MCD, ta c,
CD = MP sin CMD l mt hng s.
.8.40. Trong mt hnh vung ABCD xy dng bn cung trn vung, mi cung
trn c tm tng ng l A, B, C, D v cha hai nh k vi tm. Bn cung
trn ny ct nhau ti bn im E, F, G, H nm bn trong ABCD, cc im

34

Nguyn Hu in, HKHTN H Ni

ny cng lm thnh mt hnh vung gi l S. Gi C l ng trn tip xc vi


c bn cung trn trn. Hy so snh din tch ca S v C.

Li gii: K hiu [C] l din tch ca cung trn C, v [S] l din tch ca hnh
vung S. Khng mt tnh tng qut c th coi E l giao im ca cc cung

trn m gn AB nht, v G l giao im gn CD nht. H cc ng vung


gc EE 0 xung AB, v GG0 xung CD. Theo tnh cht i xng, E 0 , E, G, G0
l thng hng.

T AB = BG = AG ta c ABG l tam gic u v ng cao GE 0 = 3AB/2.

Tng t, G0 E = 3AB/2. Do , 3AB = GE 0 + G0 E = AB + EG, nn

EG = ( 3 1)AB v [S] = EG2 /2 = (2 3)AB 2 .


t I, K l cc tip im ca ng trn C vi cc cung trn tm C v A,

.c

tng ng. Li do tnh i xng th A, I, K, C thng hng. Khi

2AB = AK + IC = AC + IK = 2AB + IK,

nn, IK = (2 2)AB. Do vy

2)
(3

AB 2 > (2 3)AB 2 = [S].


[C] = IK 2 =
4
2

Vy din tch ca ng trn C ln hn din tch ca hnh vung S.

E0

E
G

G0

I
H
A

.8.41. K hiu u(k) l c l ln nht ca s t nhin k. Chng minh rng


n

2
2
1 X u(k)

.
2n k=1 k
3

35

thi olympic c

Li gii: t v(k) l c ln nht ca k c dng ly tha ca 2, nn u(k)v(k) =


k. Trong {1, 2, ..., 2n} c 2ni1 gi tr ca k sao cho v(k) = 2i vi i n 1,
v mt gi tr sao cho v(k) = 2n . Do , v tri bng
n

2
n1 ni1
X
1 X u(k)
1
2

= n+
.
n
2
k
4
2n+i
i=0
k=1

T tng ca chui hnh hc ta c


n

2
1 X u(k)
2
2
n
n

=
4
+
(1

4
)
>
.
2n k=1 k
3
3

.8.42. Tm tt c cc s thc tha mn h

y 3 = 2z 1

.c

z 3 = 2x 1.

x3 = 2y 1

Li gii: Trc ht ta ch ra rng x = y = z. Gi s tri li rng x 6= y. Nu


x > y, th y = (x3 + 1)/2 > (y 3 + 1)/2 = z, nn y > z, v tng t z > x, mu

thun. Tng t, nu x < y th y < z v z < x, mu thun. Nn cc nghim


ca h phng trnh c dng x = y = z = t vi t l nghim ca phng trnh
t3 = 2t 1. Vy, nghim ca h phng trnh l



1 + 5 1 5
.
,
x = y = z = t, t 1,
2
2
.8.43. nh ngha cc hm s
f (x) = x5 + 5x4 + 5x3 + 5x2 + 1
g(x) = x5 + 5x4 + 3x3 5x2 1.
Tm tt c cc s nguyn t p m tn ti s t nhin 0 x < p, sao cho c
f (x) v g(x) u chia ht cho p, v vi tng gi tr ca p, hy tm tt c cc
gi tr ca x tng ng.

36

Nguyn Hu in, HKHTN H Ni

Li gii: Ch rng
f (x) + g(x) = 2x3 (x + 1)(x + 4).
Nn nu p l c ca f (x) v g(x) th v p l s nguyn t nn p phi l c
ca t nht mt trong cc s sau 2, x, x + 1, x + 4. T f (0) = 1 v f (1) = 17
dn n p 6= 2. Hn na, p cng khng th l c ca x, v khi , f (x) 1
(mod p) l v l. Nu p l c ca x + 1 th f (x) 5 (mod p), ko theo, p l

c ca 5 nn p = 5 v c ngay rng x = 4. Trng hp p l c ca x + 4 th


f (x) 17 (mod p) nn p = 17 v d thy x = 13 l tha mn.

.c

Vy cc li gii ca bi ton l p = 5, x = 4 v p = 17, x = 13.

Chng 9

thi olympic Irland

Li gii: Ta c:

.9.44. Tm tt c cc cp s nguyn (x, y) sao cho 1 + 1996x + 1998y = xy


(x 1998) (y 1996) = xy 1998y 1996x + 1996.1998 =

.c

19972
Do 1997 l s nguyn t, nn ta c: x 1998 = 1; 1997; 19972. Vy c 6

gi tr (x, y) tha mn l



(x, y) = 1999, 19972 + 1996 , 1997, 19972 + 1996 ,



(3995, 3993) , (1, 1) 19972 + 1998, 1997 , 19972 + 1998, 195

.9.45. Cho ABC, M l im trong tam gic. Goi D,E,F ln lt l hnh


chiu ca M xung BC, CA, AD. Tm tp hp tt c cc im M tha mn
\
F
DE = .
2

\ = MCE
\ v
Li gii: T cc t gic ni tip MDBF v MDCE ta c MDE
\ + MBC
\ = hay BMC
\ = 5
\ = MBE
\ do F
\
MDF
DE = 2 MCB
6
6
M nm trn cung trn i qua B v C.
.9.46. Tm tt c cc a thc P (x) sao cho i vi mi x ta c :
(x 16) P (2x) = 16 (x 1) P (x) .

38

Nguyn Hu in, HKHTN H Ni

Li gii: Goi d = degP v a l h s ca x trong P (x) vi s m ln nht.


Khi h s ca x m ln nht bn tri l 2d a phi bng 16a do d = 4
Do v phi lc ny chia ht cho (x 1), nhng trong trng hp v phi li
chia ht cho (x 2), tng t l chia ht cho (x 4) v (x 8). Vy a thc

P (x) l bi ca (x 1)(x 2)(x 4)(x 8) l tt c cc a thc tha mn.

.9.47. Cho a, b, c l cc s thc khng m sao cho a + b + c abc. Chng minh


rng a2 + b2 + c2 abc.
Li gii: Gi s phn chng rng vi a, b, c > 0 m a2 + b2 + c2 < abc do
abc > a2 a < bc. Lm tng t ta cng c b < ca, c < ab. Do abc
a2 +b2 +c2 ab+bc+ca. Theo bt ng thc AM-GM v ab+bc+ca > a+b+c
suy ra abc > a + b + c. Tri vi gi thit. Vy bi ton c chng minh.

.c

i vi a, b S ta nh ngha php ton

.9.48. Cho tp hp S = {3, 5, 7, ...}. Vi mi x S ta t (x) l xc nh mt


s nguyn duy nht sao cho: 2(x) < x < 2(x)+1

a b = 2(a)1 (b 3) + a

a, Chng minh rng nu a, b S th a b S


b, Chng minh rng nu a, b, c S th (a b) c = a (b c).

Li gii: a, Hin nhin

b, Nu 2m < a < 2m+1 , 2n < b < 2n+1 th

a b = 2m1 (b 3) + a 2m1 (2n 2) + 2m + 1 = 2n+m1 + 1


v a b 2m1 (2n+1 4) + 2m+1 1 = 2m+n 1. V vy (a b) = m + n 1
Nu 2p < c < 2p+1 th


(a b) c = 2m1 (b 3) + a c = 2m+n2 (c 3) + 2m1 (b 3) + a



a(b c) = a 2m1 (c 3) + b = 2m1 2n1 (c 3) + b 3 +a = (a b)c.
.9.49. Cho t gic li ABCD c mt ng trn ni tip. Nu
A=B=

2
, D = , BC = 1
3
2

39

thi olympic Irland


Tm di AD

Li gii: Goi I l tm ng trn ni tip . Do ABC l tam gic u,


[ = 1050 , ICB
[ = 150 , AID
[ = 750, IDA
[ = 450 nn
BIC
AD =

BI AD
sin 150 sin 750
=
= 2 sin 150 .
BC AI
sin 1050 sin 450

.9.50. Gi A l tp con ca {0, 1, 2, ..., 1997} gm hn 1000 phn t. Chng


minh rng A ch gm nhng ly tha ca 2 hoc hai phn t phn bit c tng
l ly tha ca 2.

Li gii: Gi s tp A khng tha mn bi ton. Khi A s bao gm hn


na s nguyn t 51 ti 1997 m chng c chia thnh tng cp c tng l
2048 (V D : 51 + 1997 = 2048...). Tng t nh vy, A bao gm nhiu nht

.c

na s nguyn t 14 ti 50, gm nhiu nht na s nguyn t 3 ti 13, v c


th c s 0, do A c tng cng 937 + 18 + 5 + 1 = 997 s nguyn, tri vi

gi thit A gm hn 1000 s nguyn t tp {0, 1, 2, ..., 1997}.

.9.51. Xc nh s t nhin n tha mn nhng iu kin sau:


a, Khai trin thp phn ca n gm 1000 s

b, Tt c cc s trong khai trin l s l.


c, Hai phn t bt k lin nhau trong khai trin ca n hn km nhau 2 n v

Li gii: t an , bn , cn , dn , en l s trong khai trin ca n, l nhng s l


v hai s lin tip khc nhau 2 n v do tn cng theo th t l 1, 3, 5, 7, 9
do

an+1
an
0 1 0 0 0

1 0 1 0 0 bn bn+1

0 1 0 1 0 cn = cn+1

0 0 1 0 1 dn dn+1

en+1
en
0 0 0 0 1
Gi A l ma trn vung trong biu thc . Ta tm gi tr ring ca ca A,

40

Nguyn Hu in, HKHTN H Ni

gi s Av = v vi v = (v1 , v2 , v3 , v4 , v5 ). Do
v2 = v1
v3 = v2 v1 = (2 1) v1

v4 = v3 v2 = (3 2) v1

v5 = v4 v3 = (4 32 + 1) v1

v v4 = v5 , do 5 33 + = 3 2. Gii pt ny ta c = 0, =

1, = 3 tng ng ta c cc vect ring x1 , x2 , x3 , x4 , x5 l





(1, 0, 1, 0, 1) , (1, 1, 0, 1, 1) , (1, 1, 0, 1, 1) , 1, 3, 2, 3, 1
v

2 3
1 2+ 3
x4 +
x5
(1, 1, 1, 1, 1) = x1
3
6
6

.c

V vy

(a1000 ,b1000 , c1000 , d1000 , e1000 ) =




 2 3

999 2 +
3
2
=3
1, 3, 2, 3, 1
1, 3, 2, 3, 1
6
6

= 3499 , 2.3499 , 2.3499 , 2.3499 , 3499

V th kt qu ca bi ton l 8.3499 .

Nguyn Hu in

OLYMPIC TON NM 1997-1998

.c

49 THI V LI GII
(Tp 5)

NH XUT BN GIO DC

.c

Li ni u
th gi lnh lamdethi.sty ti bin son mt s ton thi Olympic,
m cc hc tr ca ti lm bi tp khi hc tp LATEX. ph v cc bn
ham hc ton ti thu thp v gom li thnh cc sch in t, cc bn c th
tham kho. Mi tp ti s gom khong 51 bi vi li gii.

Rt nhiu bi ton dch khng c chun, nhiu im khng hon ton

.c

chnh xc vy mong bn c t ngm ngh v tm hiu ly. Nhng y l


ngun ti liu ting Vit v ch ny, ti c xem qua v ngi dch l
chuyn v ngnh Ton ph thng. Bn c th tham kho li trong [1].

H Ni, ngy 2 thng 1 nm 2010


Nguyn Hu in

Rt nhiu on v mi hc TeX nn cu trc v b tr cn xu, ti khng


c thi gian sa li, mong cc bn thng cm.

51
89/176-05
GD-05

M s: 8I092M5

Mc lc
3

Mc lc. . . . . . . . . . . . . . . . . . . . . . . . . . . . . . . . . . . . . . . . . . . . . . . . . . . . . . .

Chng 1. thi olympic Hy Lp . . . . . . . . . . . . . . . . . . . . . . . . . . . . . . . .

Chng 2. thi olympic Hungary . . . . . . . . . . . . . . . . . . . . . . . . . . . . . . .

Chng 3. thi olympic Iran . . . . . . . . . . . . . . . . . . . . . . . . . . . . . . . . . . .


Chng 4. thi olympic Ireland . . . . . . . . . . . . . . . . . . . . . . . . . . . . . . .

18

15

Chng 5. thi olympic Italy . . . . . . . . . . . . . . . . . . . . . . . . . . . . . . . . . .

22

Chng 6. thi olympic Japan . . . . . . . . . . . . . . . . . . . . . . . . . . . . . . . . .

25

Chng 7. thi olympic Korean . . . . . . . . . . . . . . . . . . . . . . . . . . . . . . .

30

.c

Li ni u . . . . . . . . . . . . . . . . . . . . . . . . . . . . . . . . . . . . . . . . . . . . . . . . . . .

Chng 8. thi olympic Poland . . . . . . . . . . . . . . . . . . . . . . . . . . . . . . . .

38

Chng 1
thi olympic Hy Lp

.c

.1.1. Cho P l mt im nm bn trong hay trn 1 cnh bt k ca hnh vung


ABCD. Hy xc nh gi tri ln nht v gi tr nh nht c th c ca hm s

[+[
[ + DAP
[
f ( P) = ABP
BCP + CDP

Li gii:

m
n
v

C
B
t cc nh ca hnh vung tng ng vi cc gi tr 1, i, -1, -i trong mt
phng v coi P l s phc z. Khi f(P) l argument ca s phc z tho mn
z4 1
z1 zi z+1 z+1
=
i + 1 1 i i + 1 1 + i
4
z4 1
Khi | P| 1,
chy trn min phng c gii hn bi ng trn bn
4
knh 1/4, tm c to -1/4. Do gi tr ln nht ca gc t c ti 1
im trn bin ca hnh trn trn, iu xy ra khi P nm trn cnh ca
hnh vung. Do vai tr ca cc cnh l nh nhau, khng mt tng qut ta c
th gi s cnh l AB.

Nguyn Hu in, HKHTN H Ni

.c

[ gim t n ; [
BCP gim t
n 0;
Khi P chy t A n B th CDP
2
4
4

Hai gc cn li nhn cc gi tr l v 0.
2
5
3
Vy ta c gi tr ln nht v nh nht ca f ( P) ln lt l
v
4
4
.1.2. Cho hm f : (0; ) R tho mn cc iu kin sau:
(a) f tng nghim ngt
1
vi mi x>0
(b) f(x)>
x
1
(c) f(x)f(f(x)+ )=1 vi mi x>0
x
Tnh f(1).
1
1
Li gii: t k=f(x)+ . V k>0 nn f(k)f(f(k)+ )=1
x
k
1
1
1
+
Mt khc f(x)f(k)=1. Do f(x)=f(f(k)+ )=f(
)
1
k
f (x)
f (x) +
x
1
1
+
Do f tng nghim ngt nn ta c x=
1
f (x)
f (x) +
x

1 5
.
Gii ra ta thu c f(x)=
2x

1 5
D dng kim tra c rng ch c
tho mn cc yu cu ca bi.
2x

1 5
Do f(1)=
2
.1.3. Tm tt c cc s nguyn tho mn phng trnh sau:
z
13 1996
+ 2 =
2
x
y
1997

Li gii: t d=gcd(x,y), t x=dx1 , y=dy1


Khi phng trnh cho tng ng vi
1997(13)y21 +1997(1996)x21 =d2 zy21 x21
Khi x1 v y1 nguyn t cng nhau, ta phi c x21 |1997 13,

y21 |1997 1996


D dng kim tra c rng 1997 khng phi s chnh phng v r rng
n nguyn t cng nhau vi 13 v 1996. Hn na 1996 = 22 .499, v cng d
dng kim tra c rng 499 khng phi s chnh phng.

thi olympic Hy Lp
Khi (x1 , y1 ) = (1, 1) hoc (1,2)
Bi ton c chia thnh 2 trng hp:
* Trng hp 1: (x1 , y1 ) = (1, 1). Khi

d2 z = (13 + 1996)1997 = 1997.72.41


Khi 1997 nguyn t cng nhau vi 7 v 41 th d=1,7. T ta c kt qu ln
lt l:
(x,y,z)=(1,1,4011973), (7,7,81877)
* Trng hp 2: (x1 , y1 ) = (1, 2).Khi d2 z = (13 + 499)1997 =
1997.29
Do d=1,2,4,8,16. Ta li c cc kt qu ln lt l:

(x,y,z)=(1,2,1022464),(2,4,255616),(4,8,63904), (8,16,15976), (16,32,3994)


l cc kt qu thu c.

.c

.1.4. Cho P l mt a thc vi cc h s nguyn c 13 nghim nguyn phn bit.


Hy ch ra rng nu n Z khng phi l nghim ca P th | P(n)| 7(6!)2 . Hy
cho 1 v d khi du bng xy ra.

Li gii: Phn tch a thc vi cc h s nguyn thnh tch ca cc a thc


cng c h s nguyn vi bc nh hn th P(x) c th vit di dng
( x r1 )( x r2 )...( x r13 )Q( x )

trong rs l 1 trong 13 nghim phn bit ca a thc .


Do vi tt mi s nguyn x, P(x) c gi tr bng tch ca 13 s nguyn

phn bit vi 1 s nguyn khc.


R rng gi tr tuyt i nh nht ca kt qu trn l

|(1)(1)(2)(2)...(6)(6)(7)(1)| = 7(6!)2 .
T ta c iu phi chng minh.
Mt v d khi du bng t c l khi x = 0
v P(x)=(x+1)(x-1)(x+2)(x-2)...(x+7)

Chng 2
thi olympic Hungary

.c

.2.5. Mi thnh vin trong hi ng xp hng cc ng vin A, B, C theo th t.


iu ch ra rng phn ln cc th hng hi ng A cao hn nhiu so vi B v
cng c th l phn ln cc th hng B cao hn nhiu so vi C. C phi m theo
A cao hn C.
Li gii: Khng. Gi s gi hi ng c ba thnh vin, mt trong nhng

ngi xp hng A > B > C, mt trong nhng ngi xp hng B > C > A,
v l mt trong nhng ngi xp hng C > A > B. Sau , th nht v th

ba c hai thch A n B, v th nht v th hai thch c hai B to C, nhng


ch l ngi u tin thch A n C.

.2.6. Cho php a, b, c c cc bn,


m a , mb , mc
l cc cao, v
d a , db , dc
l cc khong cch t nh vo trong mt trng tm tam gic. Chng minh rng.
m a d a + mb db + mc dc =

a2 + b2 + c 2
2

thi olympic Hungary

Li gii: Cho D, E, F l chn ca chiu cao t A, B, C tng ng, v cho H


l trc tm ca hnh tam gic ABC, Sau hnh tam gic ACD l ging vi
hnh tam gic AHE. Vy
m a d a = AD AH = CE AE = AE b.
Tng t hnh tam gic ABD l ging vi hnh tam gic AHF. Vy
m a d a = AD AB = AF AE = AB c.
Do
ma da =

AE b + AF c
2

mb db =

BF c + BD a
2

mc dc =

CD a + CE b
2

Tng t

.c

Do

m a d a + mb db + mc dc

1
= ( AE b + AF c + BF c + BD a + CD a + CE b)
2
1
= (( BD + CD ) a + (CE + AE) b + ( AF + BF) c)
2
a2 + b2 + c 2
=
2

.2.7. Cho R l bn knh hnh tam gic ABC v G, H l trng tm v trc tm


tng ng. Cho F l trung im ca GH.
AF2 + BF2 + CF2 = 3R2

10

Nguyn Hu in, HKHTN H Ni

Li gii: Chng ta s dng Vector vi gc ta ti tm hnh tam gic


ABC. Sau chng ta c cng thc H = A + B + CvG = H/3. Vy F =

(G + H )/2 = 2H/3v2( A + B + C) = 3F.


Do
AF2 + BF2 + CF2
=(A-F).(A-F) + (B-F).(B-F) + (C-F).(C-F)

= A A + B B + C C 2( A + B + C) F + 3F F
= 3R2 F (2( A + B + C) 3F) = 3R2

.2.8. Mt hp cha 4 qu bng trng v 4 qu bng , chng ta cn v t ci hp


theo mt s th t m khng cn thay th. Trc khi v chng ta cn on mu ca
qu bng s v. Con s c mong i ca cc d on chnh xc l bao nhiu?

.c

Li gii: Con s c ch i ca cc d on chnh xc l 373/70. Cho

i, j 0, aij
biu th con s mong i ca cc d on chnh xc khi c i qu bng trng

v j qu bng . Gi s i>j>=1. Sau d on ca chng ta l chnh xc


vi xc sut i/(i+j), a ra con s mong mun ca cc d on chnh xc

ca

1 + ai 1,j

v sai vi xc sut j/(i+j), a ra con s mong mun ca


ai,j1
Vy
aij =

i
i
(1 + ai 1,j ) +
ai,j1
i+j
i+j

if
i>j
Cng vy, chng ta c
aij = a ji

11

thi olympic Hungary


cho
i, j 0
Nu
i=j1
sau chng ta on vi xc sut 1/2 v
aij =

1
1
1
(1 + ai 1,j ) + ai,i 1 = + ai,i 1
2
2
2

Nh
ai,i 1 = ai 1,i
. Cui cng, iu kin bt u l:

vi

ai0 = a0i = i

.c

i0

Chng ta c th s dng nhng phng trnh ny cho vic tnh ton

a4,4 = 373/70

.2.9. Tm tt c cc gii php cho nhng s nguyn ca phng trnh

x3 + ( x + 1)3 + ( x + 2)3 + ... + ( x + 7)3 = y3

Li gii: cc gii php l: (-2,6), (-3,4), (-4,-4), (-5,-6) Cho


P( x ) = x3 + ( x + 1)3 + ( x + 2)3 + ... + ( x + 7)3 = 8x3 + 84x2 + 420x + 784.
Nu
x0
th:

(2x + 7)3 = 8x3 + 84x2 + 294x + 343 < P( x ) < 8x3 + 120x2 + 600x + 1000 = (2x + 10)3

12

Nguyn Hu in, HKHTN H Ni

Vy 2x+7 < y < 2x100: do y l 2x + 8 hoc 2x + 9 nhng c hai phng


trnh
P( x ) (2x + 8)3 = 122 + 36x + 272 = 0
P( x ) (2x + 9)3 = 24x2 + 66x + 55 = 0
c bt k cn nguyn. Do vy nn ko c gii php bo vi
x0
Tip theo, ch rng P tha mn P (-x-7) = -P(x), vy (x.y) l 1 gii php nu
(-x-7,-y) l mt gii php. Do vy khng c gii php no vi

x 7

.c

Do vy (x.y) l mt gii php. Chng ta phi c

6 x 1

Cho

P(2) = 216 = 63

khng mt ly tha 3.

Chng ta c P(-1) = 440

3 x 1

P(3) = 64 = 43
vy (-2,6) v (-3,4) v ch cc gii php vi

3 x 1
Do (-4,-4) v (-5,-6) ch l gii php vi

6 x 4
Vy p n ch c th l (-2,6), (-3,4), (-4,-4) v (-5,-6).

13

thi olympic Hungary

.2.10. Chng ta c 1997 s nguyn dng khng trng nhau, bt k 10 trong s


c cng t nht chung. Tm s ln nht c th ca cc s nguyn t cng nhau gia
chng.
Li gii: s ln nht ca tng i s nguyn t trong tp hp ny l 9. Trc
tin, gi s c 10 s nguyn t..
n1 , n2 , ....n10
Sau t nht ca 10 thnh vin ca tp hp ny l
lcm(n1 , n2 , ....n10 ) = (n1 n2 ...n10 )
C bit, cho bt k N khc trong tp hp ny

lcm( Nn2 , ....n10 ) (n1 n2 ...n10 )

l chia ht cho

n1

Nh

.c

n1

c quan h vi

nj

cho
chi cho N.

2 j 10

Tng t
n1
chia cho N vi mi
i {2......10}
nh vy
ni
c quan h nguyn t.
n1 n2 ...n10

14

Nguyn Hu in, HKHTN H Ni

chia cho N. Nhng


N lcm( N.n2 ....n10) = n( 1)n( 2)...n( 10)
Vy chng ta phi c
N = n( 1)n( 2)....n( 10)
T y n lu gi mi thnh phn ca tp hp ca chng ta hn
n( 1).......n( 10)
Tp hp ca chng ta ch c th cha 11 thnh phn, mt s mu thun.
By gi chng ta khi to mt v d m c 9 s nguyn t.
Cho

p ( n)

.c

biu th s nguyn t th n v cho


)
(
[
p1 p2 ...p1988
|1 j 1988
{n1 , n2 ....n9 }
S=
pj

khi

n( i ) = p ( i )

vi

1i8

n( 9) = p( 9) p( 10)......p( 1988)

R rng bt k 2 thnh phn ca


n( 1).........n( 9)
l cp nguyn t.

Chng 3
thi olympic Iran

.c

.3.11. Gi s w1 , ..., wk l nhng s thc phn bit vi tng khc khng.


CMR : tn ti cc s nguyn n1 , ..., nk sao cho : n1 w1 + ... + nk wk > 0 v mt s
hon v ca {1, .. ., k} khng ng nht bng nhau. Ta c :

n1 w1(1) + ... + nk w (k) < 0

Li gii:
u tin, ta sp xp li bt ng thc :

Nu a1 < ... < an , b1 < ... < bn l nhng s thc, = min { ai +1 ai },


= min { bi +1 bi } th vi 1 vi hon v khng thng thng ca {1, ..., n}

: bi a (i ) bi ai
iu ny l hin nhin v nu i < j nhng (i ) > ( j) th thay bi
nhng s hp thnh vi s chuyn v ca i v j tng ln th tng bng


a j ai b j bi
Gi s rng w1 < ... < wk v s = | wi |
t = min {wi +1 wi } v chn 1 s t nhin N =
Ta t (n1 , n2 , ..., nk ) = ( N, 2N, ..., kN ) + p(1, ...., 1)

y p l s nguyn duy nht sao cho ni wi (0, s]


y l nh l bao hm rng 6= 1

ni w (i) ni wi N s N < 0

16

Nguyn Hu in, HKHTN H Ni

.3.12. Gi s im P di ng dc theo cung BC ca ng trn ngoi tip ABC,


v cho I1 , I2 tng ng l tm ng trn ni tip PAB, PAC. CMR :
a) ng trn ngoi tip PI1 I2 i qua mt im c nh.
b) ng trn ng knh I1 I2 i qua mt im c nh.
c) Trung im ca on I1 I2 nm trn mt ng trn c nh.
Li gii:
Cho B1 , C1 l im gia ca cc cung AC, AB. Do I1 , I2 l tm ng trn
ni tip ca cc tam gic ABP, ACP, ta c : C1 A = C1 B = C1 I1 , B1 A =
B1 C = B1 I2 .
Gi I l tm ng trn ngoi tip ABC v Q l giao im th 2 ca ng
trn ngoi tip ABC v PI1 I2 . Do C1 I1 v B1 I2 i qua P nn QI1 C1 v

QI1

tm Q vi gc v t s k.

.c

C1 I1
C1 A
1
QI2 B1 ng dng, vy : QC
QB1 = B1 I2 = B1 A khng i.
Do Q l giao ca ng trn ngoi tip ABC vi ng trn Apollonius
c nh, nn Q c nh v phn a) c chng minh.
( Bb+Cb)
\
[
T : I[
1 QI2 = I1 PI2 = C1 PB1 =
2
Cc tam gic QI1 I2 vi Q 6= P u ng dng vi nhau. V th nu M l
trung im ca I1 I2 th cc tam gic QI1 M cng u ng dng.
\1 , ngha l M l nh ca I1 qua qua cc php di hnh
Nu k = QM , = MQI

Do C1 I1 = C1 A khng i, I1 chuyn ng trn mt cung ca ng trn


c nh nn M nm trn mt ng trn c nh v (c) c chng minh.
Cui cng ta tnh c gc [
I1 I I2 = . V th ng trn ng knh I1 I2 i
2

qua I c nh v phn (b) c chng minh.

.3.13. Gi s f : R+ R+ l hm lin tc, gim sao cho x, y R+ ,


f ( x + y ) + f ( f ( x ) + f (y)) = f ( f ( x + f (y))) + f (y + f ( x ))
CMR : f ( f ( x )) = x.
Li gii:
t y = x, ta c:
f (2x ) + f (2 f ( x )) = f (2 f ( x + f ( x ))) (1)

17

thi olympic Iran


Thay x bi f ( x ) , ta c :
f (2 f ( x )) + f (2 f ( f ( x ))) = f (2 f ( f ( x ) + f ( f ( x )))) (2)
Ly (2) tr (1) , ta c :

f (2 f ( f ( x ))) f (2x) = f (2 f ( f ( x ) + f ( f ( x )))) f (2 f ( x + f ( x )))


Nu f ( f ( x )) > x th v tri ca phng trnh l m, do :
f ( f ( x ) + f ( f ( x ))) > f ( x + f ( x ))

V f ( x ) + f ( f ( x )) < x + f ( x ), mu thun.
iu mu thun tng t cng xy ra nu f ( f ( x )) < x.
Vy : f ( f ( x )) = x.

.c


.3.14. Cho A l mt ma trn gm cc s 0 v 1 i xng Aij = A ji , i, j sao cho
Aii = 1 vi mi i. Hy ch ra rng tn ti mt tp con ca cc dy m tng tt c
thnh phn ca vct l l.

Li gii:

Gi s ngc li , tn ti mt vct (v1 , ..., vn ) sao cho i vi wi = 0 vi mt


vi dy (w1 , ..., wn ) nhng vi 6= 0 (Tt c cc s y l chia ht cho 2) .

Cng trn tt c cc dy, ta c :

vi Aij v j = 0
j

Do ma trn l i xng, iu ny c quy v v2i Aii = 0 hoc vi = 0

(v1 {0, 1}), mu thun .

Chng 4
thi olympic Ireland
m

.4.15. Tm tt c cc cp s nguyn ( x, y) sao cho 1 + 1996x + 1998y = xy

.c

Li gii: Ta c:
( x 1998) (y 1996) = xy 1998y 1996x +
2
1996.1998 = 1997
Do 1997 l s nguyn t, nn ta c: x 1998 = 1; 1997; 19972. Vy c 6

gi tr ( x, y) tha mn l


 

( x, y) = 1999, 19972 + 1996 , 1997, 19972 + 1996 ,


 

(3995, 3993) , (1, 1) 19972 + 1998, 1997 , 19972 + 1998, 195

.4.16. Cho ABC, M l im trong tam gic. Goi D,E,F ln lt l hnh chiu ca
[ = .
M xung BC, CA, AD. Tm tp hp tt c cc im M tha mn FDE
2
[ v
Li gii: T cc t gic ni tip MDBF v MDCE ta c \
MDE = MCE
[ + MBC
[ = hay BMC
[ = 5 M
\
[ do FDE
[ = MCB
MDF = MBE
2
6
6

nm trn cung trn i qua B v C.

.4.17. Tm tt c cc a thc P( x ) sao cho i vi mi x ta c :


( x 16) P (2x ) = 16 ( x 1) P ( x ) .

19

thi olympic Ireland

Li gii: Goi d = degP v a l h s ca x trong P( x ) vi s m ln nht. Khi


h s ca x m ln nht bn tri l 2d a phi bng 16a do d = 4
Do v phi lc ny chia ht cho ( x 1), nhng trong trng hp v phi
li chia ht cho ( x 2), tng t l chia ht cho ( x 4) v ( x 8). Vy a

thc P( x ) l bi ca ( x 1)( x 2)( x 4)( x 8) l tt c cc a thc tha


mn.

.4.18. Cho a, b, c l cc s thc khng m sao cho a + b + c abc. Chng minh


rng a2 + b2 + c2 abc.
Li gii: Gi s phn chng rng vi a, b, c > 0 m a2 + b2 + c2 < abc do
abc > a2 a < bc. Lm tng t ta cng c b < ca, c < ab. Do

abc a2 + b2 + c2 ab + bc + ca. Theo bt ng thc AM-GM v ab + bc +


ca > a + b + c suy ra abc > a + b + c. Tri vi gi thit. Vy bi ton c
chng minh.

.c

.4.19. Cho tp hp S = {3, 5, 7, ...}. Vi mi x S ta t ( x ) l xc nh mt s


nguyn duy nht sao cho: 2(x) < x < 2(x)+1
i vi a, b S ta nh ngha php ton

a b = 2(a)1 (b 3) + a

a, Chng minh rng nu a, b S th a b S

b, Chng minh rng nu a, b, c S th ( a b) c = a (b c).

Li gii: a, Hin nhin


b, Nu 2m < a < 2m+1 , 2n < b < 2n+1 th

a b = 2 m 1 ( b 3 ) + a 2 m 1 ( 2 n 2) + 2 m + 1 = 2 n + m 1 + 1

v a b 2m1 2n+1 4 + 2m+1 1 = 2m+n 1. V vy
( a b) = m + n 1
Nu 2 p < c < 2 p+1 th


(a b) c = 2

m 1

( b 3) + a c = 2m + n 2 ( c 3) + 2m 1 ( b 3) + a





a ( b c) = a 2m1 (c 3) + b = 2m1 2n1 (c 3) + b 3 + a = (a b) c.

20

Nguyn Hu in, HKHTN H Ni

.4.20. Cho t gic li ABCD c mt ng trn ni tip. Nu


A=B=

, D = , BC = 1
3
2

Tm di AD
d =
Li gii: Goi I l tm ng trn ni tip . Do ABC l tam gic u, BIC
d = 150 , AID
[ = 750 , [
1050 , ICB
IDA = 450 nn
AD =

sin 150 sin 750


BI AD
=
= 2 sin 150 .
0
0
BC AI
sin 105 sin 45

.4.21. Gi A l tp con ca {0, 1, 2, ..., 1997} gm hn 1000 phn t. Chng minh


rng A ch gm nhng ly tha ca 2 hoc hai phn t phn bit c tng l ly
tha ca 2.

.c

Li gii: Gi s tp A khng tha mn bi ton. Khi A s bao gm hn


na s nguyn t 51 ti 1997 m chng c chia thnh tng cp c tng l

2048 (VD : 51 + 1997 = 2048...). Tng t nh vy, A bao gm nhiu nht


na s nguyn t 14 ti 50, gm nhiu nht na s nguyn t 3 ti 13, v c

th c s 0, do A c tng cng 937 + 18 + 5 + 1 = 997 s nguyn, tri vi


gi thit A gm hn 1000 s nguyn t tp {0, 1, 2, ..., 1997}.

.4.22. Xc nh s t nhin n tha mn nhng iu kin sau:


a, Khai trin thp phn ca n gm 1000 s
b, Tt c cc s trong khai trin l s l.
c, Hai phn t bt k lin nhau trong khai trin ca n hn km nhau 2 n v
Li gii: t an , bn , cn , dn , en l s trong khai trin ca n, l nhng s l v
hai s lin tip khc nhau 2 n v do tn cng theo th t l 1, 3, 5, 7, 9
do

0 1 0

1 0 1

0 1 0

0 0 1

0 0 0

a
a
n n +1

0 0 bn bn + 1

1 0 c n = c n +1

0 1 d n d n +1

e n +1
en
0 1
0 0

21

thi olympic Ireland

Gi A l ma trn vung trong biu thc . Ta tm gi tr ring ca ca A,


gi s Av = v vi v = (v1 , v2 , v3 , v4 , v5 ). Do
v2 = v1


v3 = v2 v1 = 2 1 v1

v4 = v3 v2 = 3 2 v1


v5 = v4 v3 = 4 32 + 1 v1

v v4 = v5 , do 5 33 + = 3 2. Gii pt ny ta c = 0, =

1, = 3 tng ng ta c cc vect ring x1 , x2 , x3 , x4 , x5 l





(1, 0, 1, 0, 1) , (1, 1, 0, 1, 1) , (1, 1, 0, 1, 1) , 1, 3, 2, 3, 1


v

2 3
1 2+ 3
x4 +
x5
(1, 1, 1, 1, 1) = x1
3
6
6

.c

V vy

(a1000 ,b1000 , c1000 , d1000 , e1000 ) =




 2 3 

999 2 +
3
1, 3, 2, 3, 1
1, 3, 2, 3, 1
=3 2
6
6


= 3499 , 2.3499 , 2.3499 , 2.3499 , 3499

V th kt qu ca bi ton l 8.3499 .

Chng 5
thi olympic Italy

.c

.5.23. Cho mt di giy hnh ch nht c chiu rng 3 cm, chiu di v tn. Gp
di giy li ch bng mt np gp.Hi phn di giy b ph bi vic gp c th c
din tch nh nht l bao nhiu ?

Li gii: Phn di giy b ph l mt tam gic.K hiu ba nh ca tam

gic l A, B, C trong AB l np gp v gc BAC nhn. H cc ng cao


[0 = BAC.
[0 = x. Ta xt hai
[ t BAB
AA0 , BB0 , CC0 ca ABC. Ch BAB

4,

th th C nm gia A v B, \
ABA0 = x v

trng hp :
Trng hp 1 : Gi s 0 < x
\0 = 2x. Ta c:
ACA

SABC = SABA0 SACA0


1
1
= .3.(3 cot x ) .3.(3 cot 2x )
2
2
9
= (cot x cot 2x )
2
9
=
csc 2x
2
Gi tr nh nht ca csc2x ch c th l 1 khi x =
nht l

4.

Do din tch nh

9
2

.5.24. Cho f l mt hm gi tr thc sao cho vi mi s thc x ta c


(a) f (10 + x ) = f (10 x )

23

thi olympic Italy

(b) f (20 + x ) = f (20 x )


Chng minh rng f l hm l ( f ( x ) = f ( x )) v tun hon (tc l tn ti T > 0
sao cho f ( x + T ) = f ( x )).

Li gii: Chn x = n 10, t (a) ta c f (n) = f (20 n)

Chn x = n t
(b) ta c f (20 n) = f (20 + n)
f (n) = f (n + 20)
T suy ra
. Do f (n + 40) = f (n)). V vy
f (n + 20) = f (n + 40)
f tun hon v c chu k l T = 40
Ta cng c f (n) = f (20 + n) = f (20 n) = f (n). Vy f l hm l.

.c

.5.25. Gc phn t th nht ca mt phng to c chia thnh cc hnh vung


dn v bi cc ng li.C th t mu cc hnh vung n v tho mn cc iu
kin sau?
(a)Vi mi hnh vung ln c mt nh dt ti gc, v cc cnh song song vi vi
cc trc to th cha nhiu hnh vung n v c t hn cc hnh vung dn
v khng c t.
(b)Mi ng song song vi ng phn gic ca gc phn t th nht ch i qua
cc hnh vung c t mu.

Li gii: C th thc hin nh sau: Trn mi ng y = x + D, ta t mu

| D | + 1 hnh vung gn trc nht.


Xt ng y = x + D vi D 0. Dc ng ny hnh vung u tin c
t nm trn ct th 1 v dng th D + 1. Hnh vung cui cng c t
mu nm trn ct th D + 1 v dng th 2D + 1. Do cc hnh vung nm
bn tay phi ca hnh vung ny ( pha trn ng y = x) l phn m
cc ng cho c cc hnh vung c t t hn, th th khng mt hnh
vung no trong s c t c. Nu ta k hiu (i, j) l hnh vung dng
th i v ct th j th hnh vung (i, j) c t khi v ch khi:
j i, i D + 1 i ( j i + 1) i
hoc j

i +1
2 .

j+1
2

Tng cc hnh vung c t trong n.n hnh vung l



n 
k+1
)1
Cn = 2(
2
k=1

24

Nguyn Hu in, HKHTN H Ni

Nu n chn, ta c Cn = 21 n2 + n 1

Nu n l , th Cn = 12 n2 + n 21
Do Cn 21 n2 vi mi n s tho mn iu kin bi.

.5.26. Cho t din ABCD.Gi a l di ca AB v S l din tch hnh chiu ca


t din ln mt phng vung gc vi AB.Hy xc nh th tch ca t din ABCD
theo S v a.
Li gii: Ta k hiu cc to A = (0, 0, 0), B = (0, 0, n), C = (0, b, c), D =

.c

(i, j, k). Khi mt phng z = 0 vung gc vi AB, v hnh chiu ca t


din ln mt phng ny l mt tam gic c nh A0 = B0 = (0, 0, 0), C0 =
(0, b, 0), D 0 = (i, j, 0). Tam gic ny c y l b v chiu cao tng ng l i.
Vy S = b 2i v a = AB = n.
tm th tch , ta xt t din nh l hnh chp c y l tam gic ABC.Khi
mt phng y c phng trnh x = 0 v chiu cao h t d c di l i.
a
Din tch ca tam gic ABC l b n2 . Vy th tch ca t din l : b in
6 = S3.

.5.27. Cho X l tp hp tt c cc ss t nhin m cc ch s ca n i mt khc


nhau. Vi mi n X, t An l tp hp tt c cc s m cc ch s ca n l mt
hon v ca cc ch s ca n.
V d n = 47 th An
= {47, 74} , n = 125 th An =
{125, 152, 251, 215, 521, 512}.
Gi dn l CLN ca tt c cc s trong An . Tm gi tr ln nht c th ca dn .

Li gii: Gi s n c nhiu hn hoc bng 3 ch s. Gi AB l hai ch s


cui th th s c hai ch s cui theo th t l BA cng thuc An . Vy hiu
ca hai s trn l : | BA AB| = |10B A 10A + b| = 9 | A B| 81.
Nu dn l c ca hai s trn th dn l c ca 81 vy dn 81

Nu n gm hai ch s , c hai ch s u khc 0 th lp lun nh trn.


Nu n gm hai ch s m mt trong hai ch s l 0 th An ch cha n.
( Vd n = 90 th An = {09, 90} = {90}) trong trng hp ny gi tr ln

nht ca dn = 90;nu n = 10 th An = {10}. Suy ra dn = 90 > 81


Vy gi tr ln nht c th ca dn l 90.

Chng 6
thi olympic Japan

.6.28. Chng minh rng bt k 9 im bt k nm trong mt ng trn ng


knh 5, tn ti hai im c khong cch nh hn 2.

.c

Li gii:

Chia hnh trn thnh 9 phn: mt phn l hnh trn bn knh 1 ng tm


vi ng trn cho v 8 hnh qut bng nhau l giao ca phn cn li
vi ng trn. Sau kim tra c rnng hai im trong mi phn c
khong cch nhiu nht l 2.

.6.29. Cho a, b, c l cc s dng. Chng minh bt ng thc sau:


( c + a b )2
( a + b c )2
3
( b + c a )2
+
+

5
( b + c )2 + a2 ( c + a )2 + b 2 ( a + b )2 + c 2

26

Nguyn Hu in, HKHTN H Ni

v xc nh khi no du bng xy ra.


Li gii: u tin, rt gn
a2

2ba + 2bc
2ca + 2cb
12
2ab + 2ac
+ 2
+ 2

2
2
2
2
2
2
5
+ b + c + 2bc a + b + c + 2ac a + b + c + 2ab

t s = a2 + b2 + c2 . Sau quy ng kh mu s ta c

5s2 (ab + bc + ca) + 10s(a2 bc + ab2 c + abc2 ) + 20(a3 b2 c + ab3 c2 + a2 bc3 )

6s3 + 6s2 (ab + bc + ca) + 12s(a2 bc + ab2 c + abc2 ) + 48a2 b2 c2

n gin hai v

6s3 + s2 (ab + bc + ca) + 2s(a2 bc + ab2 c + abc2 ) + 48a2 b2 c2

.c

10s(a2 bc + ab2 c + abc2 ) + 20(a3 b2 c + ab3 c2 + a2 bc3 ).

Thay s v khai trin biu thc ca s

(3a6 + 2a5 b 2a4 b2 + 3a4 bc + 2a3 b3 12a3 b2 c + 12a2 b2 c2 )+

+(3b6 + 2b5 c 2b4 c2 + 3b4 ca + 2b3 c3 12b3 c2 a + 12b2 c2 a2 )+

+(3c6 + 2c5 a 2c4 a2 + 3c4 ab + 2c3 a3 12c3 a2 b + 12b2 a2 b2 0. ()


p dng bt ng thc Schur, ta c:

(4a4 bc 8a3 b2 c + 4a2 b2 c2 ) + (4b4 ca 8b3 c2 a + 4b2 c2 a2 ) + (4c4 ab 8c3 a2 b + 4c2 a2 b2 ) 0,


chng minh bt ng thc (*), ta cn chng minh:

(3a6 + 2a5 b 2a4 b2 a4 bc + 2a3 b3 4a3 b2 c)+


+(3b6 + 2b5 c 2b4 c2 b4 ca + 2b3 c3 4b3 c2 a)+
+(3c6 + 2c5 a 2c4 a2 c4 ab + 2c3 a3 4c3 a2 b) 0.

27

thi olympic Japan

Ta c th chng minh bt ng thc trn bng bn biu thc khng m bi


bt ng thc AM-GM:
02

2a6 + b6
3 a4 b2
a,b,c

02

4a6 + b6 + c6
a4 bc

3
a,b,c

02

2a3 b3 + c3 b3
a3 b2 c
3
a,b,c

02

2a5 b + a5 c + ab5 + ac5


a3 b2 c.

6
a,b,c

Du bng xy ra khai v ch khi a = b = c.

.c

.6.30. Cho G l th 9 nh. Gi s rng vi bt k 5 nh ca G u tn ti t


nht hai cnh c im u v im cui thuc vo 5 im . Hi rng s cnh nh
nht c th c ca G l bao nhiu?

Li gii:

S cnh nh nht l 9, t c bi 3 chu trnh ri nhau.


Tht vy, gi an l s cnh nh nht ca th n nh tha mn iu kin

28

Nguyn Hu in, HKHTN H Ni

n+1
an . Vi mi th n nh gi li l
n1
s cnh ca th nhn c bng cch b i nh th i v tt c cc cnh

bi ra. Ta s chng minh rng an+1

gn vi nh th i. (Ta c li an , mt khc l1 + l2 + ... + ln+1 = (n 1)an+1 .


V mi cnh c m cho mi cnh khc tr hai im u cui ca n).
T , a5 = 2, ta nhn c a6 3, a7 5, a8 7, a9 9.

.6.31. Cho A, B, C, D l bn im khng ng phng. Gi s rng AX + BX +


CX + DX t gi tr nh nht ti X = X0 khc A, B, C, D. Chng minh rng
\
\
AX
0 B = CX0 D.
Li gii:

.c

X = X0

B
Gi s A, B, C, D v P c cc ta ( x1 , y1 , z1 ), ..., ( x4 , y4 , z4 ) v ( x, y, z). Ta
c hm s
q
f ( P) =

( x x i )2 + ( y y i )2 + ( z zi )2

. t gi tr nh nht, ba o hm ring ca n phi bng khng, nhng


PA
c ba hm ta ca ua + ub + uc + ud , y ua l vct n v
|| P A||
v tng t. Do tng ny bng khng, v ua .ub = uc .ud ti im P = X0 , t
ta c iu phi chng minh.

29

thi olympic Japan

.6.32. Cho n l mt s nguyn dng. CMR: c th gn cho mi nh ca mt a


gic 2n nh, mt trong cc ch ci A hoc B sao cho cc dy n ch ci nhn c
bng cch c bt u t mt nh no theo ngc chiu kim ng h, l lun
khc nhau.
Li gii: Xt mt th c hng nh sau: mi nh ca th l mt dy
c di n 1, hai nh l k nhau nu n 2 ch ci cui ca nh ny
trng vi n 2 ch ci u ca nh kia. (Ch l: y l mt th c hai

vng). Mi nh ca th c mt cnh i vo v mt cnh i ra, v th tn


ti mt ng i c hng i qua mi cnh ng mt ln. Ta c th c mt
chu trnh cn thit bng cch bt u t mt nh bt k, vit ra dy ch
tng ng vi n, sau vit thm vo ch cui cng ca mi dy, ta s

.c

gp ht cc dy trn ng i .

Chng 7
thi olympic Korean

.7.33. Chng minh rng vi bn im bt k trong mt ng trn n v th tn

ti hai im m khong cch gia chng khng vt qu 2.

.c

Li gii: - Trng hp 1:
Nu mt trong 4 im l tm ng trn th ta c ngay iu phi chng

minh.
- Trng hp 2:

Nu khng c im no trong bn im trng tm ng trn th ta k hiu


cc im ln lt l P1 , P2 , P3 , P4

Ta c t gic li Q1 Q2 Q3 Q4 , vi { Qi } l giao im ca {OPi } vi ng


trn.
Khi : P1 OP2 + P2 OP3 + P3 OP4 + P4 OP1 2
Ta c Pi OPi +1 2
on Pi Pi +1 nm trong tam gic OQi Qi +1 v vy ta c:

Pi Pi +1 max (OQi , Qi Qi +1 , Qi +1O) = max (1, 2 sin OQi Qi +1 )

.7.34. Cho hm s: f : N N tha mn hai iu kin:


a. n N, f(n + f(n)) = f(n)
b. n0 N, f(n0 ) = 1
Chng minh rng: f(n) = 1, n N

Li gii: Trc ht ch rng nu n N f(n) = 1, sau


f(n + 1) = f(n + f(n)) = f(n) = 1

31

thi olympic Korean


Cho f(n0 ) = 1,f(n) = 1, n n0
t S = {n N | f (n) 6= 1 }

Nu S 6= , gi N = maxS. Ta c f(N + f(N)) = f(N) 6= 1


N + f(N) S, N + f(N) > N

iu ny mu thun vi N = max S
Vy S = v f(n) = 1, n N
n j k
 
.7.35. Biu th tng
k theo cc s hng ca n v a =
n
k=1

Li gii: Ta s chng minh kt qu l:

a(a + 1)(2a + 1)
6

( n + 1)

Tht vy, ta quy c vic s dng du [ ] nh sau:

k=1 j =1

j =1 k=1

k=1

.c

Gi P l mt mnh , [P] c gi tr bng 1 nu P ng, ngc li P c gi tr


0 nu P sai.
Ch rng bkc l s nguyn dng v bnh phng ca n gn k nht.
j k
a 

k = j2 k
V vy:
j =1
n a 
a n 
n j k


k = j2 k = j2 k
Do :
k=1

By gi tng: j2 k m c

k=1

vi k {1, ..., n} m k j2 , j a, j2 n th s l n + 1 j2
a
n j k
a( a+1)(2a+1)
k = ( n + 1 j2 ) = ( n + 1)
V vy
6
j =1

.7.36. Cho C l mt ng trn tip xc vi cc cnh ca gc xOy v C1 cng k


mt ng trn tip xc vi cc cnh gc v i qua tm ca C. Gi A l giao
im th hai ca ng knh ca C1 qua tm ca C vi C1 v gi B l giao ca
ng knh vi C. Chng minh rng ng trn tm A i qua B tip xc vi cc
cnh ca gc xOy
Li gii: Gi T v T1 l tm ng trn C v C1 ; r, r1 ln lt l cc bn knh
ca hai ng trn . V ng vung gc TT , T1 T10 v AA ti Ox.

32

Nguyn Hu in, HKHTN H Ni

Ta c: TT = r, T1 T10 = r1 . V T1 l trung im ca AT nn
T1 T10 =

AA0 + TT 0
AA0 = 2T1 T10 TT 0 = 2r1 r
2

V vy AB = AT BT = 2r1 r. Suy ra pcm.

.7.37. Tm tt c cc s nguyn x, y, z tha mn iu kin x2 +y2 +z2 2xyz = 0


Li gii: Ta chng minh nghim duy nht l x = y = z = 0.
- Trc ht ta c x, y, z khng th l cc s l v khi tng x2 +y2 +z2 2xyz
.
l mt s l nn khc 0, do xyz ..2
Mt khc: x2 +y2 +z2 = 2xyz chia ht cho 4 khi tt c cc bnh phng chia
ht cho 4 hoc chia cho 4 d 1, x, y, z phi l cc s chn, do ta li vit

x = 2x1 , y = 2y1 , z = 2z1


Ta c: 4x12 + 4y21 + 4z21 = 16x1 y1 z1

.c

Hay x12 + y21 + z21 = 4x1 y1 z1


V v phi chia ht cho 4 nn nn ta li vit c

x1 = 2x2 , y1 = 2y2 , z1 = 2z2

Thay vo ta c: 4x22 + 4y22 + 4z22 = 32x2 y2 z2

Hay x22 + y22 + z22 = 8x2 y2 z2


Tip tc qu trnh ta c: n 1, x2n + y2n + z2n = 2n+1 xn yn zn

Tng t cc s xn ,yn ,zn l chn nn ta c th vit


xn = 2xn+1, yn = 2yn+1, zn = 2zn+1 tha mn: x2n+1 + y2n+1 + z2n+1 =
2n +2 x n +1 y n +1 z n +1
Lp li qu trnh ny ta c dy cc s nguyn ( x1 , x2 , ..., xn ) tha mn
xi = 2xi +1
.
Li c x = 2n xn x..2n , n 1 x = 0.
Chng minh tng t ta c y = z = 0.
Ghi ch: Nu th x = yz - w ta c bi ton USAMO 76/3

.7.38. Tm s nguyn k nh nht tn ti hai dy {ai }, {bi }, tha mn:




a.ai , bi 1, 1996, 19962, ... , i = 1...k
b.ai 6= bi , i = 1...k

33

thi olympic Korean


c.ai ai +1 , bi bi +1
k

i =1

i =1

d. ai = bi
Li gii: Ta chng t s k phi tm l s 1997.
Gi s c hai dy { ai }, {bi } tha mn cc iu kin ca bi ton, vi
k 1996
T iu kin b. cho ta a1 6= b1

V vy khng mt tnh tng qut, gii s a1 < b1


T iu kin a. ta c 0 m < n : a1 = 1996m , b1 = 1996n

T bi b1 i = 1, 2, ..., (K c.) v mi gi tr bi mt ly tha ca 1996, bi


i =1

chia ht cho 1996n

k
.
Do t iu kin d, ai ..1996n
i =1

Ta k hiu t l mt s jn m a j = 1996m .
n

Ta c t.1996m = ai = 0( mod1996m+1 )
i =1

i =1

i =1

i =1

.c

v th t 1996 v t k 1996
V vy ta c t = k = 1996.

Do 1996m+1 = ai = bi b1 = 1996.1996n = 1996n+1

iu ny mu thun vi m < n. V vy k 1997

Vi k = 1997 ta c v d:
a1 = a2 = ... = a1996 = 1, a1997 = 19962
b1 = b2 = ... = b1997 = 1996

.7.39. t An l tp tt c cc s thc c hnh thnh t tng:


1
1+
+ 2 2 + ... + n n
2
( 2)
( 2)
Vi j = {1, 1} , j
Tm s phn t ca An v tng ca tt c cc tch ca hai phn t phn bit ca An
Li gii:nTrc ht ta chng minh b :o

o
n

n 1, 21 + 42 + ... + 2nn | i {1, 1} = 2n , jl, | j| < 2n


Tht vy
Ta chng minh bng phng phpnquy np.
o
Khi n = 1 th c hai tp hp u l 21 , 21 , n 1, j {1, 1}

34

Nguyn Hu in, HKHTN H Ni

n
2
1
0
t j = 2n1. 1 + ... +
n , vi j l v 2 + 4 + ... + 2n = 2n
2 .



T 21 + 42 + ... + 2nn 21 + 42 + ... + 2nn = 21 + 14 + ... +

ta c | j| <
Khi tp hp v tri cha trong tp hp v phi.

1
2n

<1

2n

Xt vi j l v | j| < 2n . Khi hoc


nguyn lin tip.

j 1
2 ,

hoc

tj0 l mt trong hai s l , khi | j0 |


Do | j| 2n , v j0 l s l, | j0 | 2n1

j +1
2

l s l v chng l hai s

| j|+1
2

2n +1

V vy theo gi thit quy np tn ti 1 ,..., n1 sao cho


j0
2
n 1
1
2 + 4 + ... + 2n1 = 2n1
t n = j 2j0 {1, 1}

Khi 21 + 42 + ... + 2nn11 + 2nn =


Vy b c chng
n minh.

j0
2n 1

j2j0
2n

j
2n

o

k 2
bn/2c , |k| 2dn/2e
+
j,
k,
j

2
|
|

2[n/2]
2[n/2]
n
n
j v k l. V vy An cha phn t 2b 2 cd 2 e = 2n
tnh tng tt c
t khc nhau
cc phn
ca An ta s dng cng thc:
!2
ab = 1 a a2

a An

a An

a,b An ,a< b

.c

T b ta c: An = 1 +

j
k 2
+ 2k[n/22] v1 2[n/2
] 2[ n/2]
2[n/2]
V th trung bnh ca cc phn t ca An l 1; t a = | An | = 2n
j

By gi ta c th ghp i phn t 1 +

a An

By gi nu X, Y l hai tp cc s thc hu hn vi x = y = 0, ta c
2

( 1 + x + y ) = | X | |Y | + |Y |

x X y Y

x2

xX

+ |X|

xX
y2

y Y

y Y

T ba s hng khc khng theo gi thit ta c tng:


j2 = 13 2m((2m)2 1)

jl?,| j|<2m

d dng chng minh c bng php quy np theo m. V th

a2 =
a An

a An

a2

kle
jle
dn/2e
| j|2bn/2c | k|2

kle
jle
dn/2e
| j|2bn/2c | k|2

1+
1+

j
2[n/2]
j
2[n/2]

+ 2k[n/22]
+

2


k 2
2[n/2]

thi olympic Korean




2bn/2c (22dn/2e 1)
2dn/2e (22bn/2c 1)
1
2
n
n/2
n/2
d
e
b
c
+2
a =2 +3 2
2b n/2c
d n/2e1
2

a An

35

a An

a2 = 2 n + 1 1

Ch n chn v
n l khc nhau bccui cng, do :
!2

a,b An ,a< b

ab =

1
2

a2 = 21 (22n 2n+1 + 1)

a An

a An

.7.40. Cho tam gic nhn ABC vi AB khc AC. Gi V l giao im ca ng phn
gic gc A vi BC v gi d l chn ng cao h t A ti BC. Gi E v F l cc
giao im phn bit ca ng trn ngoi tip tam gic AVD vi CA v AB,chng
minh rng 3 ng thng AD, BE, CF ng quy.

v A, D,V, E, F nm trn ng trn, BFV =

Li gii: T ADV =

.c

CEV = 2 ta c tam gic BFV v tam gic BDA ng dng; tam gic CEV
v CDA cng ng dng.
AB CD
AC
V vy BD
BF = VB ; CE = VC
AB
AC
Nhng VB
= VC
( theo nh l ng phn gic)
CD
V th BD
BF = CE

Li c FAV = VAE, AE = AF, t

BD
BF
CD
CE

= 1 v AD,BE,CF

ng quy theo nh l Ceva.

BD CE AF
DC . EA . FB

.7.41. Mt t l mt dy gm ch s 0 hoc 1. t x v y l 2 t khc nhau ng


3 v tr. chng minh rng s t khc nhau gia x v y khng qu 5 v tr l 38.
Li gii: Khng mt tnh tng qut gi s x=00000000, y=00000111.
Gi z l mt t khc gia x v y m c t nht 5 v tr khc nhau nu v ch
nu a + b 5 vi a l ch s 1 u trn trong 5 ch s khc nhau ca z v b
l ch s 1 cui cng trong 3 ch s ca z. Hn na ta c bt ng thc 2a

7.
iu phi chng minh l (4,1),(4,2 ) v (5,b) vi {0, 1, 2, 3}
Hai iu u tin cho

5
3
3
+ = 30
4
1
2

36

Nguyn Hu in, HKHTN H Ni

t cho z v cc trng hp khc cho 23 = 8.


V th c 38 t khc nhau gia x v y m c 5 v tr khc nhau.

.7.42. Tm tt c cc cp hm s f,g:R R tha mn


a. Nu x<y th f(x) < f(y)
b. x, y R, f ( xy) = g(y). f ( x ) + f (y)

xm , x 0
Li gii: Xt cp (f,g ) cho bi: f ( x = t(1 g(t))) g( x ) =
| x|m , x < 0

vi t < 0, m > 0 l nghim duy nht.


t x = 0, t b ta c: f (0) = f (0) g(y) + f (y), suyra f (y) = f (0)(1 g(y)).

t t = f (0), ta c f (y) f (y) = t(1 g(y)). Do f l hm ng bin nn ta


khng c t = 0.

Thay cng thc ny cho f b cho ta t(1 g( xy)) = g(y)t(1 g( x )) + t(1


g(y)). T ta c: 11 g( xy) = g(y)(1 g( x )) + 1 g(y) = 1 g( x ) g(y)

.c

hoc g(xy) = g(x)g(y) vi mi x, y thuc R.


f
T g = 1 t l hm n iu ngt, v vy g(1) 6= 0
Nhng li c g(1) = g2 (1), v th g(1) = 1.

Ta c g l hm tng nn f cng tng, ta c t < 0..V g( x ) > 0, x > 0. Gi h:


R R vi h = log.g.exp.

Ta ch( x + y) = log( g(e x+y )) = log( g(e x ) g(ey )) = log( g(e x )). log( g(ey ))
= h ( x ) + h (y )

h(0) = log( g(e0 )) = 0 v h n iu ngt.


Ta cng c h(x+y) = h(x) + h(y) suy ra h(nx) = n.h(x) vi n N v h(-x) =

-h(x), do h(al phax) = .h(x) vi Q.


Xt dy hu t {yi } , {yi }
xi < x

yi > x
v s dng tnh n iu chng t rng h(x) = x.h(1), x R

t m = h(1), t ta c m > 0 v h l hm tng v g( x ) = x m , x > 0


2
Ta c g(1) < 0, m ( g(1))
= g(1) = 1 nn g(-1) = 1.
xm , x 0
T g(-x) = -g(x) nn g( x ) =
| x |m , x < 0
Ta cng c f(x) = t(1-g(x)). D dng kim tra c rng cp (f,g) l nghim

37

thi olympic Korean

m > 0 > t.

.7.43. Cho a1 , a2 , ...,an l cc s dng v k hiu: A =


1
G = (a1 ...an ) n
H = 1 n 1

a1 +...+ an
n

a1 +...+ an

A
A n
a. Nu n chn, chng minh rng H
1 + 2( G
)
2
(
n

1) A n
A
2
b. Nu n l, chng minh rng: H
n
(G)
n +
n

Li gii: Ch rng theo bt ng thc Mclranh ta c:


! n 1
G
H

1
a1 ...an ( a11 +...+ a
n )
n

V th

1
n

j =1

a1 ...an
aj

1
n

= A n 1

aj

j =1

A
H

.c

A n
(G
)
A n
A
A n
A n
T A G, ( G
) 1 nn H
(G
) 1 + 2( G
)
a.
A n
A n
2 A n
A
(G
) (G
) + n
Vi b, H
n (( G ) 1)
2( n 1) A n
A
2
H
n
(G)
n +
n
Ta c pcm.

Chng 8

thi olympic Poland

.c

.8.44. Cho cc s nguyn x1 , x2 , ..., x7 tha mn iu kin x6 = 144, xn+3 =


xn+2 ( xn+1 + xn ); n = 1, 2, 3, 4.
Tnh x7 .

Li gii: Nhn phng trnh cho vi n = 1, 2, 3 v kh nhn t chung ta


c

x4 = x3 ( x2 + x1 ) 2x3
x5 = x4 ( x3 + x2 ) 2x23

144 = x3 ( x1 + x2 )( x2 + x3 )( x3 + x4 ).[1]
Mt khc t phng trnh cho ta c bt ng thc:

144 = x6 x5 ( x4 + x3 ) 2x23 (3x3 ) 144 6x33 x3 = 1, 2.


+ Xt trng hp 1: x3 = 1
T [1], 144 = ( x1 + x2 )( x2 + 1)( x1 + x2 + 1). Nhn c 2 v vi 144 lin tip
cc cp s nguyn (1, 2), (2, 3), (3, 4), (8, 9). T x1 + x2 v x1 + x2 + 1 vi h
s 144 cc s nguyn lin tip v khi x1 + x2 2. Ta c 3 trng hp:

1a. x1 + x2 = 2 6( x2 + 1) = 144 t suy ra x2 = 23, x1 = 21. Tuy


nhin n khng tha mn, vy xi l s nguyn xc nh.

1b. x1 + x2 = 3 12( x2 + 1) = 144 t suy ra x2 = 11, x1 = 8. Khng


tha mn.
1c. x1 + x2 = 8 72( x2 + 1) = 144 t suy ra x2 = 1, x1 = 7. Th nghim

c th xy ra: x4 = 8, x5 = 16, x6 = 144. Nh vy th ( x1 , x2 , x3 ) = (7, 1, 1)

39

thi olympic Poland


ta c x7 = 3456
+ Xt trng hp 2: x3 = 2

144 = 2( x1 + x2 )( x2 + 2)(2x1 + 2x2 + 2) 36 = ( x1 + x2 )( x2 + 2)(x1 + x2 +


1). Nhn c 2 v vi h s lin tip cc cp s nguyn (1, 2), (2, 3), (3, 4) ta c
cc trng hp sau:
2a. x1 + x2 = 2 x2 = 4, x1 = 2. (Khng tha mn)

2b. x1 + x2 = 3 x2 = 1, x1 = 2. Ta th cc nghim sau: x4 = 6, x5 =


18, x6 = 144. Nh vy vi ( x1 , x2 , x3 ) = (2, 1, 2) th x7 = 3456.
Vy gi tr x7 cn tm l 3456.

.
8.45. Gii h phng trnh sau vi x, y, z l cc s thc:
3 ( x 2 + y 2 + z2 ) = 1
x2 y2 + y2 z2 + z2 x2 = xyz( x + y + z)3

Li gii: Ta c x, y, z hoc (x+y+z) khng th bng 0 v xyz( x + y + z) =


x 2 y2 + y2 z2 + z2 x 2
0
( x + y + z )2

.c

Vi 3 s thc a, b, c ta c: (a b)2 + (a c)2 + (b c)2 0 hay a2 + b2 + c2


ab + ac + bc, du "=" xy ra nu v ch nu a=b=c.
x 2 y2 + y2 z2 + z2 x 2

xy2 z+ x2 yz+ xyz2

Vy 1 = 3( x2 + y2 + z2 ) ( x + y + z)2 = xyz(x+y+z) xyz(x+y+z) = 1


Du
"="
 xy
 ra khi v ch khi x = y = z, t suy ra (x,y,z) l

1 1 1
1 1 1
3; 3; 3 ;
3 ; 3 ; 3 .

chnh l nghim ca h phng trnh.

.8.46. Bi 3: Trong t din ABCD, gia cc mt ABD, ACD, BCD, t nh D


nhn cc cnh AB, AC, BC to thnh cc gc tng ng bng nhau. Chng minh
rng mi mt tam gic ny c din tch nh hn hoc bng tng ca din tch ca 2
mt khc.
Li gii: Cho 900 l 1 gc, u tin ta chng minh 6= 900 . Nu a,
b, c tng ng l di 3 cnh BC, CA, AB v m a , mb , mc tng ng l
3 ng trung tuyn ng vi 3 cnh BC, CA, AB th din tch ca tam
gic DABl 12 mc c sin
tch gia
 . Ch rng gi tr tuyt i ca nhng


A+ B
2
v A-B l mc ccos= 2cot [ DAB] = DA DB2 .
vect l D
2

Chng ta c 3 din tch [ ABD] , [ ACD] , [ BCD] tng ng t l l






DA2 DB2 , DA2 DC2 , DB2 DC2
Ta

chng

minh

900 .

40

Nguyn Hu in, HKHTN H Ni

[ y = BDC,
[ z = ADC
[ (0 < x, y, z < 180). Ch rng 3 gc x, y, z l 3 gc
ADB,
tam din, chng ta c: x + y > z, x + z > y, y + z > x v x + y + z 360.
( tng t i vi din tch
Khi din tch ca tam gic ADB l AD.BD.sinx
2
cc tam gic BDC v ADC) v AD=BD=CD( khi = 900 ), chng ta cn
chng minh rng: sin x + sin y > sin z
xy
x+y
Ta c sin x + sin y = 2 sin 2 cos 2 , sinz = 2sin 2z cos 2z
xy

x+y

Ta cn chng minh: sin 2 cos 2 > sin 2z cos 2z


x+y
T x + y + z 360 2 180 2z . rng 0 <
xy

x+y

xy

.8.47. Cho dy a1 , a2 , ..., xc nh bi :


a1 = 0, an = a[ n/2] + (1)n(n+1)/2 , n > 1
Vi mi s nguyn k 0. Tm n tha mn:

sin 2 > sin 2z . Nh vy t 2 < 2z cos 2


gim trong [0, 180]
x+y
xy
T suy ra sin 2 cos 2 > sin 2z cos 2z .(pcm)

x+2 y 180 2z
> cos 2z . Khi hm cosin
z
2

.c

2k n 2k+1 van = 0

Li gii: t Bn c ngha l c s 2 i din ca n s. Ta chng minh bng

quy np. Gi s an l s hng ca 00 hay 11 on trong Bn tr i s ca


01 hay 11 trong Bn . Vi k=1,2,3,...,n-1, ak l s hng ca 00 hay 11 trong Bk

tr i s 01 hay 10 trong Bk . u tin xt trng hp khi n 0, 3(mod4)


th Bn gii hn trong 00 hay 11. Nh vy an bng mt s dng ca s 00
hay 11 trong tt c cc on nhng m ch s ca Bn tr i cc s 01 hay
10 trong tt c cc on nhng m s cui cng ca Bn cho bi a[ n ] . Nh vy:
2

an = a[ n ] + 1 = a[ n ] + (1)n(n+1)/2
2
2
Tng t vi n = 1, 2(mod4) ta c:
a[ n ] 1 = a[ n ] + (1)n(n+1)/2
2
2
Vy ta chng minh xong.
Nh vy vi k s nguyn cho chng ta cn tm s ca n s nguyn tha
mn 2k n 2k+1 v s ca 00 v 11 on bng s ca 01 v 10 on.

41

thi olympic Poland

Ch rng Bn c k+1 s. Vi mi Bn chng ta xy dng dy mi Cn ca 00 s


v 10 s ging nh nhau.
Bt u t cc s ca Bn v ch s gn cui ca Bn , ta cng thm vo dy
s Cn gi tr tuyt i ca s khc gia v ch s bn tri. V d vi
B1 1 = 1011 v C1 1 = 110. Khi 00 hay 11 l on trong Bn sinh ra 0 trong
dy Cn v 01 hay 10 trong Bn sinh ra 1 trong Cn . Chng ta cn tm thy s
ca n s nguyn.
Nh vy c iu phi chng minh.

[ = DEA
[ =
.8.48. Cho ng gic li ABCDE vi DC=DE v BCD
AE
trung im on AB, khi AF
BF = BC . Chng minh rng:
d = FDE,
d = BDC.
[
[ FEC
FCE

2.

Cho F l

Li gii: Gi P = EA BC v xt im C, D, E, P. Gi Q, R tng ng l

im thuc DA, DB vi chu vi ca t gic CDEF.


Gi G = QC RF ta c:

.c

[ QC sin RBG
[
[ sin DBA
[
AG
sin DCQ
CD sin QRG
=
=
GB
DE sin GQR
[ RG sin GAQ
[
[ sin BAD
[
sin ERD

Suy ra ADF = BDC.

[ AD
AE
sin ADE
=
BC
[ BD
sin CDB

.8.49. Xt n im(n 2) trn ng trn. Chng minh rng s ln nht n3 ca

cung trn vi im cui trong s n im c chiu di ln hn hoc bng 2.


Li gii: Xy dng th cho bi cc nh bi mi cp im c khong cch

ln hn hoc bng 2
Chng ta s chng minh rng khng tn ti K4 .
Gi s tn ti K4 , gi cc nh theo th t l ABCD.

Cnh c chiu di ln hn 2 i din cung c di ln hn

2.

Nh vy

mi cung AB, BC, CD, DA nh hn 2pi v chng cng nh hn 2pi, suy ra


mu thun. (pcm)

Nguyn Hu in

OLYMPIC TON NM 1997-1998

.c

48 THI V LI GII
(Tp 6)

NH XUT BN GIO DC

.c

Li ni u
th gi lnh lamdethi.sty ti bin son mt s ton thi
Olympic, m cc hc tr ca ti lm bi tp khi hc tp LATEX.

ph v cc bn ham hc ton ti thu thp v gom li thnh cc


sch in t, cc bn c th tham kho. Mi tp ti s gom khong
51 bi vi li gii.

.c

Rt nhiu bi ton dch khng c chun, nhiu im khng


hon ton chnh xc vy mong bn c t ngm ngh v tm hiu

ly. Nhng y l ngun ti liu ting Vit v ch ny, ti c


xem qua v ngi dch l chuyn v ngnh Ton ph thng. Bn c

th tham kho li trong [1].

H Ni, ngy 2 thng 1 nm 2010


Nguyn Hu in

Rt nhiu on v mi hc TeX nn cu trc v b tr cn xu, ti


khng c thi gian sa li, mong cc bn thng cm.

51
89/176-05
GD-05

M s: 8I092M5

Mc lc
3

Mc lc .. . . . .. . . . . .. . . . . . .. . . . . .. . . . . .. . . . . .. . . . . .. . . . . .. . . .

Chng 1. thi olympic Russian.. .. . . . . .. . . . . .. . . . . .. . . . . .

Chng 2. thi olympic Nam Phi .. .. . . . . .. . . . . .. . . . . . .. . . .

Li ni u ... . . . . .. . . . . .. . . . . .. . . . . .. . . . . .. . . . . .. . . . . .. . . . .

12

Chng 4. thi olympic i Loan .. . . . .. . . . . . .. . . . . .. . . . .

16

.c

Chng 3. thi olympic Ty Ban Nha.. .. . . . . .. . . . . .. . . . .

23

Chng 6. thi olympic Ukraina .. . . . . . .. . . . . .. . . . . .. . . . .

27

Chng 7. thi olympic Anh.. . . . .. . . . . .. . . . . .. . . . . .. . . . . .

34

Chng 5. thi olympic Th Nh K .. . . . . .. . . . . .. . . . . .. .

Chng 1
thi olympic Russian
m

.1.1. Chng minh rng cc s t 1 n 16 c th vit c trn cng 1

.c

dng nhng khng vit c trn 1 ng trn, sao cho tng ca 2 s


bt k ng lin nhau l 1 s chnh phng.

Li gii: Nu cc s vit trn 1 ng trn th ng cnh s 16 l

s x, y khi 16 + 1 16 + x, 16 + y 16 + 15, suy ra: 16 + x = 16 + y = 25


mu thun. Cc s c th c sp xp trn 1 dng nh sau:

16, 9, 7, 2, 14, 11, 5, 4, 12, 13, 3, 6, 10, 15, 1, 8.

.1.2. Trn cnh AB v BC ca tam gic u ABC ly im D v K


trn cnh AC v ly im E v M sao cho DA + AE = KC + CM = AB.

Chng minh rng gc gia DM v KE bng .


3
Li gii: Ta c: CE = AC AE = AD.
V tng t: CK = AM.

2
bin K
Xt php quay tm l tm ca tam gic ABC, gc quay
3
thnh M, bin E thnh D, t suy ra iu phi chng minh.
.1.3. Mt cng ty c 50.000 cng nhn, vi mi cng nhn tng s
ngi cp trn trc tip v cp di trc tip ca anh ta l 7. Vo th
2 mi cng nhn a ra mt s ch dn v gi bn photo ca n cho
mi cp di trc tip ca anh ta (nu anh ta c). Mi ngy sau mi

Nguyn Hu in, HKHTN H Ni

cng nhn gi tt c cc ch dn m anh ta nhn c vo ngy hm


trc v gi bn photo ca chng cho tt c cp di trc tip ca anh
ta nu anh ta c hoc anh ta phi t thc hin nu khng c cp di
trc tip. C nh th cho n th 6 khng cn ch dn no a ra.
Hay ch ra rng c t nht 97 cng nhn ko c cp trn trc tip.
Li gii: Gi s k l s cng nhn ko c cp trn trc tip, vo ngy
th 2 s ch dn c a ra nhiu nht l 7k, vo ngy th 3 nhiu
nht l 6.7k vo ngy th 4 nhiu nht l 36.7k vo ngy th 5 mi
cng nhn nhn c 1 ch dn ko c cp di trc tip, v vy mi
cng nhn c 7 cp trn trc tip, mi ngi a ra nhiu nht l 6
ch dn v c nhiu nht l 216.7k/7 cng nhn nhn c ch dn.
Chng ta c:

50.000 k + 7k + 42k + 252k + 216k = 518k v k 97.

.c

.1.4. Cc cnh ca tam gic nhn ABC l cc ng cho ca hnh

vung K1, K2, K3. Chng minh rng mim trong ca tam gic ABC c
th c ph bi 3 hnh vung.

Li gii: Gi I l giao im ca 3 ng phn gic ca tam gic


d < 45 v vy tam gic IAB c
ABC, v cc gc l nhn nn IAB. IBA

th c ph bi hnh vung m ng cho ca n l AB v tng


t i vi tam gic IBC v tam gic ICA.

.1.5. Cc s t 1 ti 37 c th c vit trn 1 dng sao cho mi s l


c ca tng tt c cc s ng trc n. Nu s u tin l 37 v s
th 2 l 1 th s th 3 l bao nhiu.
Li gii: Gi s cui cng l x, x phi l c ca tng tt c cc s
l 37x19, v x = 19 v s th 3 phi l c ca 38 khc 1 hoc 19, vy
s th 3 l 2.
.1.6. Tm cc cp s nguyn t p, q sao cho p3 q 5 = (p + q)2 .

Li gii: Ch c nghim duy nht l (7, 3), u tin gi s c p v q


u khng bng 3. Nu chng ng d vi Module 3, v tri th chia
ht cho 3 nhng v phi th khng, nu chng khng cng ng d

thi olympic Russian

module 3 th v phi chia ht cho 3 nhng v tri th khng v th


khng xy ra kh nng ny. Nu p = 3 ta c q 5 < 27, khng c s
nguyn t no tha mn. V vy q = 3 v p3 243 = (p + 3)2 ch c
nghim duy nht l p = 7.
.1.7. (a) a. thnh ph Mehico hn ch giao thng mi xe oto
ring u phi ng k 2 ngy trong 1 tun vo 2 ngy oto
khng c lu thng trong thnh ph. Mt gia nh cn s dng
t nht 10 chic oto mi ngy. Hi h phi c t nht bao nhiu
chic oto nu h c th chn ngy hn ch cho mi chic oto.
(b) b. Lut c thay i cm mi oto ch 1 ngy trong 1 tun
nhng cnh st c quyn chn ngy cm . Mt gia nh hi
l cnh st gia nh c quyn chn 2 ngy lien tip khng

b cm cho mi xe v ngay lp tc cnh st cm xe oto vo 1 trong


nhng ngy khc. Hi gia nh cn t nht bao nhiu xe oto
nu h s dng 10 chic mi ngy.

.c

Li gii:

(a) Nu n xe oto c s dng, s ngy c s dng l 5n. M mi


gia nh s dng t nht l 10 xe nn 7x10 5n v th n 14.

Trong thc t 14 xe tha mn yu cu ca u bi ton: 4 xe b


cm vo ngy th 2 v th 3, 4 xe b cm vo ngy th 4 v th
5, 2 xe b cm vo ngy th 6 v th 7, 2 xe b cm vo ngy

th 7 v ch nht, 2 xe b cm vo ngy ch nht v th 6.


(b) 12 xe oto l s xe h cn, u tin chng ta ch ra rng n 11

xe khng tha mn. Khi c n ngy xe b cm, mi ngy nhiu


bn
bn
xe c lu thng nhng n 11,
< 10. i vi
nht l
7
7
n = 12, gia nh cn a ra 2 ngy lin tip cho mi xe trong
nhng ngy xe ko b cm lu thng.

.1.8. Mt a gic u 1997 nh c chia bi cc ng cho ko ct


nhau to thnh cc tam gic. Hy ch ra rng c t nht mt tam gic
nhn.
Li gii: ng trn ngoi tip a gic u 1997 nh cng l ng
trn ngoi tip mi tam gic. V tm ca cc ng trn khng nm

Nguyn Hu in, HKHTN H Ni

trn bt k ng cho no nn n phi nm trong mt tam gic, v


th tam gic phi l tam gic nhn.
.1.9. Vit cc s t 1 n 1000 trn bng, Hai ngi chi ln lt xa
i 1 s trong cc s , cuc chi kt thc khi cn li 2 s:
Ngi chi th 1 thng nu tng cc s cn li chia ht cho 3, cc
trng hp cn li ngi chi th 2 thng. Ngi chi no c chin
thut chin thng.
Li gii: Ngi chi th 2 c chin thut chin thng, nu ngi
chi th 1 xa i s x, ngi chi th 2 xa i s 1001 x v th tng

ca 2 s cui cng l 1001.

.1.10.C 300 qu to, khng c qu no nng hn 3 ln qu khc. Hy


ch ra rng c th chia cc qu to ny thnh 4 nhm m khng c
11
ln nhm khc.
nhm no c cn nng hn
2
Li gii: Sp xp cc qu to tng dn theo trng lng, v ghp

.c

tng i mt qu nh nht vi qu nng nht, sau li em 1 qu


nh nht tip theo vi 1 qu nng nht tip theo tip tc cho n

ht. Ch rng khng cp no nng hn 2 ln cp khc. Nu a, d v


b, c l 2 nhm vi

a b c d th a + d 4a 2b + 2c

b + c 3a + d 2a + 2d.

By gi cc cp nng nht v nh nht to thnh 4 nhm, khng


cn nng ca nhm no gp 2/3 ln nhm khc v
3
e f g h l cc cp th e + h 3e (f + g)
2
3
f + g 2e + h (e + h).
2

Chng 2

thi olympic Nam Phi


.2.11.Cho A0 B0 C0 v mt dy A1 B1 C1 ,A2 B2 C2 ... c xy dng

.c

nh sau:
Ak+1 , Bk+1 , Ck+1 l im tip xc ca ng trn ngoi tip Ak Bk Ck vi

cc cnh Bk Ck ,Ck Ak ,Ak Bk theo th t.


(a) hy xc nh Ak+1 Bk+1 Ck+1 t Ak Bk Ck
(b) chng minh: lim Ak Bk Ck = 600
k

Li gii:

(a) Ta c Ak Bk+1 = Ak Ck+1 (v y l 2 tip tuyn xut pht t


mt im).V vy Ak+1 Bk+1 Ck+1 l tam gic cn vi Ak Bk+1 Ck+1 =

Ak
.Tng t ta c: Ck Bk+1 Ck+1 = 900 C2 k .
2
(Ak +Ck )
= 900 B2 k
2
600
(b) Ta c Bk+1 600 = 900 B2 k 600 = Bk2
0
0 60
V Bk 600 = B(2)
Hin nhin lim Bk = 600
k
k

900

Hn na Bk+1 =

.2.12.Tm tt c cc s t nhin tho mn:khi chuyn ch s u tin


xung cui,s mi bng 3,5 ln s ban u.
Li gii:
Cc ch s nh th c dng sau: 153846153846153846. . . 153846
Hin nhin nhng s tho mn gi thit phi bt u bi 1 hoc 2.
Trng hp 1: S c dng: 10N + A viA < 10N .V 27 (10N + A) =

Nguyn Hu in, HKHTN H Ni

10
10A + 1 A =

(7.10N 2)
13

Ta c 10N 1, 3, 4, 9, 10, 12(mod13).V th A s l mt s nguyn t nu:


N 5(mod6).T ta c kt qu nh trn.

Trng hp 2: S c dng 2.10N + A, A < 10N .Theo chng minh


N
trn A = (14.1013 4) .Nhng v A < 10N ,tc 10N < 4.V l.
.2.13.Tm tt c cc hm: f : Z Z sao cho:
f (m + f (n)) = f (m) + n, m, n Z
Li gii:

R rng: f (n) = kn vi k = 1, k = 1.Ta s chng minh y l kt qu


duy nht.Cho n = 0 ta c f (m + f (0)) = f (m).Xt 2 trng hp:
Trng hp 1: f (0) = 0.cho m = 0 c f (f (n)) = n.Gn f (n) bi n

ta c: f (m + f (f (n))) = f (m + n) = f (m) + f (n) f (n) = nf (1) v


n = f (f (n)) = n(f (1))2 f (1) = 1.y l kt qu trn.

.c

Trng hp 2: f (0) 6= 0. f (n) l hm tun hon v b chn .t f (M)


f (n), n.nhng f (M + f (1)) = f (M) + 1.Mu thun gii thit.

.2.14.Cho mt ng trn v mt im P pha trn ng trn trong


mt phng to .Mt ht nh di chuyn dc theo mt ng thng t

P n im Q trn ng trn di nh hng ca trng lc.Khong


cch i t P trong thi gian t l: 12 gt2 sin vi g khng i v l gc

gia PQ vi mt phng nm ngang.Hy ch ra v tr im Q sao cho


thi gian di chuyn t P n Q l t nht.
Li gii:
Cu hi l tm gi tr nh nht ca

PQ
sin

hoc gi tr ln nht ca

sin
.
PQ

Biu din mt php nghch o im P vi nh l qu o ca chnh


0
n.im cc i trn s c(gi l Q) s vch ra mt im Q vi gi
tr ln nht P Q sin , vi cao khc nhau gia P v Q .Nh vy P l
0
im pha trn ng trnQ l im pha y ng trn. tm Q
0

hy ch rng P,Q,Q cng thuc mt ng thng.Do cch tm


nh sau:
0
(a): Tm im pha y ng trn gi l Q
0

(b): Tm giao ca P Q vi ng trn , l im cn tm.


0

thi olympic Nam Phi

11

.2.15.C 6 im c ni vi nhau tng i mt bi nhng on mu


hoc mu xanh..chng minh rng:C mt chu trnh 4 cnh cng
mu.
Li gii:
Gi cc im l A,B,C,D,E,F.D thy lun c mt chu trnh tam gic
cng mu(Tht vy:Xt mt s nh no ,s c 5 cnh t cc nh
y m t nht 3 trong s chng cng mu.gi s l mu v cc
cnh ny i n A,B,C.Nu mt s cnh gia A,B,C l mu ta c
iu phi chng minh,nu khng ta cng c iu phi chng minh)
Khng mt tnh tng qut,gi cc cnh AB,BC,CA l mu .Nu
mt trong cc nh ca cnh mu khc chy n A,B,C,ta c iu

cn tm.Nu 2 trong 3 im D,E,F ca cnh mu xanh chy n 2


trong s cc nh cng mu A,B,C,ta cng c iu cn tm.Trng
hp duy nht khng xy ra l nu mt trong cc im D,E,F ca

.c

cnh mu to bi nhng im khc A,B,C;khng mt tnh tng


qut gi s AD,BE,CF l mu .Cc cnh khng theo l thuyt l

DE,EF,FD.Nu mt cnh trong s chng mu ta c dng hnh


trn(v d nu DE mu th DABE l )Nu mt trong s chng

mu xanh th DCEF mu xanh.Ta chng minh c bi ton.

Chng 3
thi olympic Ty Ban Nha
m

.3.16.Tnh tng bnh phng ca 100 s hng u tin ca mt cp s

.c

cng, vi gi thit tng 100 s hng bng 1 v tng cc s hng th


hai, th t, ..., th mt trm bng 1
Li gii:

Gi 100 s hng u tin ca cp s cng l x1 , x2 , x3 , , x100 v d l


cng sai ca cp s cng theo gi thit th nht ta c:
1
1
(x1 + x100 ) .100 = 1 x1 + x100 = .
2
50

x1 + x2 + x3 + + x100 =

Theo gi thit th 2 ta c

Suy ra d =

3
50

ta tm c:

Vy

x2 + x4 + + x100 =

1
1
((x1 + d) + x100 ) .50 = 1 x1 + x100 + d = .
2
25

1
v x1 + x100 = x1 + (x1 + 99d) = 50
hay x1 = 149
. T
50


x21 + x22 + + x2100 = 100x21 + 2dx1 (1 + + 99) + 12 + + 992 .
x21 + x22 + + x2100 =

14999
50

.3.17.A l mt tp gm 16 im to thnh mt hnh vung trn mi cnh


c 4 im. Tm s im ln nht ca tp A m khng c 3 im no
trong s cc im to thnh mt tam gic cn

thi olympic Ty Ban Nha

13

Li gii: S im ln nht cn tm l 6 c c bng cch ly cc


im hai cnh k nhau nhng b i im chung ca hai cnh .
u tin gi 4 im bn trong khng c chn, nhng im cn li
to thnh 3 hnh vung, nn nhiu nht 2 ng thng ng t mi
hnh vung c chn. Nh vy chng ta c th cho rng mt trong
s cc im trong c chn trong s sau l im O
D A1 A2 A3
C Z1

Z2

E B1 B2 B2
C

Khng c im no cng tn gi A, B, C, D, E c chn, vy nu ta


khng chn Z1 , Z2 mt ln na nhiu nht 6 im c th c chn.

.c

Nu chn Z1 nhng khng chn Z2 th A1 , A2 , B1 , B2 cng khng c


chn, v c A3 v B3 cng khng c chn, v vy mt trong hai
im A v B phi b i, mt ln na s im ln nht l 6. Trng

hp chn Z2 nhng khng chn Z1 tng t. Cui cng nu Z1 v Z2


c chn th c Ai v Bi u khng c chn, v vy s im ln

nht l 6.

.3.18.Vi mi Parabol y = x2 + px + q ct hai trc ta tai 3 im phn


bit, v mt ng trn i qua 3 im . Chng minh rng tt c cc

ng trn u i qua mt im c nh

Li gii: Tt c cc ng trn u i qua im (0, 1). Gi s


(0, q) , (r1 , 0) , (r2 , 0) l 3 im m Parabol qua, do r1 + r2 = p.

Gi s (x a)2 + (y b)2 = r 2 l ng trn lun i qua 3 im trn


do a = 2p v

p 2
1
1 2
2
p + (q b) = r
+ b2 = (r1 r2 )2 + b2
4
2
4

, khi im i xng vi im (0, q)


hay q 2 2.qb = q do b = q+1
2
qua ng knh nm ngang l im (0, 1)
p
.3.19.Cho p l s nguyn t. Tm tt c k Z sao cho k 2 pk l s
nguyn dng.

14

Nguyn Hu in, HKHTN H Ni

Li gii: Gi tr k cn tm l k =

(p1)2
4

vi p l s l ( tr trng hp

p =2 ). Trc ht xt p = 2, trong trng hp ny ta cn k 2 2k =


(k 1)2 1 l mt s chnh phng dng trng hp ny khng th
xy ra v ch c duy nht hai s chnh phng lin tip l 0 v 1
Gi s p l s l. u tin ta xt trng hp k chia ht cho p, hay
k = np, khi k 2 pk = p2 n (n 1), n v n 1 l hai s nguyn t lin

tip. Do c hai khng th l s chnh phng.


Gi s k v p l hai s nguyn t cng nhau, khi k v k p cng

l hai s nguyn t cng nhau. k 2 pk l s chnh phng khi v


ch khi k v k p l cc s chnh phng, k = m2 , k p = n2 . Do

p = m2 n2 = (m + n) (m n). Suy ra m + n = p, m n = 1 v k =
2
hoc m + n = 1, m n = p v k = (p1)
4

(p+1)2
,
4

.3.20.Chng minh rng trong tt c cc t gic li c din tch bng


1, th tng di cc cnh v cc ng cho ln hn hoc bng

2 2+ 2

.c

Li gii: Thc t ta cn ch ra rng tng di cc cnh ca t gic

li ln hn hoc bng 4 v tng di cc ng cho ca t gic li

ln hn hoc bng 2 2. i vi trng hp ng cho ta s dng


cng thc tnh din tch A = 12 d1 d2 sin , vi l gc giu hai ng

cho. T gi thit cho din tch t gic bng 1 suy ra d1 d2 2, p

dng bt ng thc AG-GM suy ra d1 + d2 2 2, ng thc xy ra

khi v ch khi di hai ng cho bng nhau v vung gc vi


nhau.
i vi trng hp cnh ta s dng cng thc tnh din tch
A = (s a) (s b) (s c) (s d) abcd cos2
vi s =

a+b+c+d
,
2

B+D
,
2

B v D l hai gc i din nhau. T gi thit cho

din tch t gic bng 1 ta suy ra (s a) (s b) (s c) (s d) 1, li


s dng bt ng thc AG-MG ta li suy ra
4 (s a) + (s b) + (s c) + (s d) = a + b + c + d
, ng thc xy ra khi v ch khi a = b = c = d.

thi olympic Ty Ban Nha

15

T ta suy ra kt lun, c hai ng thc xy ra khi v ch khi


t gic li l hnh vung.
.3.21.Lng gas chnh xc mt chic t hon thnh mt vng ng
ua c t trong n bnh gas t dc ng ua. Chng minh rng
c mt v tr m xe c th bt u vi mt bnh gas rng, c th
hon thnh mt vng ng ua m khng s ht gas ( gi s xe c
th cha mt lng gas khng gii hn)
Li gii: Ta s dng phng php qui np theo n, trng hp n = 1
d dng thy c. Cho n + 1 bnh cha phi c mt bnh cha A m
t t c th ti c bnh cha B m bnh khng c gas
cho mt vng ua. Nu chng ta dn bnh B vo bnh A v b bnh
B i, theo gi thit quy np c 1 im xut pht m xe c th hon

.c

thnh vng ua, cng im xut pht nh th cho hon thnh vng
ua vi lng phn pht ban u ca bnh cha.

Chng 4

thi olympic i Loan

.c

.4.22.Cho a l mt s hu t, b, c, d l cc s thc v f : R [1; 1] l 1


hm tha mn:

f (x + a + b) f (x + b) = c[x + 2a + [x] 2[x + a] [b]] + d

vi mi x R. Chng minh rng hm f tun hon, tc l tn ti s


p > 0 sao cho f (x + p) = f (x) vi mi x R.

Li gii: Vi mi s nguyn n ta c:
f (x + n + a) f (x + n)
= c [x b + n] + 2a + [x b + n] 2 [x b + n + a] [b] + d
= c [x b] + n + 2a + [x b] + n 2 [x b + a] + n [b] + d
= c [x b] + 2a + [x b] 2 [x b + a] [b] + d
= f (x + a) f (x)
Ly s nguyn dng m sao cho am l mt s nguyn. Khi vi mi
s t nhin k ta c:

thi olympic i Loan

17

f (x + kam) f (x)
=

x X
m
X
j=1 i=1
m
X

=k

i=1

(f (x + jam + ai) f (x + jam + a (i 1)))

(f (x + ai) f (x + a (i 1)))

= k (f (x + am) f (x))
Do f (x) [1; 1], f (x + kam) f (x) b chn nn f (x + kam) f (x) phi
bng 0. Suy ra f (x + kam) = f (x), v vy f (x) l hm tun hon.

.4.23.Cho on thng AB. Tm tt c cc im C trong mt phng sao


cho tam gic ABC sao cho ng cao k t A v trung tuyn k t B
c di bng nhau.

Li gii: Gi D l chn ng cao k t A v E l chn ng trung


tuyn k t B. Gi F l chn ng vung gc k t E xung BC.

.c

Khi EF//AD v E l trung im ca AC, v vy EF = 1/2(AD) =


\ = /6 (tt c cc gc u c hng tr khi c ).
1/2(BE) v EBC
By gi, cho P l mt im sao cho B l trung im ca AP . Khi
\ v khng i. Qu tch tt c cc im
BE//P C, v th \
P CB = EBC
\
C sao cho P
CB khng i l mt ng trn. Do , qu tch cc

im C bao gm hai ng trn bng nhau, ct nhau ti B, P (Mt


tng ng vi gc /6 v mt tng ng vi gc /6 ). Trong trng

hp c bit, khi tam gic ABC cn ta thy rng mi ng trn u


c bn knh AB v tm sao cho ABQ = 2/3 (khng c hng).

.4.24.Cho s nguyn n 3, gi thit rng dy s thc dng a1 , a2 , ..., an

tho mn ai1 + ai+1 = ki ai vi dy k1 , k2 , ..., kn l dy s nguyn dng


bt k. (trong a0 = an v an+1 = a1 ). Chng minh rng
2n k1 + k2 + ... + kn 3n
Li gii: Bt ng
2n k1 + k2 + ... + kn

Nguyn Hu in, HKHTN H Ni

18

c chng minh da vo AM-GM vi ch rng:


n
X
ai
ai+1
k1 + k2 + ... + kn =
+
a
ai
i=1 i+1

chng minh c bt ng k1 + k2 + ... + kn 3n, ta cn chng

minh k1 + k2 + ... + kn 3n 2 vi n 2 , bng phng php quy np


theo n. Vi n = 2, nu a1 a2 th 2a2 = k1 a1 , v th hoc a1 = a2 v
k1 + k2 = 4 = 3.2 2, hoc a 1 = 2a2 v k1 + k2 = 4 = 3.2 2. Vi n > 2,
ta c th gi thit tt c cc ai khng bng nhau, khi tn ti i sao
cho ai aa1 , ai+1 m du bng khng xy ra t nht mt trong hai

trng hp. Khi ai aa1 + ai+1 < 2ai v do ki = 1. Ta kt lun rng


dy m b i s hng ai cng tha mn iu kin cho vi ki1 v

ki+1 gim i 1 n v v b i s hng ki . Theo gi thit quy np, tng


ca ki s hng nh hn hoc bng 3(n 1) 2, tng ca cc s ki ban

u nh hn hoc bng 3n 2, do ta c iu phi chng minh.

.4.25.Cho k = 22 + 1 vi n l s nguyn dng bt k. Chng minh

.c

rng k l mt s nguyn t khi v ch khi k l mt c ca 3(k1)/2 + 1.

Li gii: Gi s k l mt c ca 3(k1)/2 + 1. iu ny tng ng

vi 3(k1)/2 1( mod k). V vy 3k1 1( mod k). Vi d sao cho 3d 3


mod k. V vy, d khng l c ca (k 1)/2 nhng l c ca k 1,

mt khc (k 1) l c ca d nn d = k 1 (bi vi d phi nh hn k).


Do , k l s nguyn t.



Ngc li, nu k l s nguyn t k3 = k3 = 32 = 1

3(k1)/2 k3 1

.4.26.Cho t din ABCD. Chng minh rng:


(a) Nu AB = CD, AD = BC, AC =
ABC, ACD, ABD, BCD l cc tam gic nhn.

BDth cc tam gic

(b) Nu cc tam gic ABC, ACD, ABD, BCD c cng din tch th
AB = CD, AD = BC, AC = BD
Li gii: (a) Theo gi thit 4 mt ca t din bng nhau, ta c gc
tam din mi nh c to bi ba gc khc nhau ca mt mt.
Gi M l trung im ca BC. Theo bt ng thc trong tam gic,

thi olympic i Loan

19

AM + MD > AD = BC = 2MC. Cc tam gic ABC v DBC l bng


nhau, v th AM = DM. Do 2MD > 2MC; ngha l, MD ln hn
bn knh ca ng trn nm trn mt phng BCD vi ng kinh
BC. Do , D nm ngoi ng trn v gc BDC l gc nhn. Tng
t nh vy, ta chng minh c cc gc cn li (bi ton ny l bi
ton USAMO thng 2/1972, tham kho cc cch gii khc quyn
sch USAMO ca Klamkin).
(b) V AB v CD khng song song (v ABCD)l hnh t din), ta c
th chn hai mt phng song song l mt phng (P ) cha AB v (Q)
cha CD. Gi khong cch gia mt phng (P ) v (Q) l d. Gi A0 , B 0
ln lt l hnh chiu ca A, B trn (Q), gi C 0 , D 0 ln lt l hnh

chiu ca C, D trn (P ). V cc tam gic ACD v BCD c cng din


tch v chung y CD, nn chng c cng chiu cao h. Ta v hnh

tr vi trc CD v bn knh h; r rng A, B thuc hnh tr ny. Hai


im ny cng thuc mt phng (P ) v mt phng (P ) giao vi hnh

.c

tr ti mt hoc hai ng thng song song vi CD.


A v B khng th cng nm trn mt trong hai ng thng ny,
v th hnh tr v mt phng (P ) s giao nhau ti hai ng thng,

mt ng thng i qua A v mt ng thng i qua B. Hai ng


thng song song vi nhau v cch u ng thng C 0 D 0 mt khong

l (h2 d2 )1/2 . V vy, ng thng C 0 D 0 chia i on thng AB.


Tng t, ta chng minh c ng thng A0 B 0 chia i on thng

CD, qua php chiu, mt phng (Q) bin thnh mt phng (P ),


ng thng AB chia i on thng C 0 D 0 . AB v C 0 D 0 ct nhau
ti trung im mi ng nn AC 0 BD0 l hnh bnh hnh, do ta
c AC 0 = BD0 (gi khong cch gia chng l x) v BC 0 = AD 0 (gi
khong cch gia chng l y). Vy ta c:
1/2
1/2
AC = AC 02 + C 0 C 2
= x2 + d2
1/2
= BD02 + D 0 D 2
= BD
AC = BC 02 + C 0 C 2

1/2

= AD 02 + D 0 D 2

1/2

= y 2 + d2
= AD

1/2

Nguyn Hu in, HKHTN H Ni

20

S dng hai cp cnh khc lc u, ta cng c AB = CD


.4.27.Cho X l mt tp hp cc s nguyn cho bi cng thc:
a2k 102k + a2k2 102k2 + ... + a2 102 + a0
Vi k l mt s nguyn khng m v a2i {1, 2, ..., 9} vi i = 0, 1, ..., k.
Chng minh rng, mi s nguyn c dng 2p 3q , vi p, q l nhng s
nguyn khng m, chia ht mt phn t no ca X.
Li gii: Mi s nguyn khng chia ht cho 10 th u chia ht cho
mt s hng no ca X. Ta nhn thy rng lun tn ti mt phn
t trong Xc2p 1 ch s l bi ca 4p , vi mi s nguyn p khng

m. iu ny dn ti php quy np theo p: vi p = 0, 1 v nu x l


mt bi s vi p = k, th ta c th chn a2k sao cho x + a2k 102k 0(
mod 4k+1 ) v 102k 1( mod 4k )

By gi, ta chng minh rng bt k s nguyn n no khng chia


ht cho 10 th chia ht cho mt s hng no ca x. Gi n = 2p k

.c

vi k l. Khi vi b bn trn ta c th tm ra bi s ca 2p
trong tp hp X. Gi m l bi s v d l s ch s ca m v f =

10d+1 1. Bng h qu Euler ca nh l Fermat, 10f k ( mod f k) . V




vy m 10(d+1)(f k) 1 / 10d+1 1 chia ht cho 2p k v thuc X

.4.28.Xc nh tt c cc s nguyn dng k tn ti mt hm f :


N Z tha mn:

(a)f (1997) = 1998


(b) Vi mi a, b N, f (ab) = f (a) + f (b) + kf (gcd(a, b))

Li gii: Hm f nh trn tn ti vi k = 0 v k = 1. Ta ly a = b,
thay vo (b) ta c f (a2 ) = (k + 2)f (a). p dng hai ln ta c:
f (a4 ) = (k + 2)f (a2 ) = (k + 2)2 f (a)
Mt khc ta c:
f (a4 ) = f (a) + f (a3 ) + kf (a) = (k + 1)f (a) + f (a3 )
= (k + 1)f (a) + f (a) + f (a2 ) + kf (a)
= (2k + 2)f (a) + f (a2 ) = (3k + 4)f (a)

thi olympic i Loan

21

Chn a = 1997 f (a) 6= 0 suy ra (k + 2)2 = 3k + 4 c nghim k = 0; k =


1.
Vi k = 0 ta c:

f (pe11 ...penn ) = e1 g (p1 ) + ... + en g (pn )


vi m l c nguyn t ca 1997, g(m) = 1998 v g(p) = 0 vi mi s
nguyn t p 6= m.
Vi k = 1 ta c

f (pe11 ....penn ) = g (p1 ) + ... + g (pn )


.4.29.Cho tam gic ABC nhn vi O l tm ng trn ngoi tip v
bn knh R. AO ct ng trn ngoi tip tam gic OBC D, BO ct
ng trn ngoi tip tam gic OCA E, v CO ct ng trn ngoi

tip tam gic OAB F . Chng minh rng OD.OE.OF 8R3 .

.c

Li gii: Gi D 0 , E 0 , F 0 ln lt l giao im ca AO v BC , BO v CA,


CO v AB. Do chng l nh ca D, E, F qua php ca ng trn
ngoi tip tam gic ABC, Do OD 0 .OD = OE 0.OE = OF 0.OF = R2 . V

vy, bt ng thc ca bi ton tng ng vi:

AO BO CO
>8
OD 0 OE 0 OF 0

Gi h1 , h2 , h3 ln lt l khong cch t O n AB, BC, CA. Khi


AO/OD = [AOB]/[BOD 0] = (ABh1 )/(BD0 h2 ), tng t BO/OE 0 =

(BCh2 )/(CE 0 h3 ) v CO/OF 0 = (CAh3 )/(AF 0h1 ). Do ta c:


(AF 0 + F 0 B) (BD0 + D 0 C) (CE 0 + E 0 A)
AB.BC.CA
=
AF 0 .BD0 .CE 0
AF 0 .BD0 .CE 0
r
AF 0 .F 0 B.BD0 .D 0 C.CE 0 .E 0 A
>8
AF 0 .BD0 .CE 0
r
F 0 B.D0 C.0 A
=8
=8
AF 0 .BD0 .CE 0
Du bng xy ra khi v ch khi AF 0 = F 0 B, BD0 = D 0 C, CE 0 = E 0 A, hay
ABC l tam gic u.
.4.30.Cho X = 1, 2, ..., n vi n k 3 v Fk l mt tp con gm k phn

t ca X sao cho hai tp Fk bt k c nhiu nht k 2 phn t chung.

Nguyn Hu in, HKHTN H Ni

22

Ch ra rng tn ti mt tp Mk ca X c t nht [log2 n] + 1 phn t


khng c cha trong mt tp con no ca Fk .
Li gii: Nu k log2 n th iu phi chng minh hin nhin ng,
v th ta gi s k < log2 n. t m = [log2 n] + 1. Vi mi tp con k 1 s
hng ca tp Xthuc ti a mt tp con ca Fk v mi s hng ca
Fk bao gm cc tp con k(k 1) phn t. Ta c:

n
n
1
1

=
(Fk )
k
nk+1
k1
k
Mt khc, chn mt tp con m s hng bt k ca X, s s hng ca

(Fk )
n

k
+
1
k
k
n

k

.c

Fk l:

iu c th chng minh rng con s sau nh hn 1, v th mt

tp con m phn
t
no
khng
c cha s hng no ca Fk .
m
m
P

= 2m v ta cng c th chng minh


Hin nhin
i
k
i

3.2m3 vi m k 3 bng phng php quy np theo m. V


k
vy ta c:

m
3n
1

<1
nk+1
4 (n k + 1)
k

vi n 3. Ta c iu phi chng minh.

Chng 5
thi olympic Th Nh K
m

.5.31.Cho tam gic ABC vung ti A, gi H l chn ng cao k A.

.c

Chng minh rng tng bn knh cc ng trn ni tip cc tam gic


ABC, ABH, ACH bng AH.

Li gii: t a = BC, b = CA, c = AB v s =

a+b+c
.
2

Cc tam gic ABH

v ACH ng dng vi tam gic ABC vi t s tng ng a/c v b/c


p dng cng thc din tch tam gic bng bn knh ng

trn ni tip nhn vi na chu vi, suy ra bn knh cn tm l


ab
; a ab ; b ab v tng ca chng l abc = AH
a+b+c c a+b+c c a+b+c

.5.32.Dy s {an }
n=1 , {bn }n=1 c cho bi:

a1 = , b1 = , an+1 = an bn , bn+1 = an + bn vi mi n 1
C bao nhiu b s thc (, ) tha mn a1997 = b1 v b1997 = a1 ?


2
2
Li gii: Lu rng a2n+1 + b2n+1 = (2 + 2 ) an + bn , tr = = 0.
Chng ta cn 2 + 2 = 1. V vy c th t = cos , = sin , t
bng phng php quy np ta ch ra = cos n, = sin n. T c
k
, k = 1, 3, ..., 3997
1998 b s: (0; 0) v (cos ; sin ) vi = 3998

.5.33.Trong mt hip hi bng , khi mt cu th chuyn t i X c


x cu th sang i Y c y cu th, lin on nhn c y x triu la
t i Y nu y x nhng phi tr li x y triu la cho i X nu

Nguyn Hu in, HKHTN H Ni

24

x > y. Mt cu th c th di chuyn ty thch trong sut ma chuyn


nhng. Hip hi bao gm 18 i, tt c cc i u bt u ma
chuyn nhng vi 20 cu th. Kt thc ma chuyn nhng, 12 i
kt thc vi 20 cu th, 6 i cn li kt thc vi 16,16,21,22,22,23
cu th. Tng s tin ln nht m lin on c th kim c trong
sut ma chuyn nhng l bao nhiu?
Li gii: Chng ta tha nhn rng s tin ln nht kim c bi
khng bao gi cho php mt cu th chuyn n i nh hn. Chng
ta cng c th gi k lc bng mt cch khc. Mt i bng c x
cu th th c ghi l x trc khi giao dch mt cu th hoc x
trc khi nhn mt cu th v s tin m lin on kim c bng

tng ca cc s . By gi ta xem xt cc s c ghi bi mt i


m kt thc c nhiu hn 20 cu th. Nu s lng cu th ti a
ca i trong sut qu trnh chuyn nhng l k > n th cc s

.c

k 1 v k xut hin lin tip v b i 2 s th tng s tng ln. V


th tng ca cc s trong i bng t nht l 20 + 21 + ... + (n 1).

Tng t nh vy, tng ca cc s trong i bng kt thc c n < 20


cu th t nht l 20 19 ... (n + 1). V nhng con s ny chnh

xc l nhng con s c c bi vic lun chuyn cu th t i kt


thc t hn 20 cu th sang i kt thc c nhiu hn 20 cu th.

S sp xp dn n s tin kim c l ln nht. Trong trng


hp , tng l:

(20 + 20 + 21 + 20 + 21 + 20 + 21 + 22) 2(20 + 19 + 18 + 17) = 17


.5.34.Ng gic ABCDE li c cc nh nm trn ng trn n v,
cnh AE i qua tm ng trn . Gi s AB = a, BC = b, CD =
c, DE = d v ab = cd = 1/4. Tnh AC + CE theo a, b, c.
Li gii: Nu gi 2, 2, 2, 2 l cc cung chn bi cc cnh a, b, c, d
tng ng th:
a
AC = 2 sin ( + ) =
2
Tng t vi CD.

b2 b
1 +
4
2

a2
4

Tng qut vi R l bn knh ng trn ngoi tip ng gic, th th

thi olympic Th Nh K

25

AC 2 + BD2 = 1 m

AC = a R2 b2 + b R2 a2
Khi , dn n biu thc cha R2 di du cn v ta gii phng
trnh i vi R theo cc s a, b, c, d.
.5.35.Chng minh rng vi mi s nguyn t p 7, tn ti mt s
nguyn dng n v cc s nguyn x1 , x2 , ..., xn , y1 , y2 , ..., ym khng chia
ht cho p sao cho:
x21 + y12 x22 (modp)

x22 + y22 x23 (modp)

x2n + yn2 x21 (modp)

.....

.c

Li gii: Gi n l cp ca 5/3 mod p, v t xi = 3n1i 5i1 , yi =


43n1 5i1 th mi ng d trn l tng ng tr h thc cui cng.

H thc c dng 52n 32n (modp) (ng).Vy ta c iu phi chng


minh.

.5.36.Cho cc s nguyn n 2. Tm gi tr nh nht ca :

x51
x52
x5n
+
+ ... +
x2 + x3 + ... + xn x3 + x4 + ... + xn + x1
x1 + x2 + ... + xn1

Vi x1 , x2 , ..., xn l cc s thc tha mn x21 + x22 + ... + x2n = 1.


Li gii: t S = x1 + x2 + ... + xn
S dng BT Chebyshevs cho hai dy

xi
Sxi

v x4i (c hai dy u l

dy tng). Ta c:
 4


X x5
x2
xn
x1 + x42 + ... + x4n
x1
i

+
+ ... +
S xi
S x1 S x2
S xn
n
p dng bt ng thc hm li ta c:
x2
xn
1
x1
+
+ ... +

S x1 S x2
S xn
n1

Nguyn Hu in, HKHTN H Ni

26

p dng bt ng thc gi tr trung bnh ta c:


X 4 1/
2
x
i

X x2
i

1
n

Ta c kt lun:
X

x5i
1
1
1
n
. 2 =
S xi
n1 n
n (n 1)

.c

ng thc xy ra khi x1 = x2 = ... = xn =

1 .
n

Chng 6

thi olympic Ukraina

.6.37.Mt li hnh ch nht c t mu theo kiu bn c, v trong


mi c mt s nguyn. Gi s rng tng cc s trong mi hng v

.c

tng cc s trong mi ct l s chn. Chng minh rng tng tt c cc


s trong en l chn.

Li gii: Gi s cc mu t l v en, trong vung gc tri

trn l mu . (V tng tt c cc s trong li hnh ch nht l


chn nn iu cn chng minh cng tng ng vi tng cc

mu l s chn.)

Tng cc hng th nht, th ba, . . . (t trn xung), v cc ct th


nht, th ba, . . . (t tri sang) bng tng cc s trong cc mu en
tr i hai ln tng tt c cc s trong cc mu . V tng ny l
s chn nn tng cc s trong cc en l chn.

Nguyn Hu in, HKHTN H Ni

28

.6.38.Tm tt c cc nghim thc ca h phng trnh sau:

x +x ++x
1
2
1997 = 1997
x4 + x4 + + x4
= x3 + x3 + + x3 .
1

1997

1997

Li gii: Ta s chng minh h trn ch c nghim x1 = x2 = =


x1997 = 1.
t Sn = xn1 + + xn1997 . Theo bt ng thc lu tha trung bnh 1 ,

1/4
S4
S1
=1

1997
1997


S4
1997

1/4

S3
1997

1/3

S4
1997

1/3

S4
1.
1997
V vy bt ng thc lu tha trung bnh xy ra du bng, ngha l

.c

do

x1 = = x1997 = 1

.6.39.K hiu d(n) l s l ln nht trong cc c s ca s t nhin n.

v f (2n) = n +

f (2n 1) = 2n

Ta xc nh hm f : N N sao cho
2n
d(n)

vi mi n N

ln.

Tm tt c cc s k sao cho f (f (. . . f (1) . . .)) = 1997, f c lp k


Li gii: Ch c mt gi tr ca k l 499499.
Vi mi s chn (2a )b, b l s l v a 1,
f ((2a )b) = (2a1 )b +

(2a )b
= (2a1 )b + 2a = (2a1 )(b + 2).
b

Do ly tha cao nht ca hai trong f ((2a )b) bng mt na lu


tha cao nht ca hai trong (2a )b. V vy, ly lp a ln ca f ti (2a )b
c mt s l.
1

"power mean inequality" (trong bn ting Anh), tm dch l "bt ng thc lu

tha trung bnh" (ND)

thi olympic Ukraina

29

Ngoi ra, c l ln nht ca f ((2a )b) l b + 2, ln hn c l ln nht


ca (2a )b hai n v. V vy sau a ln lp f ti (2a )b, c l ln nht l
b + 2a. Do sau a ln lp f ta nhn c s l nn f a ((2a )b) phi bng
b + 2a.
Sau mt ln lp f ti 2n 1, n 1, ta nhn c 2n . t a = n
v b = 1. Ta thy phi lp thm n ln na mi nhn c s l, s
l, b + 2a = 2n + 1. V vy phi lp n + 1 ln cho f ti 2n 1 nhn
c s l tip theo, 2n + 1.
Ta d thy l khng c s l no xut hin hai ln khi lp f ti 1, bi
v dy cc s l nhn c t vic lp f ti bt k s nguyn dng
no cng l mt dy tng ngt. c bit, tn ti mt gi tr k sao cho

n(n+1)
1
2

(1) = 2n 1.

f (k) (1) = 1997, v s k l duy nht.


By gi ta chng minh bng quy np rng

.c

D thy iu ng ti n = 1. Sau gi s n ng vi n = k,
 k(k+1)

(k+1)(k+2)
1
2
(1) = f k+1 f 2 1 (1) = f k+1(2k 1) = 2k + 1
f
(1) = f 499499 (1) = 1997,

999.1000
1
2

V 1997 = 2(999) 1, ta c

nn k = 499499 tho mn yu cu bi, v do lp lun trn, k l

duy nht.

.6.40.Hai ng gic u ABCDE v AEKP L trong khng gian sao cho


\ = 60o . Chng minh rng hai mt phng ACK v BAL vung gc.
DAK

T
L
E

D
O

C
B

Nguyn Hu in, HKHTN H Ni

30

Li gii: Nu ta quay AEKP L quanh trc AE, bt u v tr trng


\ tng cho n khi AEKP L li nm
nhau vi ABCDE, th gc DAK
trn mt phng cha ABCDE. (iu ny c th nhn c bng
cch tnh ton thy tch v hng ca vect AD v AK tng trong
mt khong xc nh). V vy c mt gc duy nht gia cc mt
\ = 60 . Tht vy, iu ny xy ra khi cc
phng sao cho gc DAK
hnh ng gic l hai mt ca khi mi hai mt u 2 (do tnh cht
i xng, tam gic DAK l u trong trng hp ny). c bit, mt
phng BAL l mt phng cha mt th ba qua nh A ca khi mi
hai mt u, ta gi mt l BALT O.
S dng thm du phy sau tn im k hiu hnh chiu tng

ng ca cc im xung mt phng ABCDE. Khi K 0 nm trn


\ Lp h trc ta sao cho ABCDE
ng phn gic ca gc DEA.

nm trn mt phng Oxy theo chiu kim ng h, v K 0 E cng chiu

vi chiu dng ca trc Ox. Xt hai vect EK v EA. Vect th nht

.c

khng c thnh phn y, v vect th hai khng c thnh phn z. Do


tch v hng ca hai vect bng tch cc thnh phn x ca

chng. V AE v K 0 E to vi nhau gc 54 , EA c thnh phn x c

tnh bi AE(cos 54 ) = cos 54o . EK c thnh phn x l K 0 E. Do



tch v hng ca chng bng K 0 E(cos 54 ). Mt khc EK.EA =
\ = (1).(1). cos(108 ) = cos 72 . Lp phng trnh t
EK.EA. cos KEA
hai kt qu trn ta nhn c K 0 E =

cos 72
cos 54

Cc tnh ton trn vect sau y s chng t K 0 A v AC vung gc:

K 0 A.AC = (K 0 E + EA).AC
= K 0 E AC cos 54 CA AC cos 72
=

cos 72
cos 54 cos 72 = 0
cos 54

V BO 0 vung gc vi AC v BO song song vi AT , nn AT 0 vung gc


vi AC. V vy cc im K 0 , A, T 0 thng hng. V K, A, T khng thng
hng nn mt phng AKT vung gc vi mt phng cha ABCDE.
Quay xung quanh A, ta suy ra mt phng ACK vung gc vi mt
2

khi thp nh din u (ND)

thi olympic Ukraina

31

phng cha BALT O, t c iu phi chng minh.


.6.41.Cho phng trnh ax3 + bx2 + cx + d = 0 c ba nghim thc phn
bit. Hi phng trnh sau c bao nhiu nghim thc:
4(ax3 + bx2 + cx + d)(3ax + b) = (3ax2 + 2bx + c)2

Li gii: t P (x) = ax3 + bx2 + cx + d v Q(x) = 2P (x)P 00(x) [P 0 (x)]2 ,

ta s m s nghim thc ca Q(x) = 0. Ta c th thc hin mt s


php rt gn m khng thay i s nghim thc ca Q(x). Trc tin
ta chia P cho mt h s t l sao cho a = 1. Sau ta tnh tin x sao
cho nghim ng gia ca P l 0, tc l d = 0 v c < 0. Khi :

Q(x) = 3x4 + 4bx3 + 6cx2 c2 .

.c

By gi ta p dng quy tc du Descartes m s nghim ca


Q(x). Du ca Q(x) l +, s, , v ca Q(x) l +, s, , , s l

tc l Q c tt c hai nghim thc.

du ca b. Trong trng hp no th mi dy trn cng ch c ng


mt ln i du. V vy Q c mt nghim m v mt nghim dng,

.6.42.K hiu Q+ l tp tt c cc s hu t dng. Tm tt c cc hm


s f : Q+ Q+ sao cho vi mi x Q+ :

(a) f (x + 1) = f (x) + 1
(b) f (x2 ) = f (x)2 .

Li gii: Ch c mt hm s tho mn l f (x) = x. T (a), f (x + n) =


f (x) + n vi mi n nguyn dng. t x = pq vi p, q nguyn dng. Ta
c:
2 
 2
 2
 

p
p
p
p + q2
2
= q+f
+f
= q + 2qf
.
f
q
q
q
q
Mt khc,
f

p + q2
q

2

=f
=f

(p + q 2 )2
q2

p2
q + 2p + 2
q
2

 2
p
= q + 2p + f
.
q
2

Nguyn Hu in, HKHTN H Ni

32

T hai phng trnh trn suy ra


 
p
= 2p
2qf
q
p
p
v vy f ( ) = .
q
q
.6.43.Tm s nguyn n nh nht sao cho vi mi n s nguyn tu , tn
ti 18 s nguyn trong c tng chia ht cho 18.
Li gii: Gi tr nh nht ca n l n = 35; tp hp 34 phn t gm 17
s khng v 17 s mt cho ta thy rng n 35. V vy ta ch cn phi
chng minh rng vi 35 s nguyn bt k lun tm c 18 s trong
c tng chia ht cho 18. Thc ra ta s chng minh rng vi mi n,

trong 2n 1 s nguyn lun tm c n s c tng chia ht cho n.


Ta s chng minh khng nh trn bng quy np theo n. D thy

iu ng vi n = 1. Nu n l hp s, vit n = pq, ta c th ly ra
tp p s nguyn m tng chia ht cho p cho n khi cn li t nht

.c

2p 1 s; ta c 2q 1 tp nh vy, v li theo gi thit quy np, c q


tp trong s c tng (ca pq s) chia ht cho q.
By gi gi s n = p l s nguyn t. S x chia ht cho p nu v ch

nu xp1 6 1 (mod p). V vy nu khng nh trn sai th tng cc


s dng (a1 + + ap )p1 trn mi tp con {a1 , . . . , ap } ca cc s

p1
cho ng d vi C2p1
1 (mod p). Mt khc, tng ca cc s dng
ep
e1
a1 ap vi e1 + + ep p 1 lun chia ht cho p: nu k p 1 cc

pk
s ei khc khng th mi tch c lp li C2p1k
ln, v cui cng
l mt bi ca p. S mu thun chng t khng nh trn l

ng trong trng hp ny. (Ch : chng minh bi ton cho,


cn phi chng minh trc tip cho trng hp p = 2, 3.)
.6.44.Cc im K, L, M, N

nm trn cc cnh AB, BC, CD, DA

ca hnh hp (khng cn l hnh hp ng) ABCDA1 B1 C1 D1 .


Chng minh rng cc tm mt cu ngoi tip ca cc t din
A1 AKN, B1 BKL, C1 CLM, D1 DMN l cc nh ca mt hnh bnh hnh.
Li gii: a vo h ta vi ABCD song song vi z =
0. Gi E, F, G, H l tm ng trn ngoi tip cc tam gic

thi olympic Ukraina

33

AKN, BKL, CLM, DMN v gi W, X, Y, Z l tm mt cu ngoi tip


cc t din A1 AKN, B1 BKL, C1 CLM, D1 DMN. Vi mi im Q ta k
hiu Q1 , Q2 , Q3 l cc ta x, y, z ca Q.
K

C
D

B1

A1

D1
Trc tin ta s chng minh EF GH l hnh bnh hnh, bng cch
chng minh trung im ca EG v F H trng nhau. Ch cn chng

.c

minh iu vi hnh chiu ca cc on thng trn hai phng


khc nhau (chng hn, xy dng h trc theo cc phng ).

Nhng r rng vi php chiu trn AB, v E v F chiu tng ng


xung trung im ca AK v BK, nn on thng gia chng c

chiu di AB
, tng t vi CD. Ta cng c lp lun nh vy vi php
2
chiu trn CD.
By gi ta c E1 + G1 = F1 + H1 v E2 + G2 = F2 + H2 . Hn na, v

W v E cch u AKN, W E vung gc vi AKN nn vung gc vi


mt phng z = 0. Vy W1 = E1 v W2 = E2 , tng t vi X, Y, Z. Vt
W1 + Y1 = X1 + Z1 v W2 + Y2 = X2 + Z2 . Ta ch cn phi chng minh
W3 + Y3 = X3 + Z3 . Ch rng c W v X u nm trn mt phng
vung gc vi ABB1 A1 v i qua trung im ca AA1 v BB1 . Do
W3 = aW1 +bW2 +c v X3 = aX1 +bX2 +c vi a, b, c l cc hng s. Tng
t, Y v Z u nm trn mt phng vung gc CDD1 C1 v qua trung
im ca CC1 , DD1 . V DCC1 D1 song song v bng ABB1 A1 , ta c
Y3 = aY1 + bY2 + d v Z3 = aZ1 + bZ2 + d vi d l mt hng s khc, cn
a, b l cc hng s cng thc trn. Bi vy W3 + Y3 = X3 + Z3 , t
hon thnh chng minh rng W XY Z l hnh bnh hnh.

Chng 7
thi olympic Anh
m

.7.45.Gi s M v N l hai s nguyn dng c 9 ch s c tnh cht l

.c

nu bt k ch s ca M c thay bi ch s ca N tng ng th ta
c mt bi ca 7. Chng minh rng vi bt k mt s t c bng

cch thay mt ch s ca N tng ng bi mt ch s ca M cng l


mt bi ca 7.

Tm mt s nguyn d > 9 sao cho kt qu trn vn cn ng khi M v


N l hai s nguyn dng c d ch s.
P
Li gii: Kt qu ng vi bt k d 2 ( mod 7). Vit M = k 10k , N =

nk 10k , y mk , nk l cc ch s. Th vi bt k k, 10k (nk mk ) 0 M (


mod 7). Ly tng theo k, chng ta c M N dM 2M ( mod 7),

v vy N M ( mod 7), c 10k (mk nk ) N ( mod 7). Vy khi


thay bt k ch s trong N bi ch s tng ng trong M chng ta
t c mt s chia ht cho 7.

.7.46.Trong tam gic nhn ABC, CF l mt ng cao, vi F trn AB,


v BM l mt trung tuyn, vi M trn CA. Cho BM = CF v MBC =
F CA, chng minh rng tam gic ABC l u.
Li gii: Gi s ACF = CBM = A, v gi s CM = AM = m. Th
MB = CF = 2m cos A. Theo nh l hm s Sin,
MB
CM
=
,
sin CBM
sin MCB

thi olympic Anh

35

v v vy sin MCB = 2 cos A sin A = sin 2A.


iu ny a n hai kh nng. Nu MCB +2A = 180, th CMB =
A = MBC. Khi CB = MC v MB = 2MC sin A. Cng c MB =
CF = AC cos A = 2MC cos A. Do sin A = cos A v vy A 45

MCB 90 , mu thun.
V vy chng ta c MCB = 2A, v vy ACF = BCF . Do tam

gic ACF ng dng vi tam gic CBM, v vy CAF = BCM. Do


BC = AB, vy tam gic ABC l u.
.7.47.Tm s cc a thc bc 5 vi cc h s khc nhau t tp
{1, 2, . . . , 9} m chia ht cho x2 x + 1.

Li gii: Cho phng trnh bc 5 l ax5 + bx4 + cx3 + dx2 + ex + f = 0.


Cc nghim ca x2 x + 1 khng phi l cc nghim thc ca x3 + 1,
l ei/3 v e5i/3 . Do a thc bc 5 l chia ht cho x2 x + 1 nu

v ch nu

.c

ae5i/3 + be4i/3 + cei + de2i/3 + eei/3 + f = 0.

Ni cc khc, v vy i sin 60(a b + d + e = 0), hoc a d = e b v

a/2 b/2 c d/2 + e/2 + f = 0, hay e + 2f + a = b + 2c + d hoc (v


a d = e b) a d = c f = e b. iu ny ko theo 1/12 ca a thc

s c cc h s p + k, q, r + k, p, q + k, r vi k > 0 v p q r.
3
Vi k cho, c C9k
cc gi tr ca p, q, r sao cho r + k 9. Tuy nhin,

cc h s phi khc nhau, nn chng ta phi tr i. C 9 2k cch


la chn 2 s khc nhau theo k, v 7 k cch la chn cc s cn

li. Tuy nhin, chng ta m c cc s dng x, x + d, x + 2d sinh


i, v c 9 2k s.
Do , vi k cho trc, chng ta c
3
C9k
(9 2k)(7 k) + 9 3k

a thc. Cng li, c (1+4+10+20+35+56)(42+25+12+3)+(3+6) = 53


a thc dng trn, v 53.12 = 636 cc a thc tt c.
.7.48.Tp S = {1/r : r = 1, 2, 3, . . .} cc s nguyn dng nghc o lp
thnh cp s cng vi di ty . Chng hn, 1/20, 1/8, 1/5 l mt

Nguyn Hu in, HKHTN H Ni

36

cp s nh vy, vi di l 3 vi phng sai l 3/40. Hn na, c


mt cp s cc i trong S vi di 3 v n khng th m rng thm
v bn phi v bn tri ca S (1/40 v 11/40 khng l phn t ca S).
(a) Tm mt cp s cc i trong S c di 1996.
(b) C hay khng mt cp s trong S c di 1997?
Li gii: C mt cp s cc i c di n, vi mi n > 1. Theo nh
l Dirichlet ko theo rng c mt s nguyn t p c dng 1 + dn vi
mi s nguyn dng d. By gi xt cp s
1+d
1 + (n 1)d
1
,
,...,
.
(p 1)! (p 1)!
(p 1)!

V cc mu s chia ht t s, nn mi phn thc l mt s nguyn


nghc o, nhng vi (1 + nd)/(p 1)! = p/(p 1)! khng phi v p l

.c

s nguyn t. Do c dy cp s cc i. ( gii (a), n gin ly


p = 1997.)

You might also like